EXAM Master Infectious Disease

अब Quizwiz के साथ अपने होमवर्क और परीक्षाओं को एस करें!

Case A 10-year-old boy presents with a 1-week history of progressing joint pain. The pain started in his ankles, and then progressed to his knees; his hips are now starting to hurt. His ankles feel slightly better. He had contact with someone who had strep throat within the past couple of weeks. The patient's heart rate is 130. On exam, there is erythema and edema over the knees and hips as well as minimal edema over the ankles. A high-pitched holosystolic murmur is noted over the apex and radiates to the axilla with a noted friction rub. Question Based on the above history and physical exam, what is the most likely diagnosis?

Correct answer: Acute rheumatic fever Explanation Acute rheumatic fever is the correct response; the scenario identifies 2 major Jones criteria: migratory polyarthritis and carditis. Arthritis with acute rheumatic fever typically starts in the legs and then migrates; most commonly, it affects the large joints. The pain can subside within 1 - 2 days, and the arthritis starts affecting another joint. Swelling and redness are common over the joints involved. Typically, a mitral or aortic regurgitation murmur is noted on exam. A friction rub indicates involvement of the myocardium. Polyarthritis and carditis are the most common major Jones criteria for acute rheumatic fever. A confirmed group A Streptococcus test needs to be done, but with the sick contact, you can safely assume the patient has strep. Juvenile idiopathic arthritis is incorrect; it would not explain the tachycardia or other abnormal cardiac physical exam findings. The eyes can be involved with juvenile idiopathic arthritis, which was not the case with the above patient. Systemic lupus erythematosus is incorrect; based on his history, the patient does not have the 4 required criteria from the American College of Rheumatology necessary to make the diagnosis. No rash was noted on exam. Nothing was stated about pleuritis, anemia symptoms, seizures, or positive ANA. Kawasaki disease is incorrect because there was nothing stated about a long-lasting high fever, conjunctivitis, rash, 'strawberry' tongue, redness on palms and soles, and/or swollen lymph nodes. Joint pain can present during the 2nd phase of Kawasaki's disease, but based on the overall scenario, Kawasaki's disease is an unlikely answer. Septic arthritis is incorrect because typically only one joint is involved and this would not explain the cardiac symptoms. Children can usually bear weight on the extremity involved.

Case A 13-year-old girl presents to the emergency department with complaints of febrile episodes (Tmax 102°F), joint aches in her knees and wrists, chest pain and a raised, red rash. She denies sexual activity or intravenous drug use. Vital signs are BP 90/60mmHg, HR 115/min, T 101°F, RR 25/min. Her physical exam is remarkable for diffuse, scattered ring shaped macules on her extremities that is consistent with erythema marginatum, a III/VI systolic ejection murmur, and guarded passive range of motion in wrists and knees bilaterally with no apparent swelling. Laboratory findings are as follows; WBC 16,000mcL, Hematocrit 35%, Platelets 350,000mcL, ESR greater than 15mm/h and a positive antistreptolysin O titer. Question What should you suspect as this patient's diagnosis?

Correct answer: Acute rheumatic fever Explanation Acute rheumatic fever is the correct answer because this patient demonstrates 3 major Jones criteria (polyarthritis, murmur consistent with mitral regurgitation, and erythema marginatum). She also has several minor Jones criteria (fever, elevated ESR, and antistreptococcal antibodies evidenced by the positive antistreptolysin O titer) (1). Still's Disease is incorrect as this is a diagnosis of exclusion and the patient does not exhibit significant anemia or leukocytosis. She also does not exhibit the type of rash that accompanies Still's Disease (salmon colored rash present on the chest and abdomen) (2). Septic arthritis is incorrect as affected joint(s) are usually swollen. She also doesn't have any major risk factors for developing septic arthritis. Systemic lupus erythematosus is incorrect as the patient does not exhibit a malar rash or discoid lesions. She also does not exhibit anemia or leukocytosis, which can be found frequently in those with SLE. Even though she does exhibit some subjective complaints that could be construed as SLE, more labwork needs to be drawn (autoantibodies) to diagnose SLE properly (2). Lyme disease is incorrect as the rash she has doesn't represent the characteristic bullseye appearance, the rash known as erythema migrans. Lyme disease is also not associated with a murmur or a positive antistreptolysin O titer.

Case A 4-year-old boy accompanied by his mother presents with fever, sore throat, muffled voice, and breathing and swallowing difficulty. The child is leaning forward with his head and nose tilted upward and forward. He is irritable, with moderate respiratory distress and inspiratory stridor. Vitals are as follows: pulse is 94/min; BP is 110/70 mm Hg; and temperature is 101 F. Direct fiberoptic laryngoscopy is performed and shows an edematous larynx. Cultures are taken, and an endotracheal tube is placed. The epiglottis cultures reveal Haemophilus influenzae, and the diagnosis of acute Haemophilus influenzae epiglottitis is made. The mother is worried about her second 1-year-old child living in the same house; the 1-year-old child is not vaccinated for H. influenzae. Question What prophylactic measure would you recommend at this time?

Correct answer: All family members, including the patient, should receive prophylactic rifampicin. Explanation The correct answer is that all family members, including the patient, should receive prophylactic rifampicin to make sure that both the person with the illness and the rest of the household members have the bacteria completely eradicated from their bodies to prevent formation of a "carrier state", in which a person is not actively sick, but has the bacteria and can spread it1. Chemoprophylaxis should be instituted as rapidly as possible2. Administering prophylactic rifampicin to the susceptible child alone or adequately treating the affected child is not sufficient. Active immunization with vaccine does not help when immediate prophylaxis is warranted. Children should receive the first dose of the vaccination at 2 months of age, with 2nd dose at 4 months, 3rd dose at 6 months (this is dependent on the brand of the vaccine) and booster dose between 12 and 15 months.

Case A 21-year-old man presents with acute onset of pleuritic chest pain accompanied by 2-3 days of fever, chills, arthralgias, and myalgias. Upon further questioning, the patient notes that 4 weeks ago he had a severe sore throat and fever, but he was not evaluated for these symptoms. Physical examination reveals a febrile patient in mild distress. A systolic murmur is noted in the left 4th/5th intercostal space that radiates to the left axilla. A friction rub is also appreciated on exam. Laboratory results reveal an elevated erythrocyte sedimentation rate (ESR) and antistreptolysin antibodies. Question What is the most appropriate initial management of this patient?

Correct answer: Aspirin Explanation Aspirin is the correct response. The patient in the above scenario most likely has acute rheumatic fever. Rheumatic fever is a systemic immune process that is often a complication of undertreated b-hemolytic streptococcal infection of the throat, resulting in inflammatory changes in the heart, skin, joints, and other tissues. It is less common in the United States than in underdeveloped countries; however, there have been new reports of outbreaks in several regions of the US. Clinical presentation of acute rheumatic fever is often an acute febrile illness and a migratory polyarthritis of the larger limb joints. Cardiac symptomatology is less common but can manifest as murmurs or a friction rub, indicating valvular and pericardial involvement respectively. The diagnosis of acute rheumatic fever is based on clinical features plus evidence of a preceding streptococcal infection. The Jones criteria for diagnosis of acute rheumatic fever were developed in 1944 and most recently updated in 1992. The presence of 2 major criteria - or one major and two minor criteria - establishes the diagnosis. Major criteria include:

Case A 50-year-old man presents with multisystem failure secondary to bilateral pneumonia. Four days prior, he presented to his primary care physician with fever, cough, and malaise; he was given a broad-spectrum antibiotic, and he progressively became worse over the course of the antibiotic treatment. His body temperature is 40°C, respiration is 35/min, and pulse is 100/min. Laboratory examination is significant for impaired liver and renal function. Patient history is significant for the use of corticosteroids. A sputum culture is performed and is significant for the presence of WBC in the Gram stain, but there are no organisms present. The culture result is negative. A urinary antigen test is ordered for Legionella pneumophila and the result is positive. Question What is the drug of choice in treating this patient?

Correct answer: Azithromycin Explanation Azithromycin is a macrolide antibiotic that is highly effective in the treatment of Legionella infections. Legionella is an intracellular pathogen; therefore, antibiotics used to treat it must be able to achieve high intracellular concentrations. Azithromycin is more effective than erythromycin, an alternative therapy. The quinolone antibiotics may also be used to treat Legionella infections and are preferred in the management of transplant patients. Metronidazole was first introduced in 1959 as an effective treatment for Trichomonas vaginalis infections. Since then, it has been found to be an effective antibiotic in the treatment of anaerobic infections, various parasitic infections, Treponema pallidum, and Campylobacter fetus. Resistance to metronidazole is rare to develop. The antibiotic enters the cell of the bacteria where a reduction of the nitro group of the drug results in the production of compounds that are toxic to the cell wall. It is considered the first drug of choice in the treatment of Clostridium difficile infections. Vancomycin was first isolated from Streptomyces orientalis. It was introduced in 1958 for the growing problem of penicillin-resistant staphylococci. By 1960, it was superseded by methicillin. In the last 10 years, the use of vancomycin has significantly increased due to the emergence of methicillin-resistant Staphylococcus aureus (MRSA), Clostridium difficile, and enterococcal disease. The antibiotic complexes to the D-alanyl-D-alanine portion of the growing cell wall and inhibits peptidoglycan and transpeptidation. Vancomycin is poorly absorbed orally and thus is given by IV. It is widely distributed throughout the body. Adverse reactions are not common; however, ototoxicity is a serious concern and blood levels must be monitored. Vancomycin is the second drug of choice for the treatment of Clostridium difficile infections. The mechanism of action of penicillins is through penicillin-binding protein (PBP) binding with subsequent inhibition of transpeptidation reaction and activation of the autolytic enzymes. Administration is orally and IM or IV. Food interferes with absorption, so it should be given 1 hour before or 2-3 hours after meals. Penicillin is widely distributed in the body, but there is poor penetration in some spaces such as the eyes, pericardium, and cerebral spinal fluid. The antibiotic is considered non-toxic, but it can cause allergic reactions in 3-10% of the patient population. Penicillin is most active against Streptococcus spp., Neisseria gonorrhoeae, Treponema pallidum, and Listeria spp. It is used clinically to treat syphilis, Streptococcus pyogenes pharyngitis, and as a prophylactic agent after rheumatic heart disease. Legionella frequently produces β-lactamase, so penicillin is not a good choice for treatment of Legionella. Cefazolin is a first-generation cephalosporin. It is effective in the treatment of gram-positive infections such as staphylococcus and streptococcus, and it is frequently used as prophylaxis against surgical site infections. Legionella frequently produces β-lactamase, so cefazolin is not a good choice for treatment of Legionella.

Case A 15-year-old girl with no record of any significant illness in the last 2 years presents after a 5-day history of fever. She has no cough, her lungs are clear, but she has a fever of 38.2°C. There is the presence of a significantly large circular rash on her abdomen. Her history is notable for having recently returned from spending the early summer at her grandparent's cottage on the shoreline of Connecticut. She spent most of her time hiking through the meadows and collecting photographs. She submits a jar containing an insect that she found feeding on her body. You analyze the insect microscopically (refer to the image). Question A CBC is ordered and the results are unremarkable. Based on the clinical history and presentation, you should order tests for what infection?

Correct answer: Borrelia burgdorferi Explanation Borrelia burgdorferi is a spirochete, and it is the etiologic agent of Lyme disease. The organism is transmitted to the host by the bite of a tick (Ixodes scapularis). The spirochete produces symptoms in the host such as headache, fever, arthritis-like pain in the joints, and a variety of neurological symptoms. Typically, at the site of the tick bite and introduction of the spirochete, a characteristic rash develops; it is target-like in appearance with expanding borders. Only the nymph and adult stage can transmit the spirochete. Co-infection with Babesia microti and/or Ehrlichia chaffeensis is common. Francisella tularensis is coccoid to ellipsoid in shape and very small. It is Gram-negative and non-motile, appearing singly. The organism is a strict aerobe. The organism is the causative agent of tularemia (rabbit fever, deerfly fever, market men's fever). The onset of the disease is sudden and includes flu-like symptoms, as well as characteristic ulcer with regional lymphadenopathy. The source of infection is the exposure to infected wild and domestic animals or their products, contaminated water, ingestion or inhalation, and blood-sucking arthropods. The organism is highly infectious, and it is the third most commonly reported laboratory associated infection. Direct isolation is best done by culturing the ulcer, pharynx, conjunctiva, lymph node biopsy, and gastric washing. Rickettsia rickettsii is a rickettsial organism that is transmitted by the bite of a tick that causes Rocky Mountain Spotted Fever. The most common tick to carry Rickettsia rickettsii is Dermacentor variabilis (common dog tick). Common initial symptoms present 2-14 days after the tick bite. These symptoms include a macular rash that initially starts on the palms and lower extremities (eventually spreading to cover the entire body), purpura, emesis, myalgias, fever, diarrhea, hypotension, and DIC. If the disease is not treated properly without delay, it is not uncommon for the patient to develop DIC and shock, with death as the eventual outcome. When Rickettsia rickettsii infection is suspected, the antibiotics of choice are Tetracycline or chloramphenicol. Ehrlichia chaffeensis, an obligate intracellular Gram-negative species of rickettsiales bacteria, causes ehrlichiosis (also known as "Rocky Mountain spotted fever"). The organism infects mononuclear cells, mainly monocytes and macrophages. Patients generally present with a febrile illness and complaints of headache, myalgias, malaise, and fever. Patients also present with elevated liver enzymes and a leukopenia that can cause anemia. The organism will produce characteristic cytoplasmic inclusions; they can be seen by microscopic examination of peripheral blood or bone marrow smears. Babesia microti is the etiological agent that causes babesiosis. Initial symptoms of babesiosis usually begin gradually and mimic those of influenza. There is malaise, aches and joint pain, headache, shaking chills with fever, abdominal pain, nausea, vomiting, fatigue, and anorexia. There is also dark urine, photophobia, and psychological symptoms. Adult respiratory distress syndrome is not uncommon. The destruction of RBCs produces anemia, thrombocytopenia, proteinuria, and hemoglobinuria. BUN, serum creatinine, and bilirubin can be elevated. Liver enzymes can also be elevated. Diagnosis can be made by the observation of characteristic ring forms in the red blood cells of a Giemsa-stained peripheral smear. The clinical history and lack of gametocyte forms on the peripheral smear help eliminate plasmodium as an identification. Most cases of babesiosis have occurred in the northeastern coastal region of the United States, particularly Cape Cod, Block Island, Nantucket, Martha's Vineyard, Eastern Long Island, Shelter Island and Fire Island, and the Connecticut mainland. Cases have also occurred in Washington State. Deer are a necessary host, and the White Footed Mouse acts as a reservoir. The Deer Tick nymph (Ixodes scapularis) is the primary vector.

Case A 24-year-old man presents with severe headache, fever and chills, fatigue, and pain in his joints. His wife states that he has 'the flu'. During the physical exam, his doctor discovers a rash on the patient's thighs and arms. The patient says he has had it for 'a while'. The rash consists of large red indurated lesions with bright red outer borders and pale centers. The patient believed it to be poison ivy or sumac, because last month he and his wife spent the July 4th weekend with his brother camping and hiking in the Catoctin Mountains of Northern Maryland. Question What microorganism is probably causing this illness?

Correct answer: Borrelia burgdorferi Explanation Lyme disease is the most common tick-borne disease reported in the United States. Borrelia burgdorferi, the etiological agent, is transmitted by Ixodes spp. ticks. B. Burgdorferi has been isolated from ticks in several locales in the southeast and the northeast, as well as in Wisconsin and Minnesota. After an incubation of up to 30 days, the first symptom is usually the development of a characteristic annular lesion, most often with a red border and partial central clearing. Within a few days, the rash spreads and flu-like symptoms develop. Lyme disease can be confirmed serologically after the first several weeks of infection - a positive antibody response to the organism can be determined by ELISA. S. Aureus can cause impetigo and S. pyogenes is implicated in erysipelas. P. Aeruginosa can cause folliculitis in patients, who have been swimming in warm water. C. immitis can also involve skin lesions from which it can be cultured. Only in Lyme disease can the clinician find the unusual skin lesions with 1 or more bright red rings, along with headache, neck stiffness, fever and chills, arthralgias, and profound malaise and fatigue.

Case A 28-year-old woman recently completed inpatient chemotherapy for acute myelogenous leukemia; she presents with mouth pain. Examination of the mouth reveals white patches; when rubbed off, they reveal a red base. Question What is the most likely diagnosis?

Correct answer: Candidiasis Explanation Oral candidiasis (or thrush) presents with painful white plaques on erythematous mucosa that can easily be rubbed off. It is common in immunocompromised patients. The above patient has recently completed chemotherapy, which places her at risk for the development of thrush. Leukoplakia also presents with white oral lesions, but the lesions cannot be removed on examination. Erythroplakia may also be associated with white lesions; however, as is the case with leukoplakia, the plaque cannot be removed on examination. Oral lichen planus may present similarly to leukoplakia; it cannot be removed on examination. Oral cancer has a variety of presentations, including the presence of white lesions. Oral cancer lesions cannot be removed on examination.

Question What clinical condition is associated with the human papilloma virus?

Correct answer: Cervical cancer Explanation The majority of cervical cancers (approximately 90%) contain human papilloma virus DNA, usually of type 16 or 18. Human papilloma viruses are members of the family Papovaviridae and have predilection for the skin and mucous membranes. Human papilloma viruses are associated with warts, respiratory papillomas, oral infections, and genital infections. Human papilloma viral infection may progress over a period of years through the various stages of cervical intraepithelial neoplasia to invasive squamous carcinoma. Hepatitis B virus (HBV) is a human carcinogen associated with hepatocellular carcinoma. The incidence of hepatocellular carcinoma is higher in a person who becomes infected with HBV earlier in life. The pattern observed is: person develops chronic hepatitis leading to cirrhosis of the liver and eventually to liver cancer 20-50 years post-infection. The genome of HBV is small and comprises a small, circular, partially double-stranded DNA molecule. HBV replicates in hepatocyte and involves RNA intermediate and a virus coded reverse transcriptase. HBV can become integrated into the cellular chromosome during chronic infection, and it may promote genetic instability in the cell. Dengue fever virus and Powassan virus are members of the family Flaviviridae. Flaviviruses are enveloped viruses with icosahedral symmetry. The genome consists of a linear single-stranded RNA molecule, and replication occurs in the cytoplasm. Dengue fever virus is transmitted by the mosquito and is associated with bone break fever and dengue shock syndrome. Bone break fever is characterized by headache, myalgia, arthralgia, and rash. Secondary exposure can result in dengue shock syndrome, and it is characterized by gastrointestinal hemorrhage. Herpes simplex virus type 1 (HSV-1) is a member of the Herpesviridae family and consists of a double-stranded DNA genome. Primary infection with HSV-1 mostly involves the mouth and/or throat. Gingivostomatitis is a classic clinical presentation of HSV-1 infection. It is characterized by formation of vesicles on the mouth and gums, which rupture to become ulcers. Coxsackievirus type A and coxsackievirus type B are members of the Picornaviridae family. Picornaviruses are naked viruses with an icosahedral nucleocapsid. The genome consists of single molecule of single stranded RNA. Coxsackievirus type A is associated with herpangina, which is commonly seen in children. Herpangina is a severe febrile pharyngitis characterized by vesicles or nodules primarily on the soft palate.

Case A 3-month-old girl presents with rapid breathing and a staccato cough, but otherwise appears well; she is afebrile. Physical examination detects fine rales over the lungs, as well as a red and thickened tympanic membrane. A chest X-ray shows bilateral patchy infiltrates. Laboratory studies indicate eosinophilia and elevated serum immunoglobulin concentrations. Her mother received limited prenatal care. Question What is the likely causative organism?

Correct answer: Chlamydia trachomatis Explanation The correct response is Chlamydia trachomatis. Chlamydia trachomatis is the most common sexually transmitted disease in the U.S. Infants born to mothers with active infection are prone to develop infections or pneumonia. The clinical picture above is highly suggestive of Chlamydia trachomatis pneumonia (i.e., staccato cough, afebrile, and eosinophilia). Staining a smear of a nasopharyngeal specimen with fluorescein-conjugated monoclonal antibody can establish the diagnosis. The specimen for this commercially available test has to be evaluated within 30 minutes. The early onset of this form of pneumonia suggests infection through direct contact during the birth process. The treatment of choice is a macrolide antibiotic such as azithromycin. Tetracycline may also be used. H. nfluenzae and S. pneumoniae typically have a more acute onset. Fever is usually present. E. coli is a rare cause of pneumonia in infants. It would usually have a more abrupt onset with fever. M. tuberculosis is uncommon in children not from endemic areas. It would also not typically present as patchy infiltrates.

Case A 27-year-old man presents with burning pain during urination and a 5-day history of urethral discharge. He sees his family physician because of his symptoms. A Gram stain and culture on a sample of the urethral discharge is performed; the results are negative. Question Ico-delete Highlights What is the most common cause of his condition?

Correct answer: Chlamydia trachomatis Explanation The most common cause of nongonococcal urethritis is Chlamydia trachomatis. Urethritis is an inflammation of the urethra. It is classified as either gonococcal urethritis (caused by Neisseria gonorrhoeae) or nongonococcal urethritis (caused by something other than Neisseria gonorrhoeae). Gonococcal urethritis is ruled out in this patient by the negative Gram stain results. Chlamydia is an obligate intracellular organism. Chlamydia infections characteristically show the development of inclusion bodies. Chlamydia was once thought to be protozoa as well as viruses, but they are neither. They are bacteria in the order Chlamydiales. Chlamydia trachomatis causes approximately half of all cases of nongonococcal urethritis in men. The other conditions caused by Chlamydia trachomatis are trachoma, inclusion conjunctivitis, and lymphogranuloma venereum. Symptoms of NGU consist of discharge from the urethra, itching, or burning associated with micturition. Diagnosis is by tissue culture, and more sensitive serological assay can be performed. Treatment with antibiotics, such as azithromycin, doxycycline, and ofloxacin, is effective. Azithromycin 2g orally as a single dose is most effective. Treatment of the sexual partner is also essential. Chlamydia psittaci does not cause nongonococcal urethritis. Chlamydia psittaci causes a systemic illness, called psittacosis, parrot fever, or ornithosis. Psittacosis is acquired from birds. Trichomonas vaginalis can cause nongonococcal urethritis in men but not nearly as frequently as Chlamydia trachomatis does. Trichomonas vaginalis is a protozoon. It causes trichomoniasis vaginitis, sometimes called trichomoniasis. Mycoplasmas are the smallest organisms characterized by the lack of cell wall. They commonly inhabit the mucosa of the respiratory and the urogenital tracts, residing extracellularly. Mycoplasma hominis and M. genitalium also cause NGU but are far less common than chlamydial urethritis.

Question A biochemistry professor cuts short his trip to South East Asia after developing malaria. When he gets home, he reviews his prescription with his family physician and is curious to know the mechanism of action of the drug he is taking. He is told that the drug inhibits synthesis of nucleic acids within the organism as well as inhibits the parasites' ability to metabolize and utilize erythrocyte hemoglobin. What drug is he taking?

Correct answer: Chloroquine Explanation Refer to the table. Chloroquine is a 4-aminoquinolone drug that is mainly used for the treatment of malaria. It has also been used in the treatment of rheumatoid arthritis and SLE. The exact mechanism of action of chloroquine has not been determined. It may inhibit synthesis of nucleic acids within the organism, and it may also inhibit the parasites' ability to metabolize and utilize erythrocyte hemoglobin. Interference with phospholipid metabolism within the parasite has also been proposed. Sulfisoxazole inhibits the conversion of para-aminobenzoic acid to dihydropteroic acid, a mechanism that is synergistic with the inhibition of dihydrofolate reductase by chloroquine. Metronidazole is used primarily for anti-amebiasis, anti-giardiasis, or anti-trichomoniasis because it covalently binds to protozoan guanine, which prevents further replication of DNA. Suramin treats trypanosomiasis by reducing ATP synthesis. Melarsoprol is an arsenic-containing compound used in initial treatment of trypanosomiasis by inhibition of sulfhydryl-containing enzymes (such as pyruvate kinase).

Question A 3-month-old infant is brought in by her parents because they are concerned about her health. They report that the infant has grown lethargic and weak. When they try to feed her, she is unresponsive to the presence of the baby bottle or breast-feeding. Physical examination is remarkable for hypotonia of the extremities and a flaccid neck. She has a poor gag and suck reflex. Her body temperature is normal, with a slightly decreased respiration rate. Further questioning of the parents reveals that the infant had been previously induced to bottle feed by coating the nipple with honey. Blood is drawn for toxin studies, and they come back positive. The honey is also examined for the presence of toxin, and it is also found to be positive. What organism is causing the infant's disease?

Correct answer: Clostridium botulinum Explanation Clostridium botulinum is a Gram-positive, spore-forming bacillus that is anaerobic. The organism produces a potent paralytic neurotoxin that can be fatal. In infants, clostridium botulinum produces a disease called 'infant botulism'. The incubation period is 3 - 30 days. Signs and symptoms of infant botulism are lethargy, weakness, poor feeding, constipation, hypotonia, poor head control, and poor gag and suck reflex. The duration of the illness is variable. Foods that are implicated in the disease are honey, and home-canned vegetables and fruits. The stool, serum, and food should be tested for the presence of toxin. Testing is usually done at specialized reference laboratories. Since botulinum antitoxin is not recommended for infants, supportive care is the treatment of choice. Campylobacter jejuni is a Gram-negative bacilli that is thermophilic and microaerophilic. The organism is oxidase-positive. The organism is a major human enteric pathogen that is equal to or greater than Salmonella and Shigella in incidence. The incubation period is 2 - 5 days, with a duration of illness of 2 - 10 days. Symptoms of Campylobacter infection include diarrhea (often bloody), cramps, and vomiting. The organism is found in raw and undercooked poultry, unpasteurized milk, and contaminated water. Supportive care is the treatment of choice; however, in severe cases, antibiotics such as erythromycin and a quinolone may be indicated. Also, Guillain-Barré syndrome can be a sequela. Bacillus cereus is a Gram-positive, spore-forming bacillus that is aerobic. It is catalase-positive, beta-hemolytic on sheep blood agar, nitrate-positive, and VP-positive. It is a major cause of a rapidly occurring food poisoning that produces severe nausea and vomiting; occasionally, there will be diarrhea. The duration of symptoms is generally about 24 hours. The source of the bacteria is improperly refrigerated cooked and fried rice, and meats. Diagnosis is usually performed clinically; however, a laboratory diagnosis when requested should involve the analysis of stool and food specimens for culture and toxin identification. Vibrio parahaemolyticusis a Gram-negative rod that is a halophilic (salt-requiring) vibrio. The organism is curved rather than a straight rod. It is oxidase-positive and should be cultured for selective thiosulfate citrate bile salts sucrose (TCBS) agar on which the organism appears as a distinct, opaque, green colony.. The incubation period is 2 - 48 hours, with symptoms lasting 2 - 5 days. Symptoms include watery diarrhea, abdominal cramps, nausea, and vomiting. Sources of infection include undercooked or raw seafood. Vibrio parahaemolyticus, when suspected, should be alerted to the laboratory for special media culture considerations. Supportive care is the treatment of choice. Antibiotics such as tetracyclines, doxycycline, gentamicin, and cefotaxime should be given in severe cases. Listeria monocytogenes is a Gram-positive, non-spore-forming facultative anaerobic bacillus. It grows on blood agar well and colonies are beta-hemolytic. On Gram stain, they are frequently mistaken for diphtheroids. They exhibit tumbling motility at room temperature and are bile-esculin-positive. The gastrointestinal form of the disease has an incubation period of 9 - 48 hours. Symptoms are of variable length and include fever, muscle aches, nausea, and/or diarrhea. Sources of infection include unpasteurized or under-pasteurized milk, ready-to-eat deli meats, and hot dogs. Though gastrointestinal initially, the disease can evolve into an invasive stage in 2 - 6 weeks. Blood or cerebrospinal spinal fluid cultures are best for invasive disease. Because of asymptomatic fecal carriage, stool cultures are not usually helpful. Treatment of choice is supportive care and antibiotics. In cases of invasive disease, intravenous ampicillin, penicillin, or TMP-SMX is recommended.

Case A 10-year-old girl presents with chest pain and joint swelling. Her mother says that about 2 weeks ago, her daughter had a sore throat with fevers and pus around her tonsils. She was prescribed a 14-day course of penicillin on the initial visit, but her mother stopped the medications after 2 days because the fever subsided. She also experienced chest pain and recurrence of fever. Her vital signs: blood pressure 120/80 mmHg, heart rate 110 beats/min, temperature 102.3°F. On physical examination, normal S1/S2 and a II/VI holosystolic murmur are heard. Her EKG shows a sinus tachycardia with a prolonged PR interval. Question What would have been the best way to prevent this disease?

Correct answer: Compliance with prescribed antibiotics Explanation The clinical picture is suggestive of acute rheumatic fever. Penicillin is prescribed to eliminate the streptococcal infection. In this case, not finishing the prescribed course of antibiotics did not eliminate the streptococcal infection and allowed the strep infection to progress to rheumatic fever. Steroids decrease myocardial inflammation in carditis. Steroids also reduce joint symptoms when responses to salicylates have been inadequate. Salt water gargles do not remove the cause of the sore throat. Aspirin and acetaminophen are prescribed to help reduce fever and inflammation but do not eliminate strep infections.

Question An audiologist is asked to evaluate a mentally disabled 1-year-old who does not seem to be responding appropriately to sound. The child has known congenital heart disease. What disorder might this child have?

Correct answer: Congenital rubella Explanation The incidence of congenital rubella decreased markedly in the United States since widespread immunization began. The two most common features are deafness and congenital heart disease; other features are intellectual disability, cataracts and other eye disorders, purpura, cerebral palsy, and bone abnormalities. Congenital syphilis can occur from the transmission of Treponema pallidum from mother to fetus any time during gestation. The fetus can be treated either in utero by giving the mother medication or after birth. Left untreated, notched central incisors called Hutchinson's teeth develop, along with intellectual disability, paresis, and interstitial keratitis. Chronic lead poisoning in children causes anemia, abdominal pain, intellectual disability, and various intellectual deficits. The treatment consists of removal and chelation. Fetal alcohol syndrome is associated with both binge drinking and consistently elevated levels of alcohol intake during gestation. The classical features include low birth weight, microcephaly, intellectual disability, cleft palate, cardiac defects, poorly formed concha, and joint deformities. Cretinism is severe fetal hypothyroidism resulting primarily from maternal iodine deficiency, but postnatal iodine deficiency can also be responsible. Features include stunted growth, intellectual disability, coarse hair, dry skin, a large tongue, and umbilical hernias.

Question A 3-year-old boy is admitted to the emergency room in acute respiratory distress. The patient has a body temperature of 40° C, a respiratory rate of 70/min, and a pulse of 130/min. Auscultations of the lungs are unremarkable. An examination of the throat reveals an exudate in the posterior pharynx that is yellowish and membranous. Bleeding occurrs when it is scraped and removed. The parents of the child reveal that the child has no prior immunizations. A throat culture was ordered and worked up specifically for an organism that selectively grows on cystine tellurite agar. What organism is causing this child's disease?

Correct answer: Corynebacterium diphtheriae Explanation Corynebacterium diphtheriae is an aerobic, club-shaped, Gram-positive rod that causes the disease diphtheria. Selective media (cystine tellurite agar) is used for the isolation and identification of the bacteria; the organism produces black colonies. The organism produces a toxin (diphtheria toxin) that is the major virulence factor; it enters the circulation and inhibits protein synthesis in a variety of tissues (the heart, nerves, and kidneys are particularly targeted). The disease will usually occur in individuals that have not been properly immunized (especially children). On physical examination, the patient will have a pseudomembrane formed at the back of the throat. This pseudomembrane is composed of bacteria, fibrin, dead epithelial cells, and red and white blood cells. Aspiration of this pseudomembrane can cause death by suffocation. In unvaccinated children, the mortality rate is approximately 20%. Treatment is usually with both antibiotics and diphtheria antitoxin. Bordetella pertussis, a fastidious Gram-negative bacillus, is the causative agent of whooping cough. The organism is difficult to recover in cultures; therefore, PCR testing and DFA are the tests of choice because results are acquired within hours instead of days. The organism specifically binds to ciliated epithelial cells. Since the nasopharynx is lined with ciliated epithelial cells, specimens obtained from this site are more reliable at obtaining valid results then any other specimen source. The infection is limited to the upper airways and pneumonia is a rare occurrence; therefore, chest radiographs are usually normal. Children with whooping cough have paroxysms of coughing. When they gasp for breath, the sound of this inspiration is the 'whoop' of whooping cough. Abnormal oxygen exchange is common and can cause the child to turn red, and sometimes blue. Repetitive coughing can lead to vomiting or choking on respiratory secretions. Respiratory syncytial virus (RSV) is the major cause of lower respiratory tract illness (i.e., pneumonia, bronchiolitis, and tracheobronchitis) in young children. The virus belongs to the Paramyxoviridae family and to the genus Pneumovirus. The virus is very contagious, and illnesses tend to peak in the winter or spring. Wheezing, rhonchi, rales, interstitial infiltration, and hyperinflation may be present on chest radiographs. Fevers are usually seen and range between 38° - 40° C. Hypoxemia may be profound in children. There is a need for the measurement of an infant's arterial oxygen saturation because the illness is clinically difficult to assess. Because of its contagious nature, infants in hospitals are placed in isolation to prevent outbreaks. The diagnosis of RSV can be made with reasonable accuracy based on the clinical and epidemiologic findings. Confirmation can be made by viral isolation (unfortunately this is time consuming) or by rapid diagnostic tests that are available and provide results in about 10 minutes. Ribavirin, an antiviral agent, is available for treatment of RSV in infants in whom the diagnosis is made during the early stages of the disease. It has been shown to decrease viral shedding and increase oxygenation. Haemophilus influenzae is a small Gram-negative coccobacillus that requires 2 supplements for growth (factor X and factor V). It is non-motile, grows best at 5 - 10% carbon dioxide, and grows on chocolate agar (because of the availability of X and V factors), but not on BAP or MacConkey agar. Pneumonia due to H. influenzae Type b typically occurs in patients between 4 months and 4 years old, patients with primary lung disease, and patients that are alcoholics. Radiologic findings are those of a segmented, lobar, bronchopneumonic, or interstitial pattern (listed in descending order of frequency). Cavitation is rare. Pleural effusion occurs in about 1/2 the cases, and the fluid is usually found to be sterile when cultured. Streptococcus pyogenes is a Gram-positive coccus, catalase-negative, and beta-hemolytic on blood agar; it appears as chains on Gram stain. Definitive identification to distinguish it from other beta-hemolytic streptococci is the detection of its specific 'A' antigen by latex agglutination techniques. Pneumonia with this organism is extremely rare; it is associated in children with streptococcal pharyngitis, scarlet fever, and streptococcal pyoderma. Bacteremia is uncommon. It is universally sensitive to penicillin.

Case A 15-year-old girl presents with a 2-day history of excessive vaginal discharge. She had unprotected sex with a boyfriend 4 days ago, and he later informed her that he has gonorrhea. On speculum exam, the girl is found to have a moderate amount of off-white and frothy vaginal secretions. The appearance of the cervix is normal. A cervical swab reveals copious Gram-negative intracellular diplococci. Question What should be the first step in the management of this patient?

Correct answer: Counsel the patient about safer sex and treat Explanation The patient should be provided counseling on the role of safer sex practices, condom use, the risks of sexually transmitted infections, and HIV transmission. Counseling skills characterized by respect, compassion, and a nonjudgmental attitude towards all patients are essential in obtaining a thorough sexual history and for delivering effective prevention messages. Treatment options for uncomplicated gonococcal infections involving the cervix, urethra, pharynx, and rectum in adults and adolescents include the following: cefixime 400mg orally as a single dose, or ceftriaxone 125mg IM as a single dose. Another treatment option is ciprofloxacin 500 mg orally as a single dose, plus a regimen to treat chlamydia (azithromycin 1g orally as a single dose). Although discussing medical treatment with an adult should be a consideration for minor patients, parental consent is not necessary to provide treatment of a sexually transmitted infection in a minor. Following safer sex counseling, the minor patient should be treated. Establishing other contacts and the reporting of gonorrhea are important public health aspects in the prevention of sexually transmitted infections and their sequelae.

Case A 35-year-old patient is admitted to the ER presenting with a severe headache. The headache is now 1 week old and has progressively worsened to the point where the patient finds it difficult to tolerate. He has been experiencing photophobia, malaise, and fevers that have gone up to 40° C. His present temperature is 38.3° C. The patient appears confused and disoriented, and his response to questioning is delayed. He mentions a 3-year history of IV drug abuse. His vital signs are all within normal limits. A chest examination and radiograph is normal; a CT scan is normal. The patient's mental status continues to deteriorate, and he is admitted. A lumbar puncture is done, and the results can be found in the table. An India ink prep prepared by the microbiology laboratory produces the following (refer to the image), and a specific antigen test is positive. The organism is recovered from the CSF 48 hours later. Question What is the most likely organism causing the patient's illness?

Correct answer: Cryptococcus neoformans Explanation Cryptococcus neoformans is an opportunistic yeast. It is the most common agent causing central nervous system infection in patients with HIV. Cryptococcus neoformans meningitis usually begins as an asymptomatic pulmonary infection that progresses to fungemia and meningitis in the immunocompromised individual. The organism's natural habitat is the soil, and it is found to grow well in pigeon droppings. Areas with an abundance of pigeons are particularly good sources of infection for these individuals. The India ink prep is a quick test that can be used to detect the presence of Cryptococcus neoformans in the CSF. The organism has a large capsule surrounding it that is impenetrable to the India ink; a clearing is seen around the organism when examined microscopically. However, the test has limited sensitivity. Antigen detection tests are more sensitive, but the turn-around time is considerably longer. Neisseria meningitidis is a Gram-negative diplococci that is kidney bean-shaped. It is a major cause of meningitis in young children and young adults. It is gamma hemolytic on blood agar. Infants have protective immunity from the mother that wanes as they approach childhood. Streptococcus pneumoniae is a Gram-positive lancet-shaped coccus that is catalase-negative and occurs in pairs. It is a common cause of meningitis, but is more common in elderly people and especially those that have underlying conditions, are malnourished, or are alcoholics. The organism is alpha-hemolytic on blood agar.

Case A 42-year-old man with a history of HIV presents with blurred vision. The patient also notes occasional small spots in his vision. The patient is noncompliant with his medications. Physical examination reveals vital signs of temperature 100.8°F, BP 114/80 mm Hg, RR 12/min, and pulse 76 BPM. Decreased visual acuity, as well as loss of peripheral vision, is demonstrated. The funduscopic examination is shown below. Blood work is notable for a CD4 count of 30 cells/mm3. Question What organism is most likely responsible for this patient's symptoms?

Correct answer: Cytomegalovirus Explanation The correct answer choice is cytomegalovirus. This HIV-positive patient, who is noncompliant with his medications, presents with blurry vision, loss of peripheral vision, as well as spots in his vision. This presentation is common in CMV retinitis. This disease occurs typically in the late stages, particularly when the CD4 count is less than 50 cells/mm3. In the figure, funduscopic examination reveals areas of infarction and hemorrhage. Classic cotton wool spots are also noted. Mycobacterium-avium intracellulare typically presents with a variety of symptoms such as persistent fever, night sweats, weight loss, chronic diarrhea, abdominal pain, and severe anemia. Toxoplasma is a common cause of ring enhancing masses on CT in patients with HIV. Epstein-Barr is more commonly associated with infectious mononucleosis, nasopharyngeal carcinoma, and Burkitt's lymphoma. HSV-1, or oral herpes, is the virus that causes cold sores. They are tingling or painful spots on the edge of the lip where it meets the skin of the face. HSV-2, or genital herpes, is the cause of very painful genital or anal ulcers. HSV infection of the eye, which causes a keratoconjunctivitis, manifests with an acute onset of pain, watery discharge, itching, blurred vision, lid swelling, and conjunctival injection. Uveitis may also occur with HSV infection; retinitis, though described, is rare.

Case A 30-year-old man receives a bone marrow transplant (BMT) for treatment of acute myeloid leukemia (AML). He develops increasing respiratory failure 3 weeks later. A bronchoalveolar lavage yields cells that are enlarged and have prominent intranuclear inclusions. Question What is the most likely diagnosis?

Correct answer: Cytomegalovirus infection Explanation Cytomegalovirus (CMV) produces a cytopathic effect, with enlarged cells having prominent intranuclear inclusions. In this patient, CMV has produced pneumonitis, which has led to the development of respiratory failure. The cytomegalovirus is a common pathogen in the immunosuppressed, in whom it causes morbidity and mortality. The treatment against CMV is ganciclovir, which is toxic and expensive; it is only indicated in immunosuppressed patients. Ganciclovir is given by intravenous infusion over 1 hour, 5mg/kg every 12 hours, for 14 - 21 days. Toxoplasmosis is not common in the lungs. Toxocysts contain bradyzoites, which are about 2 microns in size. They may be surrounded by tachyzoites of the same size as the bradyzoites. Candida albicans yields budding cells with pseudohyphae in the bronchoalveolar lavage. Pneumocystis carinii pneumonia (PCP) yields foamy material that appears acellular with H and E stains. Tuberculosis produces a granulomatous reaction. There may be giant cells of the Langhans variety, but a bronchoalveolar lavage is not very diagnostic without an AFB stain.

Case A 15-year-old girl has presented with a chief complaint of a rash. She does not take any over the counter or prescription medication and she does not have any pertinent medical history, nor does she have any drug allergies. She denies known exposure to any sick contacts in the last several days, but does admit to traveling to Tennessee and hiking in the Smoky Mountains the prior week. Further questioning reveals that the patient admits to feeling feverish (although the patient has not formally taken her fever), headache, lack of appetite, and muscle pain. Physical examination reveals a rash on her bilateral wrists, forearms, and bilateral ankles; it consists of numerous small, flat, pink macules, that are non-pruritic and non-scaly. Question Immediately knowing the diagnosis, what treatment regimen is most appropriate for this patient?

Correct answer: Doxycycline Explanation The patient case scenario results in a diagnosis of Rocky Mountain Spotted Fever (RMSF). This is a spirochetal disease that is spread via an infected tick. Despite its namesake, diagnosis of this has been highest in the following states: Arkansas,Delaware, Missouri, North Carolina, Oklahoma, and Tennessee, which is of course where our patient had traveled the week prior to her visit. Incidence of RMSF is highest during the summer months, specifically June and July. Symptoms of RMSF typically begin around 2-14 days after the bite of the tick; many times patients will not even realize or feel the bite. Various symptoms that a patient may present with include the following: fever, rash, headache, nausea, vomiting, abdominal pain, muscle pain, lack of appetite, or even conjunctival injection. Patients will typically not experience the full extent of the list and may only have a few of the symptoms. The classic rash of RMSF is a rash that first appears usually around 2-5 days after onset of a fever and will be small, flat, pink non-pruritic macules that appear on the wrists, forearms, and ankles and spread to include the trunk and potentially the palms and soles. A more red to purple, petechial rash is typically present after the 6th day of symptoms. Treatment should be initiated as soon as diagnosis is made. Doxycycline is the first line treatment for adults and children of all ages and should be initiated immediately whenever RMSF is suspected. Another option that may be considered is chloramphenicol.

Question A 38-year-old man who is known to be HIV-positive presents with a 2-week history of headache and a 1-day history of mild right-sided weakness. On exam, he is alert, mildly dysarthric, and has a mild to moderate right hemiparesis. A contrasted head CT shows 3 ring enhancing lesions in the left hemisphere. What is the proper initial management of this patient?

Correct answer: Empiric treatment with sulfadiazine and pyrimethamine Explanation Opportunistic processes, such as infections and neoplasia, are often associated with CNS involvement in AIDS. Toxoplasmosis is one of the most common opportunistic infections affecting the brains of AIDS patients. Frequently, it is the presenting manifestation in patients diagnosed with AIDS. Most commonly, toxoplasmosis presents as an acute or subacute process, causing an acute mental status change, hemiparesis, headache, and seizure. CT and MRI scans show ring-enhancing lesions with mass effect. Treatment consists of sulfadiazine and pyrimethamine. Because lymphoma can mimic CNS toxoplasmosis, a biopsy may be necessary if the mass does not respond to treatment. Cryptococcal meningitis is another common opportunistic infection seen in AIDS patients. This occurs as a meningoencephalitis due to Cryptococcus neoformans. Both toxoplasmosis and cryptococcal meningitis occur in about 5% of AIDS patients. This is usually a subacute process; it causes symptoms such as headache (most common), changes in mental status or consciousness, meningismus, fever, and occasionally seizures. Unlike toxoplasmosis, cerebral imaging is usually normal; however, hydrocephalus and basal ganglionic cryptococcomas can be seen. CSF analysis with lumbar puncture is diagnostic, with classical findings of low glucose, a paucity of inflammatory cells, and a positive India ink test (about 70% sensitive) or cryptococcal antigen (about 95% sensitive). Treatment consists of amphotericin B, which is often given with flucytosine. Since a significant number of patients relapse after therapy, many authorities advocate chronic oral fluconazole therapy. Roxithromycin therapy is used in treating C. pneumoniae, Legionella, and bacterial genital tract infections. This pharmaceutical, a semisynthetic derivative of erythromycin, has intracellular bioactivity, following endocytosis and concentration by phagocytes.

Case A 10-year-old boy presents with swelling on his face; it has been progressively increasing in size. He is an immigrant from East Africa. On examination, he has mild pallor and large swelling involving his right maxilla. A biopsy taken reveals a starry sky pattern of lymphocytes. Question What organism is associated with his condition?

Correct answer: Epstein Barr virus (EBV) Explanation The clinical presentation is suggestive of Burkitt's lymphoma.This is a highly malignant form of Non-Hodgkin's lymphoma that is associated with the Epstein-Barr virus (EBV). It is common in children in Central and East Africa. Patients may present with a rapidly enlarging jaw or abdominal mass. Histology reveals a "starry sky pattern" of rapidly proliferating malignant lymphocytes. Cerebrospinal fluid cytology should be included as part of the initial evaluation, as it has a high propensity to metastasize to the central nervous system. Treatment is by combination chemotherapy with meningeal prophylaxis. Human papilloma virus is associated with cervical cancer. Schistosoma haematobium is associated with squamous cell bladder carcinoma. Hepatitis B virus is associated with hepatocellular carcinomas. Human T-cell lymphotropic virus type 1 is associated with some T-cell leukemias commonly identified in Japan.

Case A 21-year-old woman developed a low-grade fever, sore throat, malaise, and fatigue that lasted several days. Physical examination revealed swollen lymph nodes and discomfort in the left upper quadrant of the abdomen. Examination of peripheral blood smear revealed 50% of atypical lymphocytes of the total white cells. Result for heterophile antibody test was positive. Question What is the most probable cause of infection?

Correct answer: Epstein-Barr virus Explanation Epstein-Barr virus and cytomegalovirus elicit a large T-cell response, resulting in a mononucleosis-like syndrome. EBV infection is subclinical and milder in children than adolescents or adults. EBV initiates infection in the epithelial cells of the oropharynx and then spreads to the B lymphocytes in lymphatic tissue and blood. Classical lymphocytosis is associated with the activation and proliferation of suppressor T cells, and it leads to infectious mononucleosis. The atypical lymphocyte, also known as "Downey cells", increases in peripheral blood during the second week of infection. Major T-cell response causes swollen lymph glands, spleen, and liver. Fever, malaise, fatigue, pharyngitis, lymphadenopathy, and hepatosplenomegaly are the classical symptoms associated with infectious mononucleosis. Cytomegalovirus is associated with heterophile-negative mononucleosis. Polyclonal B cell activation by EBV leads to the production of heterophile antibodies, which can be detected by the end of the first week of infection and last for several months. Respiratory syncytial virus, varicella-zoster virus, and influenza virus are not associated with infectious mononucleosis.

Case A 25-year-old Caucasian male landscaper presents with a 2-week history of generalized malaise and an "unusual rash" on his right thigh. The patient reports that this rash has been widening, but he denies any pruritus or pain in association with his complaints. In the past week, he has also noticed a constant headache and mild fever. The past medical history is unremarkable. The physical exam reveals vital signs within the normal limits and enlarged non-tender diffuse lymph nodes in cervical and inguinal areas, as well as an erythematous rash with central clearing and few satellite lesions. Question If your diagnostic impression of Lyme disease were to be confirmed, how would this patient's disease stage be defined?

Correct answer: First stage Explanation The correct response is first stage. This specific patient is at risk for contracting Lyme disease since he works outdoors and is at higher risk for insect bites (e.g., a tick bite). The rash, characteristically referred to as erythema migrans, is highly suggestive of Lyme disease. This disease occurs more commonly during the summer months and is caused by a spirochete called Borrelia burgdorferi. The clinical presentation of Lyme disease has been divided into 3 phases. The first phase is characterized by malaise and myalgias, as well as an enlarging erythematous rash at the site of the tick bite carrying the spirochete. The second phase involves the myocardium, causing conduction abnormalities and arrhythmias; it can involve the nervous system either centrally or peripherally. The third and final stage consists of mild neuropsychiatric disturbances and/or multiple-joint arthritic involvement. The terms early-onset and late-onset are not applied in the staging of Lyme disease. Tetracyclines are considered the treatment of choice, and they are currently the first-line therapy for Lyme disease.

Case A 36-year-old woman presents with vaginal discharge. She has a history of itching and white discharge. She is sexually active with her husband only. She takes oral contraceptive pills for contraception, and she has never been diagnosed with an STD (sexually transmitted disease). On examination, the vagina is hyperemic and covered with white cottage cheese-appearing discharge. Question What is the treatment of choice?

Correct answer: Fluconazole Explanation Candida vaginitis is the colonization of vagina and vulva by the fungus Candida albicans. It is favored by the acidic media, and it usually flourishes in patients on antibiotics or in those who are immunocompromised. It also flourishes due to increased wetness of the area. Culture on Sabouraud agar gives specific white colonies and a characteristic shape under microscope. Treatment with miconazole is usually effective. Some physicians add cortisone cream to relieve itching, which is usually severe. The discharge is curdy white. Herpes simplex is caused by infection with herpes simplex virus. The patient presents with severe pain and vesicles. Treatment with acyclovir is effective. Bacterial vaginosis, formerly known as non-specific vaginitis, is caused by a mixture of microorganisms, of which Haemophilus and Gardnerella are the most common and well-known. It causes malodorous discharge and is treated with metronidazole, tinidazole, or clindamycin. Trichomonas is caused by Trichomonas vaginalis, a flagellated parasite that causes copious vaginal discharge. Treatment with metronidazole is recommended. Clobetasol may be used for treatment for vulvar lichen sclerosis, which manifests as marked inflammation, epithelial thinning, and distinctive dermal changes accompanied by symptoms of pruritus and pain.

Question What is the only oral regimen that is recommended for the treatment of vulvovaginal candidiasis?

Correct answer: Fluconazole 150 mg PO x 1 dose Explanation Fluconazole (Diflucan) is the only oral treatment for vulvovaginal candidiasis. Metronidazole (Flagyl) may be used to treat bacterial vaginosis (BV) and trichomoniasis. Clindamycin is an option for treating pregnant women with BV. Tinidazole may be used to treat trichomoniasis. Valacyclovir (Valtrex) may be used to treat HSV.

Case A 32-year-old man presents with bilateral blurred vision and a large number of "floaters." The patient is HIV-positive and has experienced failure of long-term highly active antiretroviral therapy (HAART). His CD4+ T cell count is 50 cells per mm2. Fundoscopy shows white exudates around the retina with peripheral hemorrhages. Question What treatment would be most appropriate for this patient?

Correct answer: Ganciclovir Explanation The correct response is ganciclovir. Cytomegalovirus (CMV), as well as varicella-zoster and herpes simplex viruses, infects the retina or choroid. CMV retinitis is usually associated with severe immunosuppression, and it is particularly associated with patients who have HIV/AIDS. This infection is seen less frequently in patients undergoing highly active antiretroviral therapy (HAART) and usually occurs when the CD4+ T cell count is 50 cells per mm2 or lower. Diagnosis is made on clinical grounds, although over half of patients with CMV retinitis can be expected to have positive results on blood culture, urine culture, or antigenemia assay. Both ganciclovir and foscarnet have been used to treat CMV retinitis, but these drugs are virostatic, and the benefit of therapy is to limit further progression of disease. Untreated, CMV retinitis is destructive and can cause blindness.

Question A 17-year-old boy presents for an annual physical exam prior to the start of college. For the last 3 weeks, he has been camping in Los Alamos. Although he and his friends brought bottled water on the trip, he says the rivers were so clean that they all ended up just drinking from them directly. On further questioning, the boy explains that while he has been in generally good health, over the past few weeks he has been suffering from loose bowel movements and excessive flatulence. In addition, he intermittently has mild nausea and chronic abdominal pain that is interfering with his appetite. He thinks he has lost a little weight. He reports no history of fever, vomiting, or other non-gastrointestinal complaints. What is the probable cause of the boy's infection?

Correct answer: Giardia lamblia Explanation Despite vast improvements in sanitation, food-borne illnesses remain a fairly common occurrence. Prompt laboratory investigation is required in order to establish the diagnosis. This usually only occurs when a relatively large number of individuals simultaneously experience a similar constellation of symptoms after having consumed a common food source. Therefore, many cases of food-borne illness often go undiagnosed. Several infectious sources are associated with food-borne illness. The clinical symptoms associated with them demonstrate significant overlap, yet several of the most common causes have distinguishing features. Giardia lambliais a protozoon that causes upper intestinal enteritis marked by fatigue, chronic diarrhea, steatorrhea, colicky abdominal pain, and bloating. Infection can occur with ingestion of a single cyst, and symptoms generally appear within 1 week of exposure. Fecal contamination of drinking water is a common source of infection, as is person-to-person infection via hand-to-mouth contact with the feces of an infected individual. This method has caused outbreaks to occur in institutional facilities such as day care centers. Diagnosis of Giardia lamblia can be made by visualization of trophozoites or cysts in appropriately prepared fecal specimens. Staphylococcus aureus produces illness indirectly via a specific enterotoxin. Patients present with rapid onset of symptoms, usually within 30-60 minutes of ingestion. Patients typically experience nausea and cramps followed by vomiting. Duration of symptoms is short, usually 24-48 hours. Since humans are the principal reservoir for Staphylococcus, uncooked foods that require extensive handling, such as salads, eggs, dressings and sandwich meat, are the most common sources. Staphylococcus aureus also causes dermatologic lesions, such as pustules and abscesses; when present on the skin of food handlers, they can be a source of contamination. In addition, Staphylococcus aureus is present in cows; therefore, milk and cheese products that are not adequately refrigerated can also cause illness. Bacillus cereus is another organism that causes illness via the production of enterotoxins. 2 enterotoxins are produced by these spore-forming bacteria, 1 that is heat stable and another that is inactivated by heat (heat labile). The heat stable species primarily causes vomiting, whereas the heat labile form results predominantly in diarrhea. Intoxication caused by Bacillus cereus is most commonly associated with foods - classically rice - that have been reheated after having been left at room temperature. Illness usually occurs within a few hours of ingestion and usually resolves within 24 hours. Clostridium perfringens is also a toxin-producing organism. Cramping and diarrhea are the most common symptoms of illness caused by C. perfringens. Nausea also occurs, but vomiting is rare. Inadequately heated meat products are the most common source. Onset of symptoms occurs within 12 hours of ingestion, and symptoms usually abate in a day. Although often asymptomatic, infection with the Entamoeba histolytica protozoan can produce a host of intestinal syndromes, the most severe of which is acute dysentery marked by fever, chills, and bloody or mucoid diarrhea. Less severe colitis, indistinguishable from other causes of inflammatory bowel disease, can also occur, making the diagnosis of amebiasis difficult. In addition to intestinal illness, Entamoeba histolytica can also cause liver abscesses. As with giardiasis, fecally contaminated food or water is the most common method of transmission.

Case A 3-year-old boy is evaluated for a 24-hour history of diarrhea. His mother reports that he had 5 episodes of foul-smelling, watery diarrhea associated with decreased appetite. A few other children at the same day care center have also presented with the same problem. On physical examination, the child is well hydrated and his abdomen is tender. Stool microscopy shows the presence of motile trophozoites representing the etiological agent. Question What is most likely causing the patient's symptoms?

Correct answer: Giardiasis Explanation Diarrhea can be a manifestation of all of the diseases listed. Among these diseases, giardiasis is the only infection that is diagnosed by microscopic detection of motile trophozoites of the etiological agent in stool. Other manifestations described are also commonly observed in giardiasis. Giardiasis is a parasitic infection caused by an intestinal flagellate, Giardia lamblia (Giardia intestinalis). G.lamblia is the most common intestinal protozoan pathogen of humans. Giardiasis, an infection occurring worldwide, is common in day care centers and institutionalized individuals; it often occurs as outbreaks. It is transmitted by ingestion of food and water contaminated with cysts of the organism. Although in some individuals the disease may be asymptomatic, most people present with acute watery diarrhea associated with abdominal discomfort, bloating, and foul-smelling feces. Giardia cysts can be detected in stool samples of persons with symptomatic and asymptomatic infections. Immunosuppressed persons are susceptible to massive infection and severe clinical manifestations. The most widely used drug for treatment of giardiasis is metronidazole. Tinidazole and nitazoxanide are other drugs of choice. Cryptosporidiosis is a highly infectious disease caused by Cryptosporidium hominis. Outbreaks of diarrhea due to cryptosporidiosis are commonly found in day care centers, transmitted by ingestion of food or water contaminated with oocysts of the parasite. Cryptosporidium has assumed great importance as a frequent cause of severe intractable diarrhea in AIDS patients. Diagnosis is established by demonstration of sporulated oocysts in the feces by modified acid-fast or other staining methods, such as fluorescent staining with auramine phenol. Nitazoxanide and paromomycin are drugs effective against cryptosporidiosis. Cyclosporiasis is an infection caused by Cyclospora cayetanensis. Infected patients manifest with diarrhea, usually between 2 and 11 days after consumption of food or water contaminated with oocyst-laden feces. The diagnosis is established by identification of oocysts in stool samples stained by modified safranin, acid-fast, or by autofluorescence with UV-light microscopy. Treatment is with trimethoprim-sulfamethoxazole Isosporiasis, caused by Isospora belli, is commonly found in outbreaks in day care centers and mental institutions. After an incubation period of 7 to 11 days, patients manifest with watery diarrhea that can persist for several months. Diagnosis rests upon detection of oocysts in freshly passed stools. Stool concentration techniques are usually necessary, as the presence of the parasite in the feces is scant. Treatment is with trimethoprim sulfamethoxazole. Microsporidiosis is caused by microsporidia, minute intracellular parasites that reproduce by spores. Microsporidia had been known as animal parasites, but are now increasingly recognized as a group of parasites causing opportunistic infections in humans, mostly in AIDS patients. They can cause a wide range of illness from diarrhea to involvement of the CNS, eyes, viscera, muscles, and disseminated disease. Diagnosis is established by visualization of spores of microsporidia in stools, body fluids, or tissues after appropriate staining or electron microscopy. Albendazole is effective against microsporidiosis.

Case An 8-year-old girl presents with a 3-day history of fever, generalized muscle weakness, bilateral knee pain, and chest pain. Question What illness, contracted about 1 month ago, would support the diagnosis of Acute Rheumatic Fever (ARF)?

Correct answer: Group A Streptococcus infection of the upper respiratory tract Explanation The correct answer is group A Streptococcus infection of the upper respiratory tract, as this type of infection is a prerequisite to the development of ARF. Patients typically experience pharyngitis about 2-4 weeks earlier than the onset of symptoms associated with ARF. In addition to the preceding infection, there are other criteria, called Jones criteria, required to diagnose a patient with ARF. The major Jones criteria include carditis, erythema marginatum, subcutaneous nodules, chorea, and arthritis. The minor Jones criteria include fever, polyarthralgias, reversible prolongation of the PR interval on EKG, history of rheumatic fever, rapid erythrocyte sedimentation rate, and a history of streptococcal infection. Either 2 of the major criteria or 1 major and 2 minor criteria are required for the diagnosis of ARF. Proteus mirabilis infection of the urinary tract is not the correct answer. This organism can be seen in the urine, particularly in patients with renal calculi, and can lead to irritative voiding complaints. However, this type of infection would not predispose a patient to ARF. Helicobacter pylori infection of the GI tract causes inflammation of the lining of the GI tract. It has been linked to the development of ulcers, gastritis, and even stomach cancer. Haemophilus influenza infection of the upper respiratory tract is not the correct answer, as this organism does not lead to ARF. H. influenza can be responsible for various diseases, including pneumonia, bacteremia, bacterial meningitis, and otitis media. E. coli infection of the urinary tract is not the correct answer, as this organism does not lead to ARF. E. coli is the most common cause of uncomplicated urinary tract infections.

Case A 72-year-old man presents to the office with a 1-week history of a tingling and burning sensation on the left side of his forehead. He also reports reddening and blisters in the same area; they have been present for the past 3 days. There is also intense pain, and he reports a mild prodrome of headache. On exam, vesicles are present about the temple region in a dermatomal distribution that does not cross midline. Question What is the most likely diagnosis?

Correct answer: Herpes zoster Explanation Reactivation of the varicella-zoster virus results in Herpes zoster (shingles). Symptoms of herpes zoster include the warning symptoms of unilateral eruption, which include a tingling or burning sensation that is limited to a specific part of the body. The common sites of cutaneous involvement include the thorax, followed by the neck, face, and lumbosacral area. The prodromal phase is followed by the appearance of characteristic skin lesions, beginning as a maculopapular rash that follows a dermatomal distribution. The maculopapular rash evolves into vesicles on an erythematous base. Vesicles appear as dense, deep, small blisters (vesicles) that ooze and crust. The cutaneous lesions heal in 7-10 days; they result in scarring and pigmentary changes. Lymph node swelling may occur. Herpes zoster is accompanied by excruciating pain and an intense burning sensation. Ocular complications (mucopurulent conjunctivitis, episcleritis, keratitis, and anterior uveitis) occur when there is involvement of the ophthalmic division of the trigeminal nerve. The three main objectives in the treatment of Herpes zoster are treatment of the acute viral infection, management of the acute pain associated with herpes zoster, and the prevention of postherpetic neuralgia. These objectives are fulfilled by the use of antiviral agents, oral corticosteroids, and adjunctive individualized pain management modalities. The symptoms of Herpes simplex virus 1 (HSV-1) can be very painful. Blisters typically form on the lips, but they may also erupt on the tongue. The blisters eventually rupture as painful open sores. Healing occurs without scarring, and blisters disappear in 3-14 days. Increased salivation and foul breath may be present. The infection may be accompanied by difficulty in swallowing, chills, muscle pain, or hearing loss. About 60% of HSV-1 infections recur within a year. Recurrences are usually much milder than primary infections, and they are known commonly as cold sores or fever blisters. The infection with Herpes simplex virus 2 (HSV-2) usually occurs in or around the genital area 2-8 days after exposure to the virus. Flu-like discomfort and fever, nerve pain, itching, lower abdominal pain, urinary difficulties, and yeast infections (in women) may precede or accompany the eruption of the characteristic skin blisters. A new crop often occurs during the second week, and it is accompanied by swollen lymph glands in the groin. The symptoms may last as long as 6 weeks. In many cases, women whose lesions occur inside the vagina may be unaware that they have HSV-2. Lesions inside the vagina can cause a discharge, but they are not visible and cause minimal nerve pain. The outbreak of infection is often preceded by an early group of symptoms, known as a prodrome, which may include itchy skin, pain, or an abnormal tingling sensation. Impetigo is a skin disorder characterized by crusting skin lesions that is caused by bacterial infection. It may follow a recent upper respiratory infection, such as a cold or other viral infection. It is similar to cellulitis, but more superficial, involving the superficial skin. It is caused by streptococcus, staphylococcus, or both. Typically, this lesion begins as a cluster of tiny blisters that burst, followed by oozing and the formation of a thick honey-colored or brown-colored crust that is firmly stuck to the skin. Local lymph nodes near the infection may be swollen. The usual cause of folliculitis is bacteria (usually Staphylococcus). It can also be caused by a fungus. It may occur anywhere on the skin, and it is usually the result of injury or damage to a hair follicle. The injury can be caused by friction from clothing, blockage of the follicle, or by shaving. Symptoms include a rash, pimples, or pustules located around a hair follicle that may crust over, itching, and erythema of the skin.

Case A woman comes to your office after a 6-month sabbatical during which she worked in caves in the eastern part of South America. The patient presents with fever, chills, productive cough, and joint stiffness that started 1 month before her return. Physical exam reveals 3 ulcerated lesions on her inner cheek. Question What is the most likely diagnosis?

Correct answer: Histoplasmosis Explanation Histoplasmosis is a chronic respiratory infection caused by inhaling the spores of the fungus Histoplasma capsulatum, which is found in bird and bat droppings common along river valleys. Most cases are mild or asymptomatic. Risk factors include travel or residence in central/eastern United States or South America, pre-existing COPD (chronic obstructive pulmonary disease), and environmental or occupational exposure to the droppings of chickens, bats, and blackbirds. Immunocompromised people are also at risk. Symptoms include fever, chills, cough (with mucus or pus), skin lesions, and joint stiffness. The associated skin lesion usually presents as a lesion on the mouth or inner cheek as a papule; it may ulcerate. Tests include chest X-ray, sputum culture, (invasive) open lung biopsy, bronchoscopy (with or without transtracheal biopsy), histoplasma complement fixation titer, and CBC. The recommended first-line outpatient treatment is oral itraconazole. Acute pulmonary eosinophilia is a self-limiting inflammation of the lungs associated with the infiltration of eosinophils in the lungs and blood. The etiology includes exposure to various drugs, parasitic infestation (especially ascariasis in children), nickel exposure, recent blood transfusion, or lymphangiogram. Symptoms may include general malaise, loss of appetite, fever (greater than 2 to 3 days), productive cough (mucoid sputum), chest pain, shortness of breath, wheezing, rapid respiratory rate, headache, and muscle pain. Tests include auscultation of the lungs (revealing fluid), bronchoscopy (showing hypersensitivity reaction), open lung biopsy (invasive way to show hypersensitivity reaction), CBC (increased white blood cell count, particularly eosinophils), sputum smear (KOH test), and chest X-ray. If a cause is found, therapy consists of removing the offending drug or treating the infection with antibiotic therapy. If no cause is found, steroid therapy is given. Pulmonary actinomycosis is an infection caused by Actinomyces israelii or actinomycete bacteria that causes disease of the chest, mouth, jaw, and pelvis. The bacterium is in the normal flora of the mouth and gastrointestinal tract of humans. Symptoms include lethargy, weight loss, fever, productive cough, draining sinuses, night sweats, shortness of breath, and chest pain. Poor dental hygiene and a dental abscess may predispose it to facial lesions. In the chest, it results in cavities in the lung and pleural effusion, which may spread through the chest wall and produce sinuses. Tests include a CBC showing anemia, chest X-ray, culture of specific tissue, and bronchoscopy with culture. The goal of treatment is to control infection. Long-term treatment with penicillin or an alternative is necessary to ensure a cure. Surgical drainage of the pleural effusion and surgical resection of lung lesions may be necessary to control the infection. Acute coccidioidomycosis is a disease caused by breathing in a fungus found in the soil in certain parts of the southwestern United States, Mexico, and Central and South America. Dark-skinned people and people with a weak immune system will have more serious infections. Infection is caused by breathing in the spores of a fungus (Coccidioides immitis) found in desert regions. About 60% of infections cause no symptoms and are only recognized by a positive coccidioidin skin test. Symptoms include cough, chest pain (mild pain to severe constriction), fever, chills, night sweats, headache, muscle aches and stiffness, joint stiffness, and rash on the lower legs (erythema nodosum). Tests include sputum smear (KOH test), sputum culture, serum coccidioides complement fixation titer, CBC with differential (shows elevated eosinophils), chest X-ray, and coccidioidin or Spherulin skin test. The acute disease is almost always benign and goes away without treatment. Bed rest and the treatment of flu-like symptoms until the fever disappears may be recommended. Amphotericin B is used for progressive disease. Pulmonary asbestosis is a respiratory disease caused by inhaling asbestos fibers; it results in pulmonary fibrosis. Asbestos-related disease includes pleural plaques (calcification), malignant (cancerous) tumors called mesotheliomas, and pleural effusion. Cigarette smoking increases the risk of developing the disease. Symptoms include shortness of breath on exertion, cough, tightness in the chest, chest pain, nail abnormalities, or clubbing of fingers. Tests include auscultation of the lungs (revealing crackling), chest X-ray, pulmonary function tests, and CT scan of the lungs. Although there is no cure, supportive treatment of symptoms includes respiratory treatments to remove secretions from the lungs by postural drainage, chest percussion, and vibration.

Question What vaccination should be used in post-exposure prophylaxis for measles?

Correct answer: Human Immune Globulin (IG) Explanation Human immune globulin (IG) is used to provide antibodies for short-term prevention of measles and can prevent or modify measles in a susceptible person if given within 6 days of exposure to measles. The MMR vaccine is an attenuated (weakened live virus) routinely administered as a two-dose measles vaccination for primary (pre-exposure) prevention of measles. The HBsAG vaccine is used for hepatitis B. The influenza vaccine is made from allantoic fluids of infected embryonated chicken eggs. The Serogroup C vaccine is used for meningococcal disease. The BCG vaccine is used for tuberculosis. Even though complications associated with the vaccines have been reported, they are still used for protecting against viral infections worldwide.

Case A 20-year-old primigravida with 12 weeks of gestation presents with a 2-day history of low-grade fever and swelling in her neck. She does not have any significant medical history and denies previous blood transfusion. On questioning, she states that she is not sexually promiscuous and that she is living with her husband and their pet cat. She is a homemaker and spends her free time gardening. She has received all immunizations, and they are up-to-date. She also states that she has abstained from sex since learning of her pregnancy 2 months ago. On examination, her vitals are temp 99°F, PR 88/min, BP 110/70 mm Hg, and RR 20/min. She also has painless, prominent cervical lymph nodes. Abdominal examination reveals a just-palpable uterus. Question The fetus is at increased susceptibility to what infection?

Correct answer: Infection by Toxoplasma gondii Explanation The patient's history (gardening and presence of cat) and the clinical presentation are indicative of toxoplasmosis in the mother. This specifies a higher fetal susceptibility to infection by T. gondii than infection by T. pallidum, cytomegalovirus, rubella, or herpes simplex virus. Toxoplasma gondii is a protozoan parasite that causes toxoplasmosis. Infection mainly occurs by the ingestion of food or water contaminated with oocysts shed by cats or by eating undercooked or raw meat containing tissue cysts. Primary infection (toxoplasmosis) is usually subclinical; however, in some, it manifests as cervical lymphadenopathy with low-grade fever or ocular disease. Infection that is acquired during pregnancy may cause severe fetal damage. Diagnosis of toxoplasmosis can be confirmed by direct detection of the parasite or by serological techniques. The most commonly used treatment modality, and probably the most effective, is the combination of pyrimethamine with sulfadiazine and folic acid. Congenital toxoplasmosis occurs by vertical transmission from a recently infected pregnant woman to her fetus. Severe fetal consequences can occur when the transmission is closer to conception. The clinical manifestations include the classic triad of chorioretinitis, hydrocephalus, and intracranial calcifications. Other manifestations include anemia, jaundice, thrombocytopenia, microcephaly, convulsions, spasticity and palsies, intellectual disability, and learning disabilities. Syphilis, caused by the spirochete T. pallidum, is a sexually transmitted disease. Rubella is a RNA virus, and congenital rubella syndrome can be prevented by measles-mumps-rubella MMR vaccination. Cytomegalovirus infection is usually asymptomatic. Symptomatic disease usually manifests as a mononucleosis syndrome in immunocompetent individuals. In herpes simplex virus (HSV) infection, HSV-1 is transmitted primarily by contact with infected saliva; HSV-2 is sexually transmitted. Congenital HSV infection results from genital HSV infection with HSV-2 type.

Case A 30-year-old Caucasian man presents with a 3-day history of fever with chills and severe weakness. There are no other complaints. The patient has had multiple sex partners in the past. He also gives a history of travel to South America and consumption of street food while working there 1 month before presentation. He admits to intravenous drug abuse and cocaine abuse (snorting) in his early 20s. He often ventures out into the woods and has been bitten by several insects in the recent past. Abdominal exam reveals mild hepatomegaly. You send for routine lab investigations, including CBC, comprehensive panel, and serology of HIV and Hepatitis B and C. Liver enzymes are elevated and anti-HCV comes back positive. Anti-HIV and HBV are negative. Question How did this patient most likely acquire the hepatitis C infection?

Correct answer: Intravenous drug abuse Explanation The most likely means of transmission of hepatitis C virus (HCV) is by exposure to infected blood, and IV drug abuse is the most common source. In the United States, intravenous drug abuse is the most common mode of transmission of hepatitis C. Other modes of transmission include receiving blood or blood products, organs, needlestick injuries in healthcare, and vertical transmission from an infected mother. Other less frequent modes include sexual transmission from an infected partner, sharing personal items such as razors (which may have blood on them), and from healthcare procedures such as dental procedures or injections. There is limited information on whether cocaine snorting carries an additional risk of hepatitis C transmission. Hepatitis A, not hepatitis C, is transmitted by contaminated food from regions endemic for hepatitis A, such as South America, Asia, and Africa. Tick bites are not known to transmit HCV.

Case A 33-year-old woman comes to your office after a 6-month sabbatical working in caves in the eastern part of the South America. Upon questioning, the patient reports fever, chills, productive cough, and joint stiffness that started 1 month before her return. Physical exam reveals 3 ulcerated lesions on her inner cheek. Question What is your treatment recommendation?

Correct answer: Itraconazole Explanation The correct response is itraconazole. Histoplasmosis is a chronic respiratory infection caused by inhaling the spores of the fungus Histoplasma capsulatum, found in bird and bat droppings common along river valleys. Most cases are mild or asymptomatic. Risk factors include travel or residence in central/eastern United States or South America, environmental or occupational exposure to droppings of chickens, bats, or blackbirds, pre-existing COPD (chronic obstructive pulmonary disease), and immunocompromised people. Symptoms include fever, chills, cough (with mucus or pus), skin lesions, and joint stiffness. The associated skin lesion usually presents as a lesion on the mouth or inner cheek as a papule; it may ulcerate. Tests include chest X-ray, sputum culture, (invasive) open lung biopsy, bronchoscopy (with or without transtracheal biopsy), histoplasma complement fixation titer, and CBC. 2015 updated guidelines recommend Itraconazole 200 mg 3 times daily for 3 days and then 200 mg once or twice daily for 6-12 weeks for patients who continue to have symptoms for 11 months (B-III). Acute pulmonary eosinophilia is a self-limiting inflammation of the lungs associated with infiltration of eosinophils in the lungs and blood. The etiology includes exposure to various drugs, parasitic infestation (especially ascariasis in children), nickel exposure, recent blood transfusion, or lymphangiogram. Symptoms may include malaise, loss of appetite, fever (greater than 2 to 3 days), productive cough (mucoid sputum), chest pain, shortness of breath, wheezing, rapid respiratory rate, headache, and muscle pain. Tests include auscultation of the lungs (revealing fluid), bronchoscopy (showing hypersensitivity reaction), open lung biopsy (invasive way to show hypersensitivity reaction), CBC (increased white blood cell count, particularly eosinophils), sputum smear (KOH test), and chest X-ray. If a cause is found, therapy consists of removing the offending drug or treating the infection with antibiotic therapy. If no cause is found, steroid therapy should be given. Pulmonary actinomycosis is an infection caused by Actinomyces israelii or actinomycete bacteria; the infection causes disease of the chest, mouth, jaw, and pelvis. The bacterium is in the normal flora of the mouth and gastrointestinal tract of humans. Symptoms include lethargy, weight loss, fever, productive cough, draining sinuses, night sweats, shortness of breath, and chest pain. Poor dental hygiene and a dental abscess may predispose it to facial lesions. In the chest, it results in cavities in the lung and pleural effusion, which may spread through the chest wall and produce sinuses. Tests include a CBC (showing anemia), chest X-ray, culture of specific tissue, and bronchoscopy with culture. The goal of treatment is to control infection. Long-term treatment with penicillin or an alternative is necessary to ensure a cure. Surgical drainage of the pleural effusion and surgical resection of lung lesions may be necessary to control the infection. Acute coccidioidomycosis is a disease caused by breathing in a fungus found in the soil in certain parts of the southwestern United States, Mexico, and Central and South America. Dark-skinned people and people with a weak immune system will have more serious infections. Infection is caused by breathing in the spores of a fungus (Coccidioides immitis) found in desert regions. About 60% of infections cause no symptoms and are only recognized by a positive coccidioidin skin test. Symptoms include cough, chest pain (mild pain to severe constriction), fever, chills, night sweats, headache, muscle aches and stiffness, joint stiffness, and rash on the lower legs (erythema nodosum). Tests include sputum smear (KOH test), sputum culture, serum coccidioides complement fixation titer, CBC with differential (shows elevated eosinophils), chest X-ray, and coccidioidin or Spherulin skin test. The acute disease is almost always benign and goes away without treatment. Bed rest and treatment of flu-like symptoms until the fever disappears may be recommended. Amphotericin B is used for progressive disease. Pulmonary asbestosis is a respiratory disease caused by inhaling asbestos fibers; this results in pulmonary fibrosis. Asbestos-related disease includes pleural plaques (calcification), malignant (cancerous) tumors called mesotheliomas, and pleural effusion. Cigarette smoking increases the risk of developing the disease. Symptoms include shortness of breath on exertion, cough, tightness in the chest, chest pain, nail abnormalities, or clubbing of fingers. Tests include auscultation of the lungs (revealing crackling), chest X-ray, pulmonary function tests, and CT scan of the lungs. Although there is no cure, supportive treatment of symptoms includes respiratory treatments to remove secretions from the lungs by postural drainage, chest percussion, and vibration.

Question What is the most accurate statement regarding histoplasmosis?

Correct answer: Itraconazole, amphotericin B, ketaconazole, and fluconazole are all reasonable choices for therapy Explanation For the treatment of histoplasmosis, itraconazole is the most useful drug in most patients; this is due to its effectiveness, convenience of oral dosing, and relatively favorable side-effect profile. Amphotericin B, ketoconazole, and fluconazole can be considered reasonable alternative therapy in most patients with histoplasmosis, although ketoconazole is inadequate as sole treatment in patients with acquired immunodeficiency syndrome (AIDS). The most common clinical manifestation of histoplasmosis is a self-limited, influenza-like respiratory illness; chronic, cavitary pulmonary infection, and disseminated disease are less frequent. The principal route of acquisition of histoplasmosis is by inhalation of spores that are usually aerosolized from contaminated soil. The causative fungus, Histoplasma capsulatum, exists as the mycelial form in soil and as the yeast form in the body. Histoplasmosis has become one of the most common opportunistic infections in immunocompromised patients, such as those with AIDS.

Case A 2-year-old boy is from a poor rural family. He is experiencing a childhood exanthematous disease that involves a maculopapular rash and a fever. It started 7 days ago. He is now suffering from corneal ulcers and pneumonia. The family practitioner suspects the diagnosis of measles. Question Refer to the image and case. Which of the followings are the all-prodromal signs of this disease?

Correct answer: Koplik's spots, coryza, fever, cough, and conjunctivitis Explanation This child has the features of measles. Not having the immunizations properly, living in a low socioeconomic area, having a maculopapular rash with corneal ulcers, and pneumonia as complications indicates this child has measles. The other pyrexial exanthematous diseases will not cause corneal ulcers as a complication. The measles (rubeola) virus is in the Paramyxovirus family, transmitted by respiratory droplets produced by coughing and sneezing both during the prodromal period and a few days after the rash appears. After an incubation period of 10-14 days, the prodromal phase is characterized by fever, conjunctivitis (causing photophobia), coryza, cough, and Koplik's spots (bright red lesions with a white central dot that are located on the buccal mucosa). All of the prodromal signs should be present and should be followed by a typical maculopapular rash to be able to diagnose measles clinically. Rash appears at the exanthematic stage and lasts approximately 5 days.

Case A 32-year-old woman returns from a 2-week camping trip along the Appalachian Trail in New York; she presents with a low-grade fever and a non-pruritic rash on her back and buttocks. Physical exam reveals 2 large 5 cm erythematous lesions, with central clearing on her low back and buttocks. There is no lymphadenopathy. CBC with differential and ESR are normal. Question What is the most likely diagnosis?

Correct answer: Lyme disease Explanation Given this history and clinical presentation the tic-born, Lyme disease should be suspected and treated empirically. Serological testing for antibody to the spirochete Borrelia burgdorferi may be inconclusive; serological confirmation should be done 4 - 6 weeks after exposure. Rhus dermatitis is a contact dermatitis to plants in the Rhus family (e.g., poison oak, poison ivy). The initial lesions are pruritic fluid-filled vesicles in a linear pattern. Pityriasis rubra pilaris is a chronic disorder, manifesting as red scaly plaques and keratotic follicular papules most commonly involving the palms and soles of the feet. Typical lesions of erythema nodosum are erythematous, tender nodules on the anterior shins associated with fever, chills and malaise. The characteristic target lesion of erythema multiforme develops abruptly on the extensor surfaces in a symmetric fashion also involving the palms and soles.

Case A 17-year-old boy was diagnosed as HIV-positive 4 months ago. He has had little access to medical care for most of his life and has been homeless for the past year. You discover that he has no reliable evidence of routine child vaccinations. His CD4+ T-cell count is 100/mm3. Question Which vaccine would be contraindicated in this patient?

Correct answer: MMR Explanation The correct answer is MMR. Deriving the answer involves understanding which vaccines contain live virus and understanding that live virus vaccines are contraindicated in HIV-positive patients whose CD4 count is less than 200. Vaccines that contain live virus: MMR, varicella, oral polio, yellow fever, BCG Vaccines that contain inactivated virus or viral particles: influenza, pneumococcus, DPT, hepatitis A, hepatitis B The table below categories common vaccines based on whether they contain live or killed virus.

Case A 2-year-old boy is from a poor rural family. He is experiencing a childhood exanthematous disease that involves a maculopapular rash and a fever. It started 7 days ago. He is now suffering from corneal ulcers and pneumonia. The family practitioner suspects the diagnosis of measles. Question What best describes the most common distribution of rash in these cases? Refer to the image.

Correct answer: Maculopapular rash that starts behind the ears and spreads to the face, down the trunk and extremities Explanation A typical measles rash is a maculopapular rash that starts behind the ears and around the forehead, spreads to the face, and then progresses cephalad, down the trunk and extremities. Pneumonia, mentioned in this case, is a common complication. Erythema infectiosum (fifth disease), also known as slapped cheek disease, begins with cheek erythema, and the maculopapular rash starts on the arms and moves down the trunk and legs. A pruritic rash appearing initially on the trunk and then spreading peripherally is consistent with the diagnosis of varicella (chickenpox). Varicella rash, a papulovesiculopustular rash, starts with papules and then changes rapidly to the vesicles and the pustules. The lesions are usually present in several stages at the same time. Diffuse erythema of mucous membranes, palms, and soles on a child with fever should make one think about Kawasaki's disease. This is an important differential diagnosis of measles; it is a non-infectious, life-threatening, systemic vasculitis. It presents with fever, conjunctivitis, erythema of palms, soles, and mucous membranes, polymorphous rash, strawberry tongue, as well as dry, cracked lips. The most serious complications are cardiac with coronary vasculitis and aneurysm formation. A diffuse erythematous rash with circumoral pallor surrounded by a flush face is a scarlet fever rash. This rash is more prominent on the abdomen and cutaneous folds (Pastia's lines), and it is characteristic with strawberry tongue (inflamed papilla protruding through a bright red coating).

Case A 35-year-old man has just returned to the United States from Thailand. He presents with a 3-day history of chills, fever, headache, myalgia, weakness, and loss of appetite. For the past 12 years he lived in Thailand where he had 3 episodes of malaria. He gives a history of exploratory laparotomy and splenectomy following an auto accident in his 20's. Question Based on the information provided, what is the most important diagnosis to consider?

Correct answer: Malaria Explanation While many diagnoses could give symptoms similar to those described, malaria must be 1st on one's list of clinical suspicions if for no other reason than the good potential for a cure (from adequate treatment). Malaria should be considered if a febrile patient is in, or has recently left, a malarious locality. Splenectomy increases the risk of severe malaria. Malaria is an infection caused by the protozoan Plasmodium injected by anopheline mosquitoes; this causes invasion of RBCs, hemolysis, and reticuloendothelial hyperplasia. It is common in the tropics. The incubation period for P. falciparum is usually 7 - 14 days, but may be as long as 1 year if the patient is semi-immune or has taken prophylaxis. Following a prodrome of headache, malaise, myalgia, and anorexia, the patient develops paroxysms that last 8 - 12 hours; there is a sudden coldness and a severe rigor for up to 1 hour. These symptoms are then followed by high temperature, flushing, vomiting, and drenching sweats. Classical tertian and sub-tertian periodicity (paroxysms separated by 48 and 36 hours) are relatively rare. Daily (quotidian), or irregular paroxysms are more common. Signs include anemia, jaundice, and hepatosplenomegaly without lymphadenopathy or rash. There are no relapses after a successful cure. Diagnosis is by repeated microscopy of thick and thin blood films. In partially treated patients, bone marrow smears should be examined if blood smears are negative. Schistosomiasis (bilharzias) is the most prevalent disease caused by flukes. S. mansoni is more prevalent in Africa, the Middle East, and Brazil. S. japonicum is more prevalent in South East Asia. S. haematobium is more prevalent in Africa, the Middle East, Spain, Portugal, Greece, and the Indian Ocean. The snail vectors release cercaria which can penetrate the skin (e.g., during paddling). Traveling via the lungs, the parasites finally reach the portal and then the mesenteric veins (S. mansoni and S. japonicum) or the bladder veins (S. haematobium). The eggs released from these sites cause granulomata and scarring. Clinical schistosomiasis is an immunological process on the part of the human host; it is known to be due to a type IV hypersensitivity reaction (at least to S. mansoni) to schistosomal eggs. Clinical features include abdominal pain and bowel upset, and later, hepatic fibrosis, granulomatous inflammation, and portal hypertension in the case of S. mansoni. S. japonicum affects the bowel and liver, and may migrate to the lungs and CNS. Urinary schistosomiasis (S. hematobium) starts with frequency, dysuria, hematuria, and incontinence, and may progress to hydronephrosis and renal failure. The diagnosis is based on finding eggs in the urine or feces. Treatment is with praziquantel. Influenza is the acute infection of the respiratory tract by RNA orthomyxovirus, which has 3 types: A, B, and C. Subtyping for type A is by hemagglutinin (H) and neuraminidase (N) characteristics. The disease caused may be so mild as to pass unnoticed in some people, while in others (i.e., the elderly and the debilitated) it can cause morbidity. It may cause an epidemic or a pandemic. The symptoms include abrupt fever, malaise, headache, myalgia, vomiting, and depression. Convalescence may be slow. Serology (rising titers over 2 - 3 weeks) is helpful in establishing a firm diagnosis. Culture is possible. American trypanosomiasis (Chagas disease) is caused by Trypanosoma cruzi and spread by reduviid bugs. The patient may present acutely with fever, lymphadenopathy, and hepatosplenomegaly; there may also be a long latent period (e.g., 20 years) followed by signs of multi-organ invasion and damage. It especially affects the heart and smooth muscle of the gut. African trypanosomiasis (sleeping sickness) is caused by Trypanosoma gambiense. It causes a slow-wasting illness with a long prepatent period (West African variety). The organism enters the skin following a bite from an infected tsetse fly; it spreads to nodes, blood, spleen, heart, and brain. Diagnosis is through demonstration of the organisms in the blood or lymph nodes. Cultures should be taken daily for 12 days. CSF should be examined. Smallpox, a severe, highly infectious viral disease, is now eradicated. It was spread mainly by droplet infection; it caused fever, headache, muscle aches, and a severe blistering rash that left deep, pitted scars. The mortality was sometimes as high as 20%.

Question Ico-delete Highlights A young couple will be traveling through South America next month. They have asked the advice of a travel medicine clinic as to what kind of malaria prophylaxis they should use. The woman is 1-month pregnant. What statement about malaria is true?

Correct answer: Malaria can be transmitted by blood transfusion Explanation While most people contract malaria from the bite of infected female Anopheles mosquitoes, it can also be transmitted via blood transfusion and congenitally from mother to fetus. Both chloroquine and hydroxychloroquine are considered generally safe for use during pregnancy. Although mefloquine is not recommended for use during pregnancy, it is generally considered acceptable for use during the 2nd and 3rd trimesters. In many areas, chloroquine resistance has been reported to Plasmodium falciparum. There are 4 species of organisms causing malaria: Plasmodium falciparum, P. vivax, P. ovale, and P. malariae. P. vivax and P. ovale can cause relapses for up to 4 years after the discontinuation of malaria prophylaxis.

Question Subacute sclerosing panencephalitis is a late neurological sequelae of infection from what virus?

Correct answer: Measles virus Explanation Subacute sclerosing panencephalitis (SSPE) is a serious, late, and rare neurological complication occurring 1-10 years after recovery from measles. This fatal disease is characterized by a very slow replication and spread of measles virus in the brain. Patients with SSPE do not show a high number of measles viruses in the brain, but they demonstrate unusually high levels of measles antibody in the blood and cerebrospinal fluid. Many years after recovering from measles, patients develop changes in personality, behavior, and memory; those symptoms are followed by myoclonic jerks, blindness, and spasticity. Orthoreovirus mostly causes asymptomatic or mild infections in human. Persistent infection with Hepatitis B can develop into chronic hepatitis, cirrhosis, and hepatocellular carcinoma. Papillomavirus is associated with skin warts, Condyloma acuminatum, genital malignancies, respiratory papillomatosis, and focal epithelial hyperplasia. Parvovirus B19 is the etiological agent of erythema infectiosum primarily seen in children. Parvovirus B19 is also associated with chronic anemia in immunodeficient patients. Infection with this virus during pregnancy can cause anemia and congestive cardiac failure in the fetus and can lead to miscarriage or stillbirth.

Case A 3-year-old girl tells her parents that she has itching in what seems to be the perianal area. The itching is something that wakes her at night, and this is when she has complained most to her parents. Her pediatrician performs an anal swab, and microscopic examination reveals ova. Question What is the most likely recommended treatment for this patient?

Correct answer: Mebendazole (Vermox) Explanation This patient has an intestinal infection with Enterobium vermicularis, commonly known as pinworms. This most commonly occurs in children, and the possible symptoms include perianal or perineal itching, dysuria, insomnia, restless sleep, and vulvovaginitis. Children complain about the symptoms, especially the itching, more during the night. The physical examination should include inspection of the perianal area, and may even include a digital rectal examination or an anal swab. Microscopic examination will reveal ova, but the female worm may also be visualized. Parents can test at home by placing clear tape over the child's perianal skin during the early morning. The treatment for pinworms should include either mebendazole (Vermox), albendazole, or pyrantel pamoate, which are all anti-worm medications. Cefdinir is a cephalosporin that can be used to treat various bacterial infections, specifically otitis media, tonsillitis, sinusitis, and various skin infections in children; however, it is not effective against worms. In the oral form, miconazole is an antifungal used as an oral buccal tablet for the treatment of oropharyngeal candidiasis in adults. Alternatively, it can also be found in creams used to treat candidiasis of the diaper area; however, it is not effective against worms. Doxycycline is a tetracycline antibiotic that is usually not used in pediatrics, but can treat various bacterial infections in adults. Severe acne, sexually transmitted diseases, and urinary tract infections (usually complicated) can be treated with doxycycline. It would not be effective in the treatment of worms. Metronidazole can be given orally or intraveneously to treat susceptible anaerobic infections, including those occurring in the gastrointestinal system, on the skin, or in the lower respiratory tract. It would not be effective against worms.

Case A 23-year-old Hispanic man presents with a 2-week history of upper abdominal pain, occasional right lower chest pain, and low grade fever with malaise and appetite loss. He has been in USA for 2 years, and he recently went to Mexico about 3 months ago to visit his family. He denies nausea or vomiting. He does remember having a short, self-limiting period of diarrhea right after returning from Mexico. He has no past medical history and takes no medications. He does not smoke or drink. On examination, his temperature is 101.2 F, pulse 90/minute, BP 120/76 mm Hg, and respiratory rate 16/minute. There is no icterus, cyanosis, or pallor. Chest exam reveals decreased breath sounds in the base of the right lung and normal heart sounds. Abdominal examination shows tenderness in the right upper quadrant and an enlarged liver about 2 inches below the subcostal margin. Bowel sounds are normal, there is no ascites or splenomegaly, and rectal exam is unremarkable. Labs show WBC of 18,000/uL, Hb 12g%, platelets 320,000/uL, ALT 76 U/L, AST 62 U/L, and AP 85 U/L. Gamma GT is normal. CXR reveals small right-sided pleural effusion, and EKG is normal. Stool culture and blood cultures are pending. Ultrasound of the liver shows a single abscess 3 inches by 2 inches in the right lobe of the liver. Question What regimen would be best for this patient?

Correct answer: Metronidazole Explanation This Hispanic patient has amebic liver abscess. Treatment is with metronidazole for 7-10 days. Because metronidazole is well-absorbed from the intestines, oral administration is adequate. If diagnosed early and treated promptly, amebic liver abscess has a very low mortality. In contrast, pyogenic liver abscess is associated with high spiking fevers, signs and symptoms of sepsis, jaundice, and a focus of infection in the peritoneal cavity or biliary tract. Ultrasound may show multiple abscesses rather than a single abscess due the mode of hematogenic transmission. The risk factors are his origin, recent travel to Mexico (which is endemic for amebiasis), and typical signs and symptoms. Preceding diarrhea may be seen. Amebiasis is caused by the protozoan Entamoeba histolytica. The parasite exists in 2 forms: a cyst stage, which is the infective form, and a trophozoite stage, which is the form that causes invasive disease. Most infections occur in developing countries due to poor socioeconomic conditions and sanitation levels. In developed countries, such as the United States, amebiasis is mainly seen in migrants from and travelers to endemic countries. Most infections are asymptomatic, but amebic dysentery, amebic liver abscess, and rarely other manifestations such as pulmonary, cardiac, or brain involvement can occur. Fever, abdominal pain, pleuritic chest pain, mild elevation of liver enzymes, and leukocytosis without eosinophilia are characteristic. Stool culture may be positive for E. histolytica in 75% of cases.

Case A 12-year-old girl presents because her joints are hurting. Her knees and ankles have been painful, warm, and swollen for several weeks now. Symptoms first started in her left knee, then her left ankle became involved, and then her right knee was affected. Her left knee is "almost back to normal now," but the other involved joints are becoming more painful. She is a middle school student and has been unable to go to school for the past 3 days because of the pain. She is taking acetaminophen arthritis formula, but the pain continues to get worse. Her mother did not want to give her non-steroidal anti-inflammatory medications because of her age. Review of systems (ROS) is positive for fever, chills, malaise, mild chest pain when lying down, moderate headaches, and weight loss. ROS is negative for sexually transmitted diseases, trauma to affected joints, tick exposure, and recent travel. Question What component of the physical exam would lead you to suspect acute rheumatic fever as the cause of her joint pain?

Correct answer: Mitral regurgitation Explanation Acute rheumatic fever occurs after a pharyngeal group A streptococcal (GAS) infection. It most commonly occurs in children between 3-15 years. The Jones criteria are used to diagnose the disease. If a patient has a history of GAS infection and 2 major criteria or GAS infection and 1 major and 2 minor criteria, the diagnosis of acute rheumatic fever can be made. Major criteria: Carditis and valvulitis Chorea Erythema marginatum Subcutaneous nodules Minor criteria: Arthralgia Fever Prolonged PR interval Elevated serum markers of acute phase reactants Other infectious diseases may also cause arthritis. Erythema infectiosum is associated with human parvovirus B19. Erythema migrans is associated with Lyme disease. Jaundice is associated with Hepatitis B. Fever is a non-specific finding in a patient with inflammatory arthritis.

Case A 24-year-old man with a past medical history of HIV positive status for 2 years presents due to an ongoing chronic cough that he has had for the past 8 months. He admits to a mild fever that comes and goes during that same time period. A 5-pound unintentional weight loss is also discovered since his last visit to your office, which was approximately 9 months ago. He states he has noted an increased amount of breathlessness with just simple activities, which was never bothersome before the cough began. The patient denies smoking, and his TB test is negative. Question What is the most likely organism causing this patient's signs and symptoms?

Correct answer: Mycobacterium avium complex Explanation The patient being described in the above scenario is more than likely suffering from a pulmonary infection secondary to Mycobacterium avium complex (MAC). This type of infection is almost indistinguishable from tuberculosis, but causes a chronic, slowly progressive pulmonary infection in both immunocompromised and immunocompetent patients. Symptoms, when they present, are less severe and more chronic than a Mycobacterium tuberculosis infection. Symptoms most commonly seen include the following (as well as how often they are found): cough (91%), sputum production (85%), weight loss (53%), breathlessness (51%), chest pain (34%), hemoptysis (32%), and fever or night sweats (17%). Mycobacterium lepraeis the organism that leads to Hansen disease, otherwise known as leprosy. This disease state is rarely seen in the United States but is endemic in tropical and subtropical Asia, Africa, and Central/South America. Signs and symptoms are pale macular/nodular and erythematous skin lesions, as well as superficial nerve thickening, associated anesthesia, and motor abnormalities. Bilateral ulnar neuropathy is highly suggestive of this diagnosis. This is inconsistent with our patient. Mycobacterium haemophilum, Mycobacterium bovis, and Mycobacterium chelonae are all potential causes of lymphadenitis, also known as scrofula. These organisms are much more prevalent in Northern Europe and signs are not consistent with the patient scenario above.

Case A 28-year-old HIV-positive man presents with night sweats and chronic cough with very little sputum. His sputum for acid-fast rods is positive. His PPD skin test is negative. His CD4 count is 120/mm3. His sputum specimen is sent for culture, and his treatment is started. Question What organism is likely to be isolated from this patient's sputum?

Correct answer: Mycobacterium avium-intracellulare complex Explanation Mycobacterium avium-intracellulare (MAI) complex is composed of 2 species, M avium and M intracellulare; they are very difficult to distinguish from each other by standard laboratory tests. They cause opportunistic infections in immunosuppressed patients, such as patients with AIDS whose CD4 cell count is less than 200/mm3. MAI is the most common bacterial infection in AIDS patients. The lungs are primarily affected, but the infection can spread to other organs as well. Mycobacterium kansasii can cause lung disease (similar to tuberculosis), skin infection, and subcutaneous lymph nodes. It is susceptible to antituberculosis drugs. Mycobacterium marinum causes swimming pool granuloma. These are granulomatous, ulcerating skin lesions that occur in skin abrasions from swimming pools or aquariums. The natural habitat of the organism is both fresh water and salt water. It is susceptible to tetracyclines, trimethoprim-sulfamethoxazole, and to the usual antituberculous drugs. Mycobacterium scrofulaceum causes a granulomatous cervical adenitis, usually in children. The organism enters through the oropharynx and infects the draining lymph nodes. Treatment is usually surgical removal of the lymph gland. Mycobacterium fortuitum-chelonae complex is a saprophyte complex found in soil and water. It rarely causes human disease.

Case A 20-year-old male college student presents with fever, chills, malaise, headache, photophobia and confusion with numerous petechiae on his extremities and trunk. On examination, he has positive Brudzinski and Kernig signs. A CT scan of the head is within normal limits. A lumbar puncture reveals increased leukocytes, particularly polymorphonuclear neutrophils, increased protein, and decreased glucose levels. Gram staining of the CSF reveals Gram-negative cocci in pairs. Question What is the most likely causative agent for the infection in this patient?

Correct answer: Neisseria meningitidis Explanation Meningitis can be caused by bacteria, viruses, and fungi. Neisseria meningitidis is the most likely causative agent based on the Gram staining finding and the age and clinical presentation of the patient. N. meningitidis are Gram-negative diplococci usually found as normal flora of the upper respiratory tract and cause symptoms such as skin lesions, fever, malaise, and headache. N. meningitidis is the most common cause of acute bacterial meningitis in patients 18-60 years of age, and 2 classic signs of acute bacterial meningitis are positive Kernig and Brudzinski signs. Kernig sign is the inability to fully extend the knee when the hip is flexed to 90 degrees due to severe hamstring stiffness. Brudzinski sign is flexion of the hips and knees with passive flexion of the neck due to neck stiffness. Treatment for bacterial meningitis often involves vancomycin and ceftriaxone because they have good central nervous system penetration. Bordetella pertussis is incorrect. Bordetella are Gram-negative rods causing whooping cough in humans. The organisms colonize in the upper respiratory tract and result in fever, malaise, and a cough characterized by an inspiratory gasp (whoop) producing its name. This infection is highly contagious and is spread by coughing and nasal drops. The patient in this case does not have a cough. Francisella tularensis is incorrect. Francisella is another Gram-negative rod that is responsible for tularemia (rabbit fever). The reservoir is in rabbits, and it is transmitted by ticks. The bacteria multiply at the site of penetration in the skin and are then transported to regional lymph nodes. Patients suffer from a high fever, chills, headache, and other symptoms similar to the plague. Haemophilus influenzae is incorrect. Although H. influenzae can cause meningitidis in infants and children 6 months to 6 years, the incidence of meningitis due to H. influenzae has declined significantly since the mid-1980's as a result of the widespread use of vaccination against this organism. The age of this patient rules out H. influenzae as the causative agent in this case. Yersinia pestis is incorrect. Yersinia is a Gram-negative rod that causes Bubonic plague in humans. Yersinia is transmitted by fleas from rodents to humans. The patient in this case does not have Bubonic plague.

Case A 50-year-old man presents for the evaluation of a 1-year history of progressive cognitive, motor, and behavioral problems. He complains of inattention, reduced concentration, slowing of processing, and difficulty changing mental sets. What started as slow movements now is clumsiness and problems with coordination. His friend states that a patient is "not himself anymore" and has become apathetic, non-communicative, and "down." He is HIV positive and was diagnosed with AIDS 2 years ago because of the presence of Pneumocystis carinii with CD4 of 100. However, he had an excellent response to antiretroviral therapy, and his last CD4+ lymphocyte counts were normal and viral load undetectable. On examination, you find an apathetic male in mild distress. Neurological exam shows loss of coordination, unsteadiness, generalized weakness (more pronounced in legs), ataxia, and tremor. Question What should be the next diagnostic step in this patient?

Correct answer: Neuroimaging methods Explanation Progressive cognitive, motor, and behavioral problems in an HIV-infected patient points to the diagnosis of AIDS dementia complex. Brain imaging is essential in the evaluation of patients with AIDS dementia complex, not only for the confirmation of the diagnosis, but also to exclude opportunistic infections, including progressive multifocal leukoencephalopathy, toxoplasmosis, tuberculosis, and cryptococcosis, as well as tumors, primary CNS lymphoma and/or the ependymal changes consistent with cytomegalovirus encephalitis. Neuroimaging in AIDS dementia complex will show cerebral, often basal ganglia atrophy, and white-matter abnormalities (described as "fluffy," "ground-glass," or even diffuse). Formal neuropsychological tests are not diagnostically specific. In the particular clinical context, they might help you to determine whether symptoms and signs are consistent with AIDS dementia complex and determine the stage of the disease. Electroencephalography (EEG) is the recording of spontaneous electrical activity of the brain. It is not specific and is used primarily in the diagnosis of epilepsy. Sometimes it is used in the diagnosis of coma, encephalopathies, sleep disorders, and brain death. Cerebrospinal fluid examination is also non-specific; it might be even normal or you may find slightly elevated spinal fluid protein, IgG, oligoclonal bands, and lymphocytosis. CD4/CD8 ration reflects the ratio in blood. HIV may be isolated either directly from the CSF or in culture. CSF examination is better used in differential diagnosis rather than as a method to diagnose ADC, since CSF findings in these patients are not specific. Although AIDS dementia complex is usually observed in the late stages of acquired immunodeficiency syndrome, when CD4+ lymphocyte counts fall below 200 cells/mL, lymphocyte counts is not diagnostic for AIDS dementia complex.

Case A 43-year-old man visits the internal medicine clinic of a university hospital presenting with a 3-week history of shortness of breath, fever, and chills. Examination shows a temperature of 38.0° C, and laboratory results suggest hypoxia. Previous history shows the patient has been HIV-1 positive for 4 years and presently has a CD4+ T-cell count of 50/mm3. A presumptive diagnosis of Pneumocystis carinii pneumonia (PCP) is made, which is confirmed by bronchoalveolar lavage. Question What physical finding in PCP would be expected from chest auscultation in this patient?

Correct answer: No findings Explanation The correct response is no findings. Pneumocystis carinii pneumonia (PCP) is an important opportunistic infection in patients with HIV-1 infection and CD4+ T-cell counts <200/mm3. Despite frequent radiographic findings typical of pulmonary edema and bilateral interstitial infiltrates, auscultation is usually completely normal. Diagnosis of PCP is difficult, as manifestations are often subtle and sometimes exclude respiratory symptoms. Induced sputum results are variable, with sensitivity for P. carinii detection of 30 - 90%. Bronchial lavage is 98 - 100% sensitive for P. carinii detection in persons with AIDS, and lung biopsy is seldom necessary.

Case A 9-month-old female infant presents with an acute onset of a rash on her trunk. She has a 4-day history of fever up to 104°F, but the mother states her daughter has no fever today. She has had some diarrheal stools, but no vomiting. No coughing or nasal congestion has been noted. The child has previously been well. Her past medical history is unremarkable, and she is up to date on her immunizations. She attends daycare, and her mother notes that some children have been ill with non-specific febrile illnesses over the past 2 weeks. Her only medication has been ibuprofen for the fever. Physical exam shows a temperature of 98.8°F, pulse of 124 BPM, and respiratory rate of 28/min. She is alert and shows no other abnormalities. Her exam shows normal tympanic membranes in the ear and normal conjunctivae in the eye. The neck shows some shotty anterior cervical adenopathy; the throat shows slight erythema of the posterior pharynx, and the skin appears with an erythematous maculopapular rash most pronounced on the trunk. Her chest is clear to auscultation, heart rhythm is regular without murmurs, abdomen is soft and non-tender, and her neurological exam is normal. Question What is the most appropriate intervention for this patient?

Correct answer: Observe the child and reassure the parent. Explanation The clinical syndrome in this vignette is typical for roseola infantum, also known as exanthem subitum. Typically, the patient will have a moderate-to-high fever for 3-4 days, followed by defervescence and the outbreak of an erythematous macular or maculopapular rash that usually begins on the trunk, spreads to the arms and neck, and occasionally affects the face and legs. The rash is usually gone within 3 days. Occasional cervical lymphadenopathy is seen, as well as a slight pharyngeal inflammation and coryza. The patient can present with a bulging fontanelle and suffer from febrile convulsions during the high fever stage of the illness. Human Herpesvirus 6 is the etiologic agent of the vast majority of cases. Once the rash has appeared and the child appears well, the only treatment is reassurance for the parents in regard to the benign nature of the condition. Patients with the pre-rash presentation can cause a diagnostic challenge in that pneumococcal bacteremia may also present with a high fever and a well-appearing child. In that case, a CBC and blood culture may guide therapy towards perhaps presumptive treatment with antibiotics until the blood culture results were known. Oral acyclovir might be used for the treatment of varicella (chickenpox). These lesions appear as intensely pruritic erythematous macules and evolve to clear fluid-filled vesicles. Clouding and umbilication of the vesicles occur between 24 and 48 hours, and it is followed by the crusting over of the lesions and development of scabs. Different lesions will be found in different stages of development at the same time during the illness. The average number of lesions is approximately 300. Measles titers are appropriate in the setting of a typical measles exanthem and prodrome. The prodrome typically presents with the triad of cough, conjunctivitis, and coryza. Koplik spots may be seen in the buccal mucosa opposite the lower molars at this time. The rash appears as faint macules on the upper neck, behind the ears, and along the hairline. The rash quickly spreads to the face, upper arms, entire neck, upper chest and onto the back, abdomen, entire legs, abdomen, and finally to the feet. The temperature rise usually occurs with the onset of the rash, and it dissipates when the rash reaches the feet.

Case A 55-year-old African American woman presents to you with fatigue, non-productive cough, and hair loss, as well as admitting to a 40 pack-year smoking history. You take a thorough history and perform an appropriate physical examination, and you suspect Human Immunodeficiency Virus (HIV) infection. Question Which of her physical exam findings is most specific for HIV infection?

Correct answer: Oral hairy leukoplakia on tongue Explanation The correct answer is oral hairy leukoplakia on the tongue. Oral hairy leukoplakia (OHL) presents as unilateral or bilateral non-painful white lesions that can be seen on the margins, dorsal, or ventral surfaces of the tongue or on the buccal mucosa. The OHL lesions may range from appearing smooth and flat to raised, irregular, and feathery. OHL is associated with HIV infection and immunosuppression and rarely develops in patients who are immunocompetent. Rales, vitiligo, alopecia areata, and clubbing may be seen in patients with HIV infection, but none of these findings are considered specific for HIV infection.

Case A 6-year-old child presents with a 2-day history of fever, cough, coryza, and rash. The rash started behind the ear and spread downwards towards the trunk. On examination, you notice erythematous lesions, which are primarily located on the trunk. The palm and soles are spared. On oral examination you notice 1-2mm bluish lesions surrounded by an erythematous halo. Question Which virus is most likely responsible for this patient's condition?

Correct answer: Paramyxovirus Explanation The patient is suffering from rubeola/measles, which is caused by a paramyxovirus1. It is characterized by truncal rash, centrally distributed, which usually starts at the hairline and moves down the body, sparing the palms and soles. It begins as discrete erythematous lesions that become confluent as the rash spreads1. Koplik's spots (1-2mm white or blue lesions surrounded by an erythematous halo on the buccal mucosa) are pathognomonic for measles and are generally seen during the first 2 days of infection1. This patient has both the typical rash and Koplik's spots, which help make the diagnosis of measles. Togavirus causes rubella1. The rash of rubella spreads from the hairline downwards, but unlike that of measles, the rubella rash tends to clear from originally affected areas as it migrates. In addition, it may be pruritic1. Forchheimer spots (palatal petechiae) may develop and are nonspecific1. Human parvovirus B19 causes erythema infectiosum1. It primarily affects children 3-12 years old1. The rash associated with this illness develops after resolution of a fever and appears as bright, blanchable erythema on the cheeks (slapped cheeks) and perioral pallor1. A more diffuse rash (often pruritic) then appears on the trunk and rapidly develops into lacy reticular eruptions that wax and wane (especially with temperature changes) over several weeks1. Human herpes virus 6 causes roseola/exanthem subitum1. A diffuse maculopapular eruption develops over the trunk and neck, then resolves within 2 days. The rash follows resolution of fever1. It appears as 2-3mm macules and papules initially on the trunk and sometimes on the extremities (sparing the face), which fade within 2 days1. Varicella-zoster virus causes chicken pox characterized by macules evolving into papules and vesicles on an erythematous base. Lesions are pruritic and appear in crops1.

Case A 35-year-old man is admitted to the hospital with progressive shortness of breath, fever, and worsening cough. The patient had been in good health until 2 months ago, when he began losing weight. This was associated with anorexia, intermittent diarrhea, night sweats, and then a nonproductive cough. He had lost more than 20 pounds by the time he was admitted to the hospital. His past medical history is unremarkable. He has been divorced for 5 years, and he has 1 child. He is employed as a medical equipment salesman, traveling extensively in the midwest. He admits to drinking alcohol in large amounts on weekends, but he denies tobacco and intravenous drug use. He gives history of a previous homosexual encounter. Physical examination shows that the chest was normal to percussion and clear by auscultation, except for a few scattered rhonchi. The heart is normal except for tachycardia. The abdomen is soft with normal bowel sounds. Genitalia are normal; however, there is a painful 2 cm ulceration at the anal verge. The neurologic exam is unremarkable. Chest radiological findings show diffuse bilateral interstitial infiltrates. Arterial blood gases on room air show pO2 57 mm Hg, pCO2 31 mm Hg, and pH 7.45. His alveolar-arterial O2 gradient is 55 mm Hg. Bronchoalveolar lavage fluid with lung biopsy shows the presence of cysts. Sputum cytology is negative for acid-fast bacilli. Question What is the most likely diagnosis?

Correct answer: Pneumocystis pneumonia Explanation Whenever a young patient presents with fever, progressive exertional dyspnea, hypoxia, and loss of weight, the possibility of Pneumocystis jiroveci pneumonia complicating acquired immunodeficiency syndrome (AIDS) should be considered, especially when diffuse interstitial infiltration (or patchy shadows) are found on chest radiological study. The history of a homosexual encounter favors this diagnosis. This should be followed by tests to confirm HIV. The increased alveolar-arterial O2 gradient indicates severe respiratory dysfunction. Bronchoalveolar lavage with lung biopsy is an appropriate early step in his evaluation. Finding pneumocystic cysts in the alveolar lavage is a confirmatory diagnosis for Pneumocystis jiroveci pneumonia. Treatment is based on the alveolar-arterial O2 gradient, which is considered mild when the value is less than 35 mm Hg, moderate when it is 35-45 mm Hg, and severe disease when more than 45 mm Hg. The mainstay of treatment is given intravenously or orally. Combined therapy of trimethoprim-sulfamethoxazole and corticosteroids is necessary in the treatment of a severe case of pneumocystis in AIDS. Administration of corticosteroids helps to prevent respiratory failure and death in AIDS patients. When Pneumocystis jiroveci Pneumonia is found in the absence of underlying immunosuppression from malignancy or drug, the patient fulfills the definition of AIDS. The laboratory findings are not suggestive of tuberculosis, legionella pneumonia, lung cancer, or syphilis.

Case A 42-year-old man presents with a 3-week history of itching and dermatitis. This was followed by a bout of fever and chills, diarrhea, and hepatosplenomegaly. Blood investigations reveal nothing abnormal except eosinophilia. Stool examination reveals eggs of S. mansoni, confirming the suspicion that the person was suffering from schistosomiasis. Question What is the drug of choice for this patient?

Correct answer: Praziquantel Explanation Praziquantel is an anthelmintic used in the treatment of: Schistosomiasis (drug of choice) Clonorchiasis Paragonimiasis Taeniasis Neurocysticercosis Diphyllobothriasis The drug increases cell membrane permeability to calcium. This results in marked contraction, which is followed by paralysis of the worm musculature, leading to the death of the parasite. Praziquantel tablets are taken with liquid after a meal, and they should not be chewed because the bitter taste can induce retching and vomiting. Most frequent adverse effects include: Dizziness Headache Drowsiness Lassitude Mebendazole is a synthetic broad spectrum anthelmintic with activity against intestinal nematode infections, such as ascariasis, enterobiasis, ancylostomiasis, and trichuriasis. Albendazole is a broad-spectrum anthelmintic structurally related to mebendazole and with similar anthelmintic activity. It is the drug of choice for treatment of infections caused by many intestinal nematodes. It is an accepted drug for treatment of hydatid disease and neurocysticercosis. Diethylcarbamazine is the drug of choice for treatment of lymphatic filariasis and loiasis. Suramin is used for treatment of early African trypanosomiasis.

Case A 40-year-old male inmate, who was born in the United States, has been incarcerated for the past 20 years; he gets a required annual PPD. 2 days later, the nurse, who is a new graduate, reads his PPD at 11 mm; she verbally informs you of this result. However, all of his previous readings have been 0 mm, and he has not transferred to any other institution in the past 5 years. There are no known cases of active TB in your facility. Question What should be your next step?

Correct answer: Recheck his PPD Explanation You should recheck his PPD. A common mistake reading PPD tests is that erythema is considered a positive result. Induration plus erythema is considered a positive result. An induration must be palpated for a positive result. Improper training has caused an incorrect interpretation of results, which has caused patients to be unnecessarily treated. If the patient is a true converter when a PPD is confirmed positive, obtaining a chest X-ray, HIV screen, and hepatic panel/CBC should all be considered when starting LTBI treatment. Quantiferon-TB Gold testing is not recommended; it is used for patients from countries who routinely vaccinate for TB, as doing so can cause a false positive. It is also limited availability-wise, as it has a short holding time.

Case An 18-month-old infant presents with a 5-day history of fever of 104°F. On physical examination, you note a mildly lethargic and irritable infant. There are no other clinically significant findings. You prescribe acetaminophen (Children's Tylenol) and tell the mother to monitor the infant's fever for the next few days; if the fever goes down, everything should be fine. The mother calls the next day and says that the fever has stopped, but a rash has developed and she is concerned. The infant examination reveals a diffuse, fine, maculopapular rash. Presently, the child does not appear ill. Question What is the most likely diagnosis?

Correct answer: Roseola Explanation The clinical picture is suggestive of roseola. Roseola is a benign illness in humans caused by the human herpesvirus 6 or 7. It is the major cause of acute febrile illness in young children. The most prominent feature is a fever lasting up to 8 days, the fever often exceeding 39.5°C (103.1°F). After the fever subsides, a rose-pink maculopapular rash appears. Rubella appears as a maculopapular rash beginning on the face and spreading to the entire body; it disappears by the fourth day after prodrome of low-grade fever, ocular pain, sore throat, and myalgias. Rubeola consists of the prodrome of fever, cough, conjunctivitis, and coryza. The rash appears as maculopapular, spreading down from the face and hairline to the trunk over 3 days, then becoming confluent. Erythema infectiosum rash appears as raised, fiery-red maculopapular lesions on the cheeks that give a "slapped-cheek" appearance. Varicella (chickenpox) rash appears as widely scattered red macules and papules concentrated on the trunk and face, progressing to vesicles and pustules and followed by crusting of the lesions.

Case A 27-year-old man presents with the "flu". He says that he has felt feverish, tired, and mildly nauseated for the past few weeks. He mentions a headache and non-pruritic rash. He denies any past medical history or medication use. On exam, he is afebrile. Examination reveals diffuse mild lymphadenopathy with mild hepatosplenomegaly; his soft palate has a few scattered shallow ulcerations. The palmar and plantar surfaces have a scattered papular rash that is copper colored, with a few papules on the flexor surfaces of the arms, legs, and trunk. Laboratory evaluation reveals hemoglobin of 12.8 g/dL, hematocrit of 38%, and white blood cell count of 11.1 x 103/ìL. Question What is the most likely diagnosis?

Correct answer: Secondary syphilis Explanation Secondary syphilis presents with mucocutaneous lesions and constitutional symptoms that can mimic several different disorders, and thus clinicians need a very high index of suspicion to consider syphilis as a diagnosis. Syphilis is a sexually transmitted disease that is caused by infection with the spirochete Treponema pallidum. Acquired syphilis begins as T. Pallidum enters through the mucous membranes, generally after sexual contact, and the spirochete infects the regional lymph nodes. It then rapidly spreads throughout the body. The primary stage of syphilis is characterized by a primary lesion known as the chancre. It presents as a painless ulcer with regional lymphadenopathy and usually appears about 3 to 4 weeks after initial exposure.. The next stage, secondary syphilis, is characterized by the development of cutaneous rashes and mild constitutional symptoms. The cutaneous manifestations typically include a symmetric, papular, non-pruritic rash along the flexor, palmar, and plantar surfaces. The rash appears reddish-pink in Caucasian patients and more darkly pigmented in African American patients. Additionally secondary syphilis often presents with eroding lesions on the mucous membranes, in the mouth, vagina, penis, or rectum. They appear as grayish-white patches with a red center. The cutaneous rashes of secondary syphilis appear about 3 to 6 weeks after the end of the primary stage, and they are most pronounced after 3 to 4 months. This stage will resolve spontaneously, although in pigmented individuals some of areas of the rash may have residual hyperpigmentation or depigmentation. Secondary syphilis often has mild constitutional symptoms as seen in this patient. Mild anemia, hyperbilirubinemia, and albuminuria may be present as well. Tertiary or late stage syphilis is characterized by involvement of the nervous and cardiovascular systems. Tertiary syphilis may manifest years after the initial infection, and presentations include neurosyphilis (tabes dorsalis, meningitis, and dementia), thoracic aneurysm, osteitis, or the gummas of benign tertiary syphilis. Tertiary syphilis is uncommon. Secondary syphilis is treated with penicillin. All sexual contacts from the past year should be examined and treated, as well as educated as to their infectivity and that of their sexual partners. This patient has a presentation that is classic for secondary syphilis. Although no sexual history or serologic tests for syphilis are included in the history, a high index of suspicion for this disease will lead to the correct diagnosis. Rocky Mountain spotted fever can have a similar rash, although patients are usually febrile, with a more acute course, and you would not expect lesions in the mouth. Rocky Mountain spotted fever is caused by Rickettsia rickettsii, and is transmitted by ticks. Hand-foot-and-mouth disease is an acute infection caused by an enterovirus, and it typically affects only children. The oral lesions of hand-foot-and-mouth disease are vesicles that ulcerate, and they are very painful. Influenza and streptococcal infections are not likely to produce the cutaneous findings seen in this patient.

Question A 5-year-old boy presents with a 4-day history of bloody diarrhea. He has had fever up to 104 degrees F, abdominal pain, and painful defecation. His past medical history is unremarkable, and he has had no surgeries. He is on no medications and has no drug allergies. He attends a local daycare with 9 other children. On physical examination, his abdomen is tender with hyperactive bowel sounds. While in the emergency department, he has a 5-minute generalized seizure. What pathogen is the most likely cause of the patient's seizure?

Correct answer: Shigella sonnei Explanation The most likely cause of seizures in a patient with dysentery is shigellosis. This infection, caused by Shigella sp., is characterized by abdominal pain, fever, emesis, urgency, painful defecation, and diarrhea. The diarrhea may be watery and voluminous, and it may progress to frequent bloody stools. Neurologic findings such as convulsions, headache, lethargy, and nuchal rigidity are found in up to 40% of hospitalized infected children. Other major complications include dehydration, bacteremia, sepsis, and DIC. These occur more often in young, malnourished patients. Campylobacter organisms are important causes of bacterial gastroenteritis and systemic infections, especially in newborns and immunocompromised patients. These thin, Gram-negative rods cause acute gastroenteritis, bacteremia, and focal extraintestinal infections such as meningitis, pancreatitis, cholecystitis, urinary tract infections, peritonitis, and arthritis. Seizures are rare with this infection. Non-typhoidal Salmonella infections, caused by a motile, nonencapsulated, Gram-negative rod, may cause a dysenteric picture; however, they do not typically cause seizures. Acute gastroenteritis and bacteremia are common infections. Osteomyelitis is seen in patients with sickle-cell anemia. Suppurative arthritis may occur at sites of previous skeletal trauma. Meningitis is primarily found in neonates. Giardia lamblia is a protozoon that causes diarrhea and abdominal pain with cramps, but it is not associated with seizures. Rotavirus is the most important cause of dehydrating diarrhea in early childhood, but it usually produces a watery diarrhea and does not cause seizures, except in the case of severe electrolyte imbalance.

Case A 15-year-old boy presents with a 1-week history of malaise, a low-grade fever, and a sore throat. On exam, you note pharyngeal erythema, scant yellow exudates, and enlarged posterior cervical lymph nodes. There are no current signs of airway compromise. There is no skin eruption noted, and the rapid strep screen is negative. Question What other physical examination finding is usually present with this condition?

Correct answer: Splenomegaly Explanation The clinical picture is suggestive of Epstein-Barr virus infection/infectious mononucleosis. Roughly 50% of patients will have an enlarged spleen. Sclera icterus is not a physical examination finding of infectious mononucleosis. Epstein-Barr virus does not affect the pancreas. Hepatitis can be a complication of infectious mononucleosis, but an enlarged liver is not a physical examination finding. Cullen sign is a blue discoloration of the umbilical area and is associated with pancreatitis.

Case A 25-year-old sexually active man presents with a 2-week history of a painless sore on his penis. On questioning, he denies any discharge or pain with urination. On physical examination, the sore is still present. He is afebrile and is found to have palpable inguinal lymph nodes. The lymph nodes are enlarged, but painless. Dark-field examination results are positive. Question What is the most likely diagnosis?

Correct answer: Syphilis Explanation This man has signs and symptoms of primary syphilis, a painless chancre along with painless regional lymphadenopathy. Dark-field examination of a lesion will show the spirochetes in the majority of the cases. The ulcer seen with chancroid would be painful, and there would be painful tender nodes. Gonorrhea can present with symptoms of urethritis. A purulent discharge and dysuria are consistent with gonorrhea. Intracellular diplococci on a Gram stain of the urethral discharge is also consistent with the diagnosis of gonorrhea. The lesions caused by herpes genitalis are vesicular lesions on an erythematous foundation. Soft, painful adenopathy can be seen with herpes genitalis. Fever may also be seen. Condyloma acuminatum is the cause of genital warts. These are caused by human papilloma virus. Dark-field examination is not done on genital warts.

Case Your 25-year-old patient is 31 weeks pregnant. Her third-trimester chlamydia screening comes back positive. Question How do you proceed to counsel the patient?

Correct answer: Take azithromycin 1 gm PO x 1D, then perform a test-of-cure at the Group Beta strep screening. Explanation Azithromycin 1 g po x 1 is proven safe and effective in treating chlamydia in pregnancy. A test-of-cure (TOC) is recommended for pregnant women 3-4 weeks after treatment is completed. Group Beta strep cultures are obtained at 35-37 weeks gestation, which is around the same time a TOC should be obtained for this patient. Doxycycline is not recommended in pregnancy. Abstinence from sexual intercourse is recommended for 7 days after treatment is completed. Erythromycin base 50 mg/kg/day orally divided into 4 doses daily for 14 days is the treatment for ophthalmia neonatorum. The CDC recommends sex partners be evaluated, tested, and treated.

Question A 29-year-old man presents 1 hour after stepping on a sharp nail. The nail penetrated deep into his foot; his last tetanus immunization was at the age of 6. What treatment would be most appropriate?

Correct answer: Tdap vaccine plus antibiotics Explanation The correct response is a Tdap vaccine plus antibiotics. This patient has not received a tetanus booster for over 10 years. According to the immunization schedule, those patients who have received a primary course of 3 doses of DTaP (diphtheria, tetanus inactivated toxoids, and acellular pertussis) given from 2 months to 5 years should receive booster doses 10 and 20 years after the primary course. If more than 5 years have elapsed since the last dose, a booster dose of Tdap (tetanus, diphtheria toxoid, acellular pertussis) is indicated. This vaccine is recommended for children over 7 years old and adults; the lower case "d" and "ap" indicate that the vaccine contains smaller doses that the primary vaccine course. Tdap is preferred to Td (tetanus and diphtheria toxoid only) because of the resurgence of pertussis infections due to waning immunity. In children younger than 7 years, DTaP or DT, if pertussis vaccine is contraindication or allergy is present, is the appropriate therapy. In the case of a dirty wound, antibiotics are usually administered in addition to vaccination to prevent the risk of infection. Tetanus immunoglobulin is given if the individual has not received at least 2 previous tetanus toxoid shots, or if the tetanus-prone wound has not been treated for more than 24 hours. Tetanus toxoid only is adequate to prevent the development of tetanus from the dirty wound but does not address the fact that the patient also requires a diphtheria and acellular pertussis booster. In addition, in the context of an acute wound, antibiotics are usually administered as a precaution along with tetanus toxoid. Prescribing only antibiotics is not appropriate, as it fails to address the patient's need for boosters to prevent tetanus and diphtheria. Of note, some patients have contraindications to Td, such as a history of allergy or adverse vaccine reaction, unstable neurological condition, or acute illness. In these patients, passive immunization with tetanus immunoglobulin may be indicated.

Case A 50-year-old man presents with a 3-week history of diffuse pain, stiffness, and swelling of his right knee. He lives near a wooded area and was bitten by a tick 7 months earlier; he developed a local skin lesion at the site of the tick bite, followed by flu-like symptoms. He recovered without any specific treatment. Radiography of the knee joint shows evidence of effusion. Arthrocentesis yields inflammatory fluid. Question What test is recommended for confirming the diagnosis?

Correct answer: Testing for serum antibodies Explanation Testing for serum antibodies helps to confirm a clinical diagnosis of Lyme disease. Lyme arthritis is one of the common manifestations of Lyme disease, a multi-system disease caused by Borrelia burgdorferi and a few other Borrelia species of spirochetes. In the United States, Lyme disease is the most common vector-borne disease caused by B. burgdorferi. Lyme disease is named after Lyme, Connecticut, where the first cases were detected. The main vector is Ixodes scapularis, a deer tick. Human infections follow as a result of a bite from adult or nymphal ticks infected with the spirochete. The most common reservoir of B. burgdorferi is white-footed mice; nymphal Ixodes scapularis feed on these small mammals. The infected mice maintain a persistent asymptomatic spirochetemia for life. The incidence of Lyme disease is reported to be on the increase in several states in the United States. Early Lyme disease is characterized by a skin lesion following a tick bite; this is known as erythema migrans. Dissemination occurs days or months after the tick bite, leading to musculoskeletal, cutaneous, cardiac or neurological manifestations, meningitis, and joint involvement. Several months to years after the infecting tick-bite, approximately 60% of untreated patients develop intermittent bouts of joint pain and swelling; large joints are mostly affected. In the United States, Lyme arthritis is reported as the most common late manifestation of Lyme disease, and it may present without a history of exposure or any other concomitant symptoms. B. burgdorferi has a complex antigenic composition. Differential expression of its surface proteins in the mammalian host is thought to help the organism evade a host immune response and cause persistent infection. Serological testing for B. burgdorferi antibodies is indicated for a diagnosis in patients with a characteristic clinical picture of Lyme disease, except in those with erythema migrans; antibodies may not be at a detectable level. To improve the accuracy of serological testing, two-tier testing is recommended. Detection of IgG antibodies against B. burgdorferi by this method helps to confirm the diagnosis of Lyme arthritis. Serum is tested first by enzyme-linked immunosorbent assay (ELISA); if the result is positive or equivocal, it is tested by a more specific Western blot to corroborate the result of the first test. If the ELISA test is negative, no further testing is indicated. Whole-cell sonicates of B. burgdorferi are commonly used as the source of antigen in ELISA. The ELISA test for IgG antibodies is invariably positive in late disease (e.g. arthritis). Interpretation is done according to the criteria of the Centers of Disease Control and Prevention. A positive serology in the absence of clinical features cannot be considered a marker for diagnosis. False positivity due to cross-reactivity can occur in other conditions (e.g., relapsing fever, syphilis, and rheumatoid arthritis). In endemic areas, normal individuals may have serum antibodies. Serology done in the absence of a clinical suspicion of Lyme disease may lead to misdiagnosis. Lyme arthritis usually responds to antibiotic therapy. Oral doxycycline, tetracycline, or amoxicillin can be used for treating Lyme arthritis in adults. Amoxicillin, penicillin, or erythromycin can be used for children. Intravenous administration of cefotaxime, or ceftriaxone, is found effective in those who do not respond to oral therapy. In spite of the antibiotic therapy, some individuals develop chronic arthritis. This is observed in patients with certain HLA-DRB alleles. An autoimmune phenomenon involving antigenic mimicry between the outer surface protein OspA of B. burgdorferi and human lymphocyte-function-associated antigen-1 (LFA-1) is thought to have a role in the development of long-standing arthritis. No human vaccine is currently available for Lyme disease. An ecological approach to the control of Lyme disease, based on the immunization of animal reservoirs, has been reported. It is suggested that successful vaccination of the reservoir population intervenes in the natural cycle of Lyme disease and helps in reducing the transmission of infection to humans. The detection of B. burgdorferi antibodies in synovial fluid is of diagnostic significance. B. burgdorferi in fluid culture media can be observed as highly motile bacteria by dark-field and phase-contrast microscopy. Preparations stained by silver stains or fluorescent dyes can be visualized using light microscopy, but the utility of microscopic assay for the confirmation of Lyme arthritis is limited (due to the sparseness of the bacteria in clinical samples). Barbour-Stoenner-Kelly medium (BSK II), a complex liquid medium, is used for culturing B. burgdorferi, although rarely has the bacterium been isolated from synovial fluid. The bacterium is a slow-grower and requires prolonged incubation (up to 12 weeks) before the culture can be considered negative. Culture is mostly done for research studies, and not as a routine diagnostic test in clinical practice. B. burgdorferi DNA can be detected in the synovial fluid by a polymerase chain reaction (PCR); it has about 90% sensitivity and is more sensitive than a culture. Potential false positivity, due to contamination of the sample, is considered as a drawback for PCR assays. The test is not yet standardized and not recommended for routine diagnostic use. Blood culture samples are only rarely positive, even in early Lyme disease (erythema migrans) and in patients with systemic symptoms of dissemination. Recent studies have shown better recovery of B. burgdorferi by culturing enough plasma from untreated patients with erythema migrans. Blood culture is not indicated for diagnosing Lyme arthritis.

Case A 52-year-old man presents with a 4-hour history of a grossly soil-contaminated wound on his arm. He tried to clean it with some disinfectants, but it still looks dirty. Dead tissue and soil are visible on physical examination. The patient's immunization status is uncertain. Question What is the next step for tetanus prophylaxis following surgical debridement of the wound?

Correct answer: Tetanus immunoglobulin (TIG) and Tdap vaccine Explanation For the management of patients following dirty wounds whose last tetanus vaccination is unknown or ≥5 years ago, the preferred tetanus prophylaxis includes administration of tetanus toxoid-containing vaccine (such as Tdap) plus tetanus immunoglobulin (TIG). TIG should be administered within the first 24 hours post-injury. Tetanus does not produce natural immunity, so TIG is not given alone. Rather, Tdap (or Td - tetanus toxoid and diphtheria only) should be given once the patient recovers. TIG is not necessary if the wound is clean or if the patient has received at least 3 or more doses of tetanus toxoid in the past. DTaP is given to infants and children 7 and under. It is not given to adult patients, as it is intended to confer initial immunity upon an individual. It is contraindicated in infants younger than 6 months of age and in children older than 7 years. The capitalization of the letters D and P indicate a higher antigen quantity of diphtheria toxoid and acellular pertussis vaccine as compared with the Tdap.

Case A 32-year-old farmer presents with a deep wound on his right thigh. The wound was caused by a soil-contaminated sharp object. He remembers getting his most recent tetanus vaccine when he was 17 years old. Question What is the best approach in terms of treating this wound?

Correct answer: Tetanus vaccine, TIG (tetanus immune globulin), as well as cleaning and debriding the wound Explanation The correct response is Tetanus vaccine, TIG (tetanus immune globulin), as well as cleaning and debriding the wound. Any sort of wound, especially soil-contaminated wounds, should be assessed for tetanus prophylaxis. When trauma occurs, the wound should be cleaned well and necessary debridement should be done to remove the dead and necrotic tissues; this prevents anaerobic bacteria growth. However, disinfectants will not kill the spores; therefore, vaccination is necessary in order to inactivate the spores. For grossly contaminated deep wounds, vaccination is necessary if the last booster was given more than 5 years ago. Also, TIG should be given to inactivate the toxin. The administration of both immune globulins and tetanus toxoid (at different sites of body) is an example of passive-active immunity. For minor wounds, cleaning will be sufficient only if the tetanus vaccine was given less than 10 years ago. If the vaccination history is unknown or not completed, the vaccine should be given. Clostridium tetani is an anaerobic, Gram-positive, spore-forming rod; the main reservoirs are soil and stool. Deep wounds create an anaerobic environment and increase the chance of C. tetani growth. Spores germinate in the tissues, producing an exotoxin called tetanospasmin. This toxin is carried intra-axonally to the CNS, binds the ganglioside receptors, and blocks the release of inhibitory mediators (glycine and GABA) at spinal synapses. Excitatory neurons become unopposed and cause extreme muscle spasms. The tetanus vaccine (toxoid vaccine = formaldehyde-treated toxin) made a big breakthrough in controlling tetanus cases worldwide. Tetanus is still a problem in developing countries; however, in the United States, only 100 - 150 cases are seen per year. The vaccine is given at the age of 2 months, 4 months, 6 months, 18 months, and 5 years, in combination with diphtheria and pertussis (as DTP). A booster is given at the age of 14 - 16 years and every 10 years thereafter. Penicillin or metronidazole can be given prophylactically for grossly contaminated wounds.

Case A 25-year-old landscaper who lives in Rhode Island presents with a 2-week history of generalized malaise and an "unusual rash" on the right thigh. The patient reports that this rash has been widening but they deny any pruritus or pain in association with their other symptoms. In the past week, the patient has also noticed a constant headache and mild fever. Past medical history is unremarkable. The physical exam reveals vital signs within the normal limits and enlarged non-tender diffuse lymph nodes in cervical and inguinal areas, as well as an erythematous rash with central clearing. Question Based on the most likely diagnosis, what would you suggest as the initial line of therapy?

Correct answer: Tetracyclines Explanation The patient's rash is suggestive of Lyme disease. Tetracyclines are considered the treatment of choice for Lyme disease, and they are currently the first-line therapy. Alternatives include cefuroxime and amoxicillin. This specific patient is at risk for contracting Lyme disease since they work outdoors and are at higher risk for insect bites (e.g., a tick bite). The rash, characteristically referred to as erythema migrans, is highly suggestive of Lyme disease. This disease occurs more commonly during the summer months, and it is caused by a spirochete called Borrelia burgdorferi. The clinical presentation of Lyme disease has been divided into 3 phases: the first phase is characterized by malaise, myalgias, and an enlarging erythematous rash at the site of the tick bite (this carries the spirochete); the second phase involves the myocardium, causing conduction abnormalities and arrhythmias that can involve the nervous system either centrally or peripherally; the third and final stage consists of mild neuropsychiatric disturbances and/or multiple-joint arthritic involvement. Fluoroquinolones and aminoglycosides are antibiotics that are not used in the treatment of Lyme disease. Corticosteroids are used to treat contact rashes but are not appropriate for the treatment of Lyme disease.

Case An illness characterized by fever, lymphadenopathy, and headache occurred among patrons of a riding stable; 29 people were ill. Most cases occurred in mid-October. Most of the 29 had serologic evidence of acute toxoplasmosis; an additional 5 persons at the stable had serologic evidence thereof, but they remained asymptomatic. Of the 34 persons, 20 were identified as women, and 24 were between 16 and 30 years of age. 20 people at another stable were interviewed and tested for toxoplasmosis antibodies; they were all negative. Cats from the stable associated with the outbreak were bled, and serologic tests revealed that 2 of the 3 cats had elevated toxoplasmosis titers. Cats from the stable with no infections did not have measurable toxoplasmosis titers. Question Based on this information, what is the most logical deduction?

Correct answer: The patrons acquired the illness from the cats Explanation The correct response is the patrons acquired the illness from the cats. The protozoan Toxoplasma gondii infects humans via broken skin, the gut, and the lungs. The definitive host, the cat, excretes oocysts; however, eating poorly cooked infected meat is probably as prevalent as contact with cat feces. Toxoplasmosis is distributed worldwide, but it is common in the tropics. Infection is lifelong. Congenital toxoplasmosis may induce abortion. The fact that cats at the ranch where people became ill tested positive for elevated toxoplasmosis titers while cats from the ranch where no toxoplasmosis cases were reported supports the deduction that cats were the vector for this disease. The vast majority of infections are asymptomatic. Symptomatic-acquired toxoplasmosis resembles infectious mononucleosis; it is usually self-limiting and lasts a few months. The symptoms are not particularly characteristic; they are generally limited to fever, lymphadenopathy, and headache. Ocular infection presents with posterior uveitis, which causes blurring and ache; it often occurs in the 2nd decade of life. Encephalitis and myocarditis occur only rarely, except in an immunocompromised (e.g., AIDS) host, in which it may be fatal. Both sexes are susceptible to toxoplasmosis. Although 20 of the 34 infected persons were women, this is not evidence enough to deduce that women are more susceptible, as there is no information given about the number of each sex that were exposed to the illness. The diagnosis is through a 4-fold rise in antibody titer over 4 weeks, or the presence of specific IgM, which implies an acute infection. The 'dye test' was the 1st serological test available. Parasite isolation is difficult, but a lymph node or brain biopsy may be diagnostic. It is often self-limiting without treatment. If the eye is involved, or if the patient is immunocompromised, pyrimethamine should be used along with sulfadiazin

Case An infant presents for his 2-week physical exam. On exam, white plaques on the buccal mucosa and palate are noted. Upon trying to remove the plaques, there are small punctate areas of bleeding. Question What is the most likely diagnosis?

Correct answer: Thrush Explanation Thrush (oral candidiasis), which appears as white plaques anywhere in the mouth, is not uncommon in neonates who have had exposure to Candida albicans during birth. The punctate bleeding that occurs with attempts to remove the plaques is characteristic for candidiasis. Epstein pearls describe small, white, keratinized lesions found on either side of the mid-palatine raphe. They represent epithelial tissue trapped during fetal growth, and usually disappear shortly after birth. Bohn's nodules are small, cystic lesions located on the buccal and lingual aspects of the maxillary and mandibular ridges in neonates. They are remnants of mucous gland tissue, and they also disappear shortly after birth. Milk residue may resemble the lesions of thrush, but no bleeding occurs with removal of the milk. Aphthous ulcers (canker sores) are painful ulcerations that occur on the oral mucosa. They begin as erythematous, indurated papules that erode to form necrotic ulcers with an erythematous halo. Treatment consists of pain relief; the ulcers will heal spontaneously.

Case A 28-year-old man presents with severe headaches. He states that they began a few weeks ago and that he has been taking over-the-counter medications with some relief. He reports that he has AIDS. You ask what the patient was doing prior to experiencing these headaches. He says that he was taking care of the neighbor's cat while they were on vacation. Concerned, you order a CT scan; it demonstrates multiple ring-enhanced lesions. Question What organism do you suspect?

Correct answer: Toxoplasma gondii Explanation The clinical picture is suggestive of toxoplasmic encephalitis. The causative organism is Toxoplasma gondii. The definitive hosts are cats. In immunocompromised patients, imaging of the brain is warranted. The CT or MRI typically shows multiple ring-enhancing cerebral lesions. In immunocompromised patients, histoplasmosis is usually disseminated with primary lung and multiple organ system involvement. Also, the primary vector is bird droppings and bat exposure in the Ohio and Mississippi River valleys. Findings with cryptococcus include headache, abnormal mental status, and meningismus. Intracerebral mass lesions (cryptococcomas) are rarely seen. Neurologic CMV symptoms in the immunocompromised patient include polyradiculopathy, transverse myelitis, ventricular encephalitis, and focal encephalitis. There is no indication that CMV produces cerebral lesions. Herpes simplex encephalitis presents with nonspecific symptoms. A flu-like prodrome is followed by a headache, fever, behavioral and speech disturbances, and seizures. Cerebral lesions are not seen on CT.

Case A 42-year-old man presents to the local emergency department with a history of HIV infection/AIDS. He has been non-compliant with his HIV medication and has taken no medication at all for at least 4 months. His last CD4 count was 123 cells/mcL 6 months ago, and his HIV viral load was 926,000 copies/mcL. His girlfriend states that he has been stumbling and having memory problems and confusion for the past month. He also had what his girlfriend described as a tonic-clonic seizure 1 week prior but refused to come for treatment at that time. A CT of his head revealed multiple peripheral contrast-enhancing lesions. Question What is the most likely diagnosis?

Correct answer: Toxoplasmosis Explanation The correct answer is toxoplasmosis, as it is the most common space-occupying CNS lesion in patients with HIV/AIDS with CD4 cells counts under 100 while not on prophylaxis. Presenting symptoms are commonly those that are noted in the case above. In this case, since the patient was not taking medications for several months, it is likely that his CD4 cell count has dropped below 100. HIV dementia is a progressive disease that presents with patients having difficulty with cognitive and some motor tasks. Imaging shows an overall reduction in white brain matter, not contrast-enhancing lesions. PML is unlikely, as the lesions on CT in PML are not contrast-enhancing. CNS lymphoma is possible but is not the most common space-occupying CNS lesion, as noted above, and therefore is not the most likely diagnosis. Cryptococcal meningitis typically presents with fever and headache and does not cause contrast-enhancing lesions on head CT.

Question What is associated with syphilis?

Correct answer: Treponema pallidum Explanation Treponema pallidum is a spirochete that can be identified by means of dark-field microscopy of a sample of exudative material from a lesion or by immunofluorescent staining. Like other spirochetes, it appears as a long, thin, spiral-shaped organism that moves in a helical fashion. This spirochete enters the human body primarily by sexual contact between mucous membranes or abraded infected tissues. The primary infection results in a firm, nontender chancre approximately 2-4 weeks after exposure. If untreated, signs of secondary syphilis may arise, which may include a flu-like syndrome and a skin rash. Maculopapular rashes on the hands and soles must be considered highly suspicious for syphilis. Finally, tertiary syphilis may develop in a cardiovascular form, a neurological form (tabes dorsalis), or a gummatous form. Therapy varies some based on the stage and presumed length of exposure but the mainstay remains IM or IV penicillin, with tetracycline as an alternate for the penicillin-allergic patient.

Case A 14-year-old boy presents with a 1-week history of acute watery diarrhea with vague abdominal discomfort and vomiting. Now he has developed fever, malaise, facial and periorbital edema, and myalgias. He is experiencing pain and swelling of the calf muscles. The patient gives history of consuming some delicacies prepared from pork and game meat in a restaurant about 3 - 4 weeks back. Blood examination shows moderate eosinophilia. Question What is the most likely etiological agent responsible for the patient's illness?

Correct answer: Trichinella Spiralis Explanation The most likely diagnosis in the above child is Trichinella spiralis, as the clinical symptoms of acute diarrhea, abdominal discomfort, and vomiting appeared after consumption of pork and game meat. These symptoms were followed by fever, malaise, headache, facial and periorbital edema, and myalgias. Diagnosis is further supported by the presence of eosinophilia. Human trichinosis is caused by ingestion of inadequately cooked meat containing viable Trichinella larvae. Cyst is digested by acid pepsin. In the small intestine the larvae develop into adult worms, which are expelled in feces. The larvae enter the blood stream and seed striated muscle fibers. Over the next 3 weeks, the larvae increase in length and coil. They are now capable of infecting new hosts if ingested. Larvae eventually become ingested and can remain viable for several years. Most patients with moderate or mild infestation with Ascaris lumbricoides are asymptomatic. The most common symptoms of heavy infections include intestinal blockage and impaired growth in children. When larvae migrate through tissues, they can cause allergic symptoms like urticaria, cough, dyspnea, and wheezing. Worms may be passed in vomitus or stools.

Case A 23-year-old woman visits a community clinic presenting with vaginal pain, itching, and odor. The patient reports that she was treated for gonorrhea 3 weeks earlier. Examination shows a white vaginal discharge. Cervical cultures are submitted for Neisseria gonorrhoeae and Chlamydia trachomatis. Gram stain reveals predominant, slender, Gram-positive rods with rare neutrophils. Wet mount of a cervical smear shows flagellate protozoa. Question What is the most likely cause of the presenting problem?

Correct answer: Trichomonas vaginalis Explanation Trichomonas vaginalis is a flagellated protozoan causing a sexually-transmitted infection in about 25% of sexually active women. T. vaginalis produces a persistent vaginitis which causes vulvar itching, burning, and dysuria. Discharge is often copious and sometimes frothy. Recurrence of gonorrhea can result from treatment failure, or unprotected sexual contact with an infected partner subsequent to treatment. While gonorrhea can sometimes produce vaginitis and discharge, it is often asymptomatic in women. Neisseria gonorrhoeae can be detected as intracellular Gram-negative diplococci present in a purulent discharge from the cervix. Lactobacilli, which are slender Gram-positive rods, are present in abundance as normal flora in the healthy vagina. Gardnerella vaginalis, while structurally a Gram-positive organism, stains as a Gram-negative or Gram-variable short, pleomorphic rod. Although present in the normal flora of the vagina, G. vaginalis also is a cause of vulvovaginitis; this can be presumptively identified by the presence of 'clue cells'. Clue cells are epithelial cells upon which there is an abundance of short, pleomorphic Gram-negative rods (G. vaginalis), and a few long, Gram-positive rods (Lactobacilli). Herpes simplex virus is identified by the presence of localized burning which is associated with herpetic lesions.

Case A 30-year-old HIV-positive man presents to your clinic for the first time with a CD4 count of 185, and an HIV viral load of 485,000. The patient has no medication allergies. Question What medication should be initiated based on his current CD4 count?

Correct answer: Trimethoprim/sulfamethoxazole Explanation The correct answer is trimethoprim/sulfametholxazole. Primary prophylaxis for pneumocystosis in a patient with HIV infection should be initiated when CD4 cells counts drop to 200 cells/mcL or less. Clarithromycin and azithromycin are macrolide antibiotics, and are used for prophylaxis for mycobacterium avium complex in HIV+ patients whose CD4 counts drop below 50mcL. Amoxicillin is an antibiotic used primarily for upper respiratory infections with gram-positive organisms, and is not indicated as prophylaxis. Ciprofloxacin is a broad spectrum antibiotic used for a wide range of infections, but it is not indicated for prophylaxis in an HIV+ patient.

Case A 28-year-old woman presents with a 7-year history of AIDS. 1 month ago, her CD4 cell count was 48 cells/mm3 (stable over the past 3 months), and her viral load was detectable at 6,500 copies/mL. She feels well, and she is taking antiviral medication for her HIV infection. To date, she has not had any opportunistic infections. Question Based on her current CD4 cell count, what additional medication(s) should this patient be taking for prevention of opportunistic infections?

Correct answer: Trimethoprim/sulfamethoxazole and azithromycin Explanation The correct answer is trimethoprim/sulfamethoxazole (TMP/SMZ) and azithromycin. TMP/SMZ is used as prophylaxis for pneumocystosis in patients with CD4 cell counts <200 cells/mm3. It is also effective for prophylaxis of toxoplasmosis. It is indicated in patients with CD4 cells <100 cells/mm3 (as is the case in the above patient). In addition, for patients with CD4 cells <75 - 100 cells/mm3, azithromycin for prophylaxis against Mycobacterium avium complex is recommended. Oral gancyclovir is used in the treatment of CMV retinitis, which usually occurs in patients with CD4 counts <50 cells/mm3. It is not routinely given for prophylaxis against this infection. Pyrimethamine and leukovorin are used in conjunction with dapsone for prophylaxis of toxoplasmosis in patients who are not able to take TMP/SMZ.

Case A 25-year-old man develops clinical signs of bacteremia in the hospital. Examination reveals erythema and tenderness, and a slight purulent discharge around the insertion site of a central venous catheter. Gram stain of discharge shows gram-positive cocci in grape-like clusters. Culture sensitivity of the fluid showed methicillin-resistant Staphylococcus epidermidis. Question What is the most appropriate antibiotic therapy?

Correct answer: Vancomycin Explanation Staphylococcus epidermidis is a normal inhabitant of the skin, upper respiratory tract, and the gastrointestinal tract. It is an opportunistic pathogen; it requires a break in the skin or mucosal surfaces and the body defenses to establish infection. The skin penetration of these organisms is through artificial prostheses, IV lines, or through intraperitoneal catheters. It produces viscous slime that helps in the adherence to the plastic and foreign surfaces. Infections due to contamination of the surgical site by skin organisms or an exogenous source (e.g., vascular access devices) with bacteremia are characteristic. Vancomycin is the drug of choice in methicillin-resistant S. epidermidis, and it is administered as 1 g or 15 mg/kg IV every 12 hours. Adverse reactions associated with vancomycin include fever, rash, phlebitis, neutropenia, nephrotoxicity, auditory toxicity, interstitial nephritis, and several infusion-related reactions. Penicillin, cloxacillin, cefazolin, and sulfamethoxazole/trimethoprim are not drugs of choice in methicillin-resistant S. epidermidis.

Case A 36-year-old woman presents with a 24-hour history of sudden, severe diarrhea described as profuse, gray, cloudy, watery stools without blood or fecal odor. She was recently in Bangladesh for work and returned yesterday, which was when the diarrhea began. She is also experiencing a mildly elevated temperature with a very dry mouth, headache, and severe fatigue. Question What is most likely offending organism?

Correct answer: Vibrio cholerae Explanation The clinical scenario is most consistent with infection with Vibrio cholerae. Cholera is found in Asia, especially India and Africa, with epidemics seen where famine, overcrowding, and unsanitary conditions exist. The symptoms consist of sudden onset of profuse, gray, watery diarrhea that does not contain blood, pus, or fecal odor, and is commonly referred to as rice-water stools. Hydration is most important in this illness. Several antibiotics can be used to shorten the course of the illness, including fluoroquinolones, azithromycin, and tetracycline. Enterotoxigenic E. coli and Norwalk virus do not have the distinctive rice-water stool as noted above, but typically have cramping and fever associated with the illnesses. Shigellosis causes bloody diarrhea with abdominal cramping and fever. Clostridium difficile produces a toxin causing diarrhea and fever. It is usually associated with antimicrobial use. Oral metronidazole is first-line therapy, and vancomycin is used for resistant cases.

Case A 52-year-old man steps on a piece of glass, and he is seen in your office 1 day later. On exam, his wound appears clean, and it is not infected. He has never had the primary series of tetanus immunization. The patient asks if he needs tetanus immunization. Question What should you recommend?

Correct answer: A complete series of tetanus immunizations, plus tetanus immuno globulin (TIG) should be given Explanation If there is a doubt about the completion of the original series of three tetanus immunizations, then the person should receive tetanus immunoglobulin plus a complete series of tetanus immunization. The complete series is comprised of 3 doses; the initial 2 doses are administered 4 - 6 weeks apart, and the 3rd dose 6 - 12 months after the 2nd with a 1-time dose of Tdap substituted for 1 of the doses of Td. A booster is not enough in this case. A wound that looks clean should still be treated with a tetanus immunization as soon as possible. A person who has not had tetanus immunization in the past 5 years needs adult tetanus and diphtheria toxoid immunization.

Question What is the typical incubation period for Salmonella gastroenteritis?

Correct answer: 1-2 days Explanation Salmonella gastroenteritis has a typical incubation period of 1-2 days.

Question A 6-month-old infant is brought in for routine vaccination. Measles still continues to be a major health problem worldwide. Sporadic endemics continue in the United States in spite of efforts to eradicate the disease. The vaccine used for measles is a live-attenuated vaccine, and the age at which it is given is very important. What is the current vaccination guideline for measles?

Correct answer: 12 - 15 months and 4 - 6 years Explanation The MMR vaccine is a mixture of live attenuated viruses, administered for immunization against measles, mumps, and rubella. The current recommendation for MMR vaccine is that the 1st dose should be given at 12 - 15 months. The 2nd dose of MMR is routinely recommended at 4 - 6 years of age. However, it may be administered during any visit, provided that at least 4 weeks have elapsed since the first dose and both doses are administered beginning at or after the age of 12 months. Those children who have not received the second dose at the recommended age should complete the schedule by 11 - 12 years of age. The vaccine should not be given before 12 months of age because maternal antibody in the child can neutralize the virus, thereby reducing the immune responses. Since immunity can wane, a booster dose at the age of 4 - 6 years is recommended. Because it is a live vaccine, it should not be given to immunocompromised persons or pregnant women.

Case A 3-year-old child presents with history of severe bouts of coughing for the last 10 days. During coughing his face becomes red. Cough started with a common cold but has become very severe for the last 10 days. The episode of cough often ends with a "whooping sound" during breathing and is often followed by vomiting. Child also has low grade fever off and on for the last 10 days. Immunization records of the child are not available. Blood count shows leukocytosis with lymphocytosis. Question What is the antibiotic of choice for this child for treatment and prevention of spread of infection?

Correct answer: Azithromycin Explanation Clinical features and blood report are suggestive of the diagnosis of pertussis or "whooping cough" in the above child, which is caused by Bordetella pertussis, a gram negative bacterium. Antibiotics initiated during the paroxysmal stage do not affect the duration and severity of illness. They can hasten the eradication of Bordetella pertussis in the respiratory tract. Azithromycin is the correct answer. It is more resistant to gastric acid, achieves higher tissue concentration, and has a longer half life than erythromycin. It also has the advantage of allowing less frequent administration (1-2 dose/day) and shorter treatment regimen (5-7 days). Hence azithromycin is the antibiotic of choice for treatment and prophylaxis of pertussis. Erythromycin is not preferred for infants aged < 1 month because of a higher risk of IHPS. Since erythromycin requires frequent administration and longer treatment regimens, it is not the antibiotic of first choice for the above child. Cotrimoxazole (Trimethoprim-Sulfamethoxazole) is effective in eradicating B. pertussis from the nasopharynx, but it is contraindicated in infants aged < 2 months due to risk of kernicterus. It is used only as an alternative to macrolide antibiotic in patients aged > 2 months who have contraindication to or cannot tolerate macrolide antibiotics.

Case An 18-year-old woman presents with a 1-week history of vaginal discharge that has become progressively worse. She reports recent unprotected intercourse. On exam, the white discharge appears watery and she exhibits cervical motion tenderness. A KOH wet prep is negative without clue cells, hyphae, or organisms. Question What is the most appropriate treatment?

Correct answer: Azithromycin Explanation The clinical picture is suggestive of a chlamydia infection; the most common organism is Chlamydia trachomatis. While all of the answer choices are utilized to treat gynecologic infections, the recommended treatment for chlamydia is a single oral 1 g dose of azithromycin. Other treatment options include doxycycline or levofloxacin. None of the other options are appropriate treatment options for chlamydia. Ceftriaxone is the preferred treatment for gonorrheal infections. Fluconazole treats candidiasis or yeast infections. Metronidazole is the treatment for bacterial vaginosis and trichomoniasis. Penicillin G is utilized to treat syphilis.

Case While driving cross-country in a rural part of Southern Ohio, a traveling salesman's car breaks down. His mobile phone is not working. He finds a local farmer, and the farmer lets the salesman spend the night in his farmhouse. His car is repaired the next day, and the salesman promptly returns home. The next morning, he experiences an abrupt onset of a fever, nonproductive cough, shortness of breath, and mild chest pain. Believing he has just caught a cold, the salesman thinks nothing of it. 2 days later, he is much better, but he presents to your office for an opinion. You order a CXR that reveals airspace consolidations. Question What is the most likely cause of his symptoms?

Correct answer: Blastomycosis Explanation The clinical picture is suggestive of blastomycosis. Symptoms include nonproductive cough, moderate fever, dyspnea, and chest pain. Blastomycosis infections occur most often in men who engage in occupational or recreational activities outdoors in a geographically limited area of the south-central and midwestern US (Mississippi and Ohio River valleys). Spontaneous pneumothorax does not present with fever, which is present in this patient. Goodpasture syndrome is characterized clinically by a constellation of glomerulonephritis and pulmonary hemorrhage; injury is mediated by anti-GBM antibodies, which is not evident in this patient. Signs and symptoms of community-acquired pneumonia include fever or hypothermia, cough with or without sputum, dyspnea, chest discomfort, sweats, or rigors. Bronchial breath sounds or rales are frequently auscultated; they are not present in this patient. Signs and symptoms of a mycobacterium infection include fatigue, weight loss, fever, night sweats, and cough. Pulmonary infiltrates on chest X-ray, which are most often apical, are present; they are not seen in this patient.

Question A 2-month-old infant presents with a 48-hour history of poor feeding, crying, limpness, and constipation. The doctor in charge asks the mother how she feeds the infant. The infant is given formula regularly, but the mother sometimes adds honey to make the formula sweeter. She keeps the honey in the kitchen cupboard. When the doctor examines her, she notices a very drowsy infant with flaccid paralysis. What is causing this infant's symptoms?

Correct answer: Botulism Explanation Botulism is caused by Clostridium botulinum, which is an anaerobic, Gram-positive, spore-forming rod. The reservoir for Clostridium botulinum is soil and dust. The toxin is produced in poorly preserved food, most commonly in canned alkaline vegetables and smoked fish. It is absorbed by the gut and carried out by blood to the peripheral nerve synapse. It blocks the release of acetylcholine at the myoneural junction, resulting in a flaccid paralysis. When ingested by adults, the toxin causes weakness, dizziness, blurred vision, and flaccid paralysis in 1 - 2 days. Infant botulism is the most common form of botulism. Spores are ingested from household dust and (most commonly) honey. The toxin is produced in the gut, causing constipation, limpness, dysphagia, weak feeding, and crying. It may lead to respiratory arrest. Sepsis may cause weak feeding, limpness, and drowsiness, but it would not cause flaccid paralysis. Hypothyroidism is a slow-progressing condition and usually congenital in this age group; therefore, it would not suddenly present with symptoms in 2 days. Congenital muscular dystrophies are usually present from birth and may deteriorate gradually. Lactose intolerance presents with diarrhea and abdominal distention.

Case A mother has brought her 6-month-old infant to you due to a 1-day history of poor feeding, lethargy, and weak cry. In the beginning, the infant had difficulty sucking and swallowing and was not opening his eyes. This was followed by loss of head control, weakness of the arms, trunk, and then legs. The infant is constipated and has had drooling of saliva since yesterday. The infant was healthy before the onset of the above symptoms. There is no history of fever, vomiting, cough, seizures, or difficulty in breathing. The infant has weak gag and corneal reflexes, generalized hypotonia, loss of head control, ptosis, and diminished deep tendon reflexes. Blood counts and CSF examination are within normal limits. Question What is the most likely diagnosis?

Correct answer: Botulism Explanation The most likely diagnosis is infant botulism; the 6-month-old infant was previously healthy and developed poor feeding, constipation, lethargy, weak cry, drooling from the mouth, ptosis, and weak gag and corneal reflexes; those symptoms were followed by descending paralysis, starting with loss of head control, weakness of upper limbs, trunk, and then lower limbs. Blood counts and CSF were normal, which excludes sepsis. The classical triad of botulism is symmetric flaccid descending paralysis, beginning with cranial nerve musculature with clear sensorium, no fever, and no paraesthesias. Nerve conduction velocity and sensory nerve functions are normal in botulism. Human botulism has 3 naturally occurring forms: Infant botulism is the most common in USA. Food borne botulism occurs in older children and adults due to ingestion of food contaminated by botulinum toxin through improperly canned or preserved food, honey, or corn syrup. Wound botulism occurs due to colonization of traumatized tissue by Clostridium botulinum. Ingestion of honey is a risk factor for infant botulism. The clinical features of botulism are caused by a neurotoxin produced by Clostridium botulinum, which is a gram-positive, spore forming, obligate anaerobe naturally present worldwide in soil, dust, and untreated water. Botulinum toxin is carried by the blood stream to peripheral cholinergic synapses, where it binds irreversibly, blocking acetyl choline release and causing impaired neuromuscular transmission. Tick paralysis is a disorder of acetylcholine release and occurs due to a neurotoxin produced by the wood tick or dog tick. These arachnids are common in the Rocky Mountains of North America. The tick embeds its head in the skin, usually of the scalp, and starts producing the neurotoxin. It affects large myelinated motor and sensory nerve fibers. Motor symptoms include weakness of muscles and loss of coordination. Sometimes, it may produce ascending paralysis resembling Guillain-Barre syndrome. Tendon reflexes are lost. Parasthesias may occur over the face and extremities. Diagnosis is confirmed by identifying the tick. The tick must be removed completely along with the embedded head beneath the skin. The patient then recovers completely within a few hours or few days. Spinal muscular atrophy (SMA) is an autosomal recessive disorder and manifests as generalized symmetrical muscle weakness more in proximal muscle groups, hypotonia, and fasciculations of the tongue. Trunk muscles are commonly involved, and the infant is never able to sit. The infant has poor feeding and a weak cry. Weakness of the lower limbs is greater than the upper limbs. Deep tendon reflexes are lost. Fibrillations and fasciculations are reported on EMG. SMA type1 (Werdnig-Hoffman disease) presents during the first 6 months of life. Disease is progressive, and most patients die by 10 years of age. SMA types II, III, and IV occur later in life. Myasthenia gravis is an autoimmune disorder characterized by rapid fatigue of striated muscles. It can occur during the first few days after birth in infants of myasthenic mothers. Placental transfer of ACh R-antibodies results in transient impairment of neuromuscular transmission in the neonate. Symptoms include poor sucking and swallowing, weak cry, ptosis, decreased movements of the limbs, generalized weakness, and hypotonia. Symptoms usually resolve in 4 weeks or may persist for months. Juvenile myasthenia gravis is usually present after 10 years of age. Clinical features include ptosis, facial, and oropharyngeal weakness, resulting in dysarthria, dysphagia, and difficulty in chewing. Proximal muscles of the limbs are more involved. Weakness improves after sleep or resting and is worsened by exertion. It can occur at any age, but is most common in people of both sexes between 30-50 years of age. Guillain-Barre syndrome is an autoimmune process and is characterized by symmetric weakness of muscles starting from the lower limbs and then ascending to the trunk and upper limbs. Pain in the muscles may occur early in the course of illness. In 2/3 of cases, there is a preceding history of viral illness 2-4 weeks prior to the onset of neurological symptoms. Weakness is more marked in proximal muscle groups. There is hypotonia with diminished deep tendon reflexes. Facial nerve involvement occurs in 3/4 of cases. In 80% of patients, CSF shows raised proteins (>45 mg/dL) with a normal cell count.

Case A 30-year-old woman presents with lower abdominal pain; she is thought to have pelvic inflammatory disease (PID). She admits to prostitution and illicit drug use. Labs on admission reveal blood glucose of 260 mg/dL, a positive HIV screen, and a non-reactive RPR. Aside from the abdominal/pelvic pain, the admitting physical is also notable for moderate obesity, the absence of pronounced lymphadenopathy, and an erythematous macerated rash in the intertriginous distribution. The patient reports her rash is "really itchy and wet all the time," and it began within the last 3 months; it is now at its worst. She also reports intense itching of the vulva over the last few weeks. Question The intertriginous rash is most likely a manifestation of what condition?

Correct answer: Candidiasis Explanation Candidiasis occurs because of overgrowth of this group of yeasts in particular areas of the skin, those that are chronically wet, and especially in the intertriginous zones of the groin and beneath pendulous breasts. Patients with diabetes, chronic intertrigo, and cellular immune deficiency (as in HIV) are particularly susceptible. Obese patients have more intertriginous areas and are thus at risk. The rash is said to be macerated (displaying a waterlogged or soaked appearance characteristic of the dead surface skin). It is also described as erythematous, with marked inflammation, and it may be in a "satellite lesion" distribution, i.e. smaller lesions a few cm out from the larger one, which the "satellites" appear to orbit. The lesions may show scaling, and skin scrapings will show pseudohyphae and yeast forms. The rash of secondary syphilis can indeed manifest as moist pink lesions in the intertriginous regions. This occurs in about 10% of patients with secondary syphilis and the lesions are called condylomata lata. Secondary syphilis is not the best answer, however. Syphilis is notorious for masquerading as other diseases, but there are certain clues to look for that are more typical of syphilis. The rash, which may be subtle, usually manifests as symmetric mucocutaneous lesions with some truncal distribution. Furthermore, nontender generalized lymphadenopathy is usually present, and the primary chancre of syphilis will still be present in about 15% of patients with secondary syphilis. Finally, RPR should be positive in a patient with secondary syphilis. Other factors to consider are the patient's obesity, diabetes, reports of vulval pruritus consistent with candidal vulvovaginitis and positive HIV status. In summary, secondary syphilis would be an appropriate consideration in this patient, but it would not be the best answer. Of parenthetical note, IV drug users have a high (upwards of 25%) false positive rate by reagin-type tests such as RPR. Psoriasis is a very common chronic inflammatory condition of the skin. The lesions are variably pruritic and are characterized by sharply demarcated papules and rounded plaques. A silvery scale is frequently observed covering the erythematous plaques. Depending on the distribution and character of the lesions, psoriasis can be further categorized into several sub-groupings. The most common subtype (plaque type) of psoriasis is usually distributed on the elbows, knees, sacral area/gluteal cleft, and scalp, but there is also a less common variety known as inverse psoriasis. This is where the plaque lesions form in intertriginous regions in addition to scalp, palms, and soles. Because of the moisture in the intertriginous areas, scales may not be evident. Other forms of psoriasis include eruptive or guttate psoriasis as well as some variants where the lesions are more pustular in character. The etiology of psoriasis is not well defined, but there may be some association with certain medications, such as lithium and beta-blockers. Atopic dermatitis is the skin's reaction to allergy (food, asthma, animal dander, etc.). In children and adolescents, it is frequently localized to the flexural skin creases of the antecubital and popliteal fossae. Skin injury is more often a result of scratching than of the atopic process itself, and these individuals have a higher incidence of Staphylococcus aureus skin infections than do unaffected patients. Patients with atopic dermatitis are advised to avoid irritants and to keep skin moist (which includes avoiding hot showers and profuse scrubbing). Treatment often includes the judicious use of low-dose topical glucocorticoids and conservative administration of antihistamines to reduce the itching. Intertrigo is not one of the listed answer choices, but discussion here may be helpful. Stedman's Medical Dictionary (26th edition) defines intertrigo as: "irritant dermatitis occurring between folds or juxtaposed surfaces of the skin, as between the buttocks, between the scrotum and the thigh, beneath pendulous breasts, etc., caused by friction, sweat retention, moisture, warmth, and concomitant overgrowth of resident microorganisms, and occurring in young children and obese adults." As such, a patient with intertrigo could indeed have candidiasis, but a patient with candidiasis does not necessarily have intertrigo. Lichen planus is a condition that produces primary lesions described as pruritic, polygonal, flat-topped violaceous papules. Lesions may show thin grey lines (Wickham's striae), and they have a tendency to occur on wrists and shins, but can occur anywhere on the skin. Mucous membranes including the buccal mucosa can be involved. The etiology is not completely understood, and the course is variable, but the lesions usually disappear spontaneously within several months to 2 years. Treatment may include topical glucocorticoids. Here is a list of each of the possible answer choices together with a simplified association: Atopic dermatitis—allergic, flexural creases Candidiasis—diabetic, chronic intertrigo Lichen planus—pruritic polygons Psoriasis—silvery scaled papules and plaques Secondary syphilis—the masquerader

Case A 23-year-old man presents with a 2-day history of a red, swollen, and painful right knee. The pain and swelling have been progressing steadily since they began. He has also been feeling feverish for the last few hours. He admits to having unprotected sexual intercourse with multiple women over the past few months. On examination, his temperature is 100.4°F (38°C), and BP is 110/70 mm Hg. Cardiovascular, respiratory, and abdominal exam are normal. His right knee demonstrates clinical signs of an effusion, with severe tenderness and surrounding muscle spasm. An aspirate of the joint fluid yields the following findings: WBC count- 60,000/cc, neutrophils 95% No crystals Gram stain is negative Synovial fluid culture: Gram-negative diplococci on Thayer-Martin media Question Considering the probable diagnosis, what is the most appropriate pharmacotherapy?

Correct answer: Ceftriaxone Explanation Ceftriaxone is the treatment of choice for gonococci. Several clues in the vignette point towards gonococci as being the causative organism: the Gram-negative diplococci, Thayer-Martin media, and the young sexually active patient all point to gonococci. Clindamycin is used in the treatment of anaerobic infections and mixed infections. Fluconazole is an antifungal; therefore, it does not work on bacteria. Ampicillin is incorrect. While penicillins were effective in the past, the majority of strains of gonococci are now penicillin resistant. Vancomycin is reserved for use against Staph aureus resistant to Methicillin as well as the treatment of C. Diff colitis.

Case Ico-delete Highlights A 22-year-old male off-shore medical student presents with a 2-day history of dysuria accompanied by a painful genital lesion that has formed into an ulcer. He also reports fever with a recorded oral temperature of 101°F, generalized malaise, myalgias, and headache. He denies lesion blistering, and there is no previous history of this condition. He has otherwise been in good general health. He is heterosexual and admits to multiple sexual partners in the recent past with inconsistent use of condoms. T=100.9°F; BP=130/80; P=70; RR=12. Pertinent findings on the physical examination are relegated to the genital exam. Exam reveals a non-circumcised penis with a non-indurated ulcer of the penile frenulum and foreskin, sharply defined borders, undermined edges, and purulent yellow-gray exudative discharge. There is bilaterally tender inguinal adenopathy. Testicles are descended bilaterally with no lesions or masses. Question What is the most likely diagnosis?

Correct answer: Chancroid Explanation Chancroid (also known as soft chancre) is a bacterial sexually transmitted infection characterized by painful sores on the genitalia. The genital lesions of chancroid are caused by Haemophilus ducreyi and are characterized by painful ulcers and tender inguinal adenopathy. The presence of suppurative inguinal adenopathy is almost pathognomonic for H. ducreyi infection. It is rare in the United States; however, in the early and mid-1990s, a series of epidemic outbreaks of chancroid occurred in United States urban areas associated with crack cocaine use and prostitution. The CDC recommends that diagnosis of chancroid be based on 3 criteria: The presence of painful ulcer(s) Negative serologic tests for syphilis Negative HSV testing of ulcer exudates The initial ulcer may be mistaken as a "hard" chancre, the typical lesion of primary syphilis, as opposed to the "soft chancre" of chancroid. Approximately 1/3 of the infected individuals will develop enlargements of the inguinal lymph nodes. Half of those who develop swelling of the inguinal lymph nodes will progress to a point where the nodes rupture through the skin, producing draining abscesses. The swollen lymph nodes and abscesses are often referred to as buboes.

Case A 5-week-old male infant presents with a 2-week history of prominent cough, nasal congestion, and wheezing. His symptoms have been getting progressively worse. Yesterday, the patient's mother took her son to her primary care doctor. The doctor started albuterol nebulizers every 4 hours and told her that the child had a viral infection and would get better. His mother is now particularly concerned that her child has had dry diapers for over 15 hours. The child has been irritable during this time. He has not had any fevers, vomiting, or diarrhea. The child is not tolerating breastfeeding or bottle-feeding well. The physical exam shows that the child is acyanotic and alert. The temperature is 97.8°F (36.5°C), respirations are 40/minute, and the pulse is 119/minute. There are no lymph nodes observed, and his tympanic membranes appear normal. There are rales noted diffusely on auscultation. The mucous membranes are moist and the skin has good turgor. You also detect conjunctivitis. Blood work is obtained; including a set of blood cultures, and a urine culture. The following lab values return: Hemoglobin 12.6 mg/dL Hematocrit 37.1% Platelet count 204 x 109 Eosinophilia count 6% The chest film demonstrates interstitial infiltrates and hyperinflation. Question What is the organism causing the infant's symptoms?

Correct answer: Chlamydia trachomatis Explanation Chlamydia trachomatis is seen in infants 3-16 weeks old. They are usually sick for several weeks. On exam, the infant is nontoxic, afebrile, usually tachypneic, and has a prominent cough. Conjunctivitis is seen in about 50% of cases. The chest film demonstrates diffuse interstitial patches. Human parvovirus (or parvovirus B19) causes erythema infectiosum. The disease typically affects children older than the patient. It presents as a low-grade fever, a facial rash with a slapped-cheek appearance, and a lacy, reticular-like maculopapular rash on the trunk and extremities. Gastrointestinal upset, coryza, and myalgia are associated symptoms. Parainfluenza virus presents as upper respiratory infections of several days duration. Symptoms include a hoarse, croupy, cough and inspiratory stridor. The child is restless with retractions evident in severe infections. It usually resolves in 48-72 hours, but should it progress longer, it can manifest as laryngeal obstruction. Steeple sign is seen on a lateral X-ray of the neck, with glottic and subglottic narrowing. Respiratory syncytial virus begins with pharyngitis and rhinorrhea, which are followed by 1-3 days of coughing and wheezing. In addition to wheezing, diffuse rhonchi and fine rales can be auscultated. The chest film is normal. Chlamydial infections may be differentiated from RSV by conjunctivitis and a subacute onset. Staphylococcal pneumonia has a sudden onset; the child appears toxic and febrile, with an expiratory wheeze at onset that simulates bronchiolitis. There may also be signs of abdominal distress, tachypnea, dyspnea, and localized or diffuse bronchopneumonia. There is prominent leukocytosis.

Case A 33-year-old man with no significant past medical history presents with an enlarging ulceronodule of his right thumb over the past 2 weeks associated with a low grade fever and malaise. He recalls that he had cut his right thumb adjacent to the site of the ulcer 2 weeks ago while cleaning an aquarium at his work at a local pet store. He had applied a topical over-the-counter antibiotic ointment, however, the lesion was not improved. He has not been able to flex or extend at the affected digit over the past 3 days. There is associated swelling and mild tenderness. He also states that nodules seem to have been "popping up his forearm." His physical exam reveals the below-noted findings. Question What pharmacotherapeutic intervention would be most appropriate for this patient's condition?

Correct answer: Clarithromycin and ethambutol Explanation The correct response is clarithromycin and ethambutol. This patient's most clinical presentation is most consistent with a Mycobacterium marinum infection. Once recognized M. marinum infection is highly responsive to antimicrobial therapy and is cured relatively easily with any combination of a macrolide, ethambutol, and a rifamycin. Therapy should be continued for 1-2 months after clinical resolution of isolated soft tissue disease; tendon and bone involvement may require longer courses in light of clinical evolution. Other drugs with activity against M. marinum include sulfonamides, trimethoprim-sulfamethoxazole, doxycycline, and minocycline. Penicillin VK inhibits the biosynthesis of cell wall mucopeptide; it is indicated in the treatment of streptococcal infections, actinomycosis, erysipelas, periodontal infections, and rheumatic fever. Metronidazole inhibits nucleic acid synthesis by disrupting DNA and causing strand breakage. It is amebicidal, bactericidal, trichomonacidal. It is indicated in the treatment of anaerobic bacterial infections, sexually transmitted infections, bacterial vaginosis, colorectal infections, trichomoniasis, amebiasis, and off label for giardiasis, H. pylori infections, nongonococcal infections, PID, perianal Crohn's disease, and perioral dermatitis.

Case A 36-year-old male soybean farmer presents with the inability to walk. The patient reports having received a puncture wound on his left foot several days earlier; it became infected and bloated. Examination of the foot reveals a deep, highly purulent wound that has necrotized and produces a foul odor. The wound is drained and irrigated, and a wet mount of exudate is examined microscopically. Numerous motile rods containing terminal endospores with a 'tennis racket' appearance are seen. Question What organism is most likely responsible for the infection?

Correct answer: Clostridium tetani Explanation Clostridia are anaerobic spore-forming Gram-positive rods that produce an array of toxins. Clostridium tetani spores are ubiquitous in soil, and are introduced into tissues by a puncture wound caused by a contaminated object. In the anaerobic environment of traumatized tissues, C. tetani spores germinate; vegetative cells produce toxins and proteolytic enzymes that contribute to a rapid development of necrosis. In individuals not protected by vaccination, infection may progress to localized or generalized tetanus. Upon germination of endospores, C. tetani produces a toxin (tetanospasmin) that blocks presynaptic neurotransmitter release, causing rigid paralysis. C. tetani cells have terminal endospores, resulting in a 'drumstick' or 'tennis racket' appearance. C. perfringens is a common cause of food-borne illness that may also colonize soft tissue and skin. C. perfringens also causes a crepitant cellulitis with gas formation, and a rapidly fulminating and often fatal form of fasciitis. C. perfringens can be differentiated from C. tetani microscopically by its centrally located endospores and lack of motility. C. botulinum also causes food-borne illness, often as a result of home or commercially-canned foods. As bacteria grow and lyse, they release botulinum toxin. When reaching peripheral cholinergic synapses, botulinum toxin prevents release of acetylcholine and causes a flaccid paralysis. C. botulinum can also cause wound infections, but this organism is less invasive than C. tetani. Microscopically, the presence of subterminal endospores distinguishes this C. botulinum from C. tetani. C. difficile is a frequent cause of antibiotic-associated colitis; it produces subterminal endospores. Bacteroides fragilis is present among normal flora; it is the most frequent cause of suppurative anaerobic infection. Bacteroides are Gram-negative; they do not produce endospores.

Case An 18-year-old man presents to a university health clinic with a very sore throat and headache. He has had severe pharyngitis for about 12 days. He has felt exhausted and anorexic for 3 to 4 weeks. On physical exam, he has a temperature of 40 degrees C and his BP is 118/75 mm Hg. On palpation, his spleen is slightly tender and moderately enlarged. Blood work shows relative and absolute lymphocytosis; it also shows some atypical cells. Most of the lymphocytes are identified as T cells. Question Based on these findings, what is the diagnosis?

Correct answer: Infectious mononucleosis Explanation The most common symptoms of infectious mononucleosis, which is caused by the Epstein-Barr virus (EBV), are pharyngitis, fever, and lymphadenopathy. It is frequently diagnosed in adolescents and young adults, and it is more common in settings such as high school or college. Approximately 1/2 of all patients have splenomegaly. These symptoms, combined with T-cell lymphocytosis, are highly suggestive of mononucleosis. The diagnosis can be confirmed with the 'monospot' test which shows the presence of heterophil antibodies. Fever may persist for 14 days or longer. Most patients recover within 3 to 4 weeks, but some experience malaise for months. The usual course of Streptococcal pyogenes pharyngitis is the resolution of symptoms after 3 to 5 days. A throat culture is the gold standard for this diagnosis. Influenza is characterized by an abrupt onset. In addition to the symptoms described above, patients frequently have myalgia and respiratory complaints such as cough and postnasal discharge. It usually resolves within 5 days. The most common clinical symptom of mycoplasmal pneumonia is acute or subacute tracheobronchitis. Crackles or wheezing may be heard, and patients may have small pleural effusions. Primary infection with HSV-1 can cause pharyngitis, and it usually results in ulcerative lesions of the posterior pharynx and/or tonsils. HSV antigens can be isolated in scrapings from the lesions. In 1/3 of cases, concomitant lesions of the tongue, buccal mucosa, or gingiva may occur in the course of HSV infection.

Question What is a likely cause of chronic meningitis in a patient with AIDS?

Correct answer: Cryptococcus neoformans Explanation Cryptococcus neoformans is a type of yeast with large, polysaccharide capsules. It produces a self-limited or chronic pneumonia, but the most common clinical manifestation of infection with this fungus is chronic meningitis. Although patients with impairment in cell-mediated immunity are at risk for cryptococcal meningitis, some patients with this syndrome have no identifiable immunodeficiency. Cryptosporidium causes diarrhea. The best diagnosis is sugar flotation and stool for acid-fast stain. Fever and hemolysis are characteristic for protozoan disease caused by Babesia microti. Transmission occurs by tick, and peripheral blood smear is used for diagnosis. Actinomyces inhabits the mouth, gut, and vagina. It produces cervicofacial osteomyelitis and abscess, pneumonia with empyema, and intra-abdominal and pelvic abscess. It is associated with intrauterine contraceptive devices. Candida species are oval yeast that often colonize the mouth, gastrointestinal tract, and vagina of healthy individuals. They may produce disease by overgrowth and/or invasion. Candida stomatitis (thrush) often occurs in individuals who are receiving antibiotic or corticosteroid therapy, or who have impairments of cell-mediated immunity. Candida-caused vulvovaginitis may occur in this same setting; however, it is also seen among women with diabetes mellitus or with no apparent predisposing factors.

Case A 58-year-old man with a history of chronic renal failure presents with a 2-week history of night sweats, weight loss, a productive cough, and shortness of breath. The patient had a positive PPD skin test 6 months ago, but he was never treated. You order a chest X-ray and find apical infiltrates. You collect a sputum sample and order an acid-fast stain and culture. The acid-fast stain comes back as acid-fast bacilli seen on smear. Question Assuming that your patient is HIV-negative, what is the initial phase of therapy?

Correct answer: Daily isoniazid, rifampin, pyrazinamide, and ethambutol until sensitivities are known Explanation Initial treatment for HIV-negative patients with tuberculosis (TB) is daily isoniazid, rifampin, pyrazinamide, and ethambutol until isolate sensitivities are known. Once sensitivities are known, other listed medications can be discontinued. The standard regimen is for 6 months. Supplementary riboflavin is not indicated, but supplementary pyridoxine is indicated in order to reduce central and peripheral nervous system side effects due to isoniazid treatment. Streptomycin should be used with caution in older patients and in those with renal diseases

Case A 16-year-old girl was admitted to the hospital 2 days ago with the diagnosis of fever, the cause of which is not known. She has a 1-week history of fever, chills, headache, anorexia, nausea, vomiting, and myalgia. She developed a rash yesterday. She denies abdominal pain or diarrhea. She has had no previous medical problems, hospitalizations, or surgeries. The week prior to the onset of symptoms, the patient went camping in Texas with her church youth group. No one else from the group is ill. Upon admission to the hospital, her initial laboratory results revealed leukopenia, anemia, and elevated liver enzymes. While performing a physical exam, you unexpectedly discover a tick embedded in the girl's scalp. Question After sending the appropriate diagnostic studies to the laboratory, what medication should be started empirically?

Correct answer: Doxycycline Explanation Tick-borne illnesses, such as Rocky Mountain spotted fever (RMSF) and ehrlichiosis, are often the cause of a fever that does not have an identifiable source. It is of extreme importance that children presenting with fevers that have no discernable reason receive a thorough inspection of their skin. This is especially true for those who live in, or have traveled in, areas where tick-borne illnesses are common. If no other cause can be identified, the current recommendation is to begin doxycycline when a patient has clinical findings of either RMSF or ehrlichiosis. Chloramphenicol is not currently recommended because ehrlichial susceptibility is questionable; also, a controlled, retrospective study has shown excess mortality in chloramphenicol-treated patients compared with doxycycline-treated patients. Gentamicin, clindamycin, and penicillin are common antibiotics used for other tick-borne illnesses (tularemia, babeiosis, and relapsing fever, respectively), but they are not recommended for empiric treatment of patients such as the one in the vignette.

Case A 7-year-old boy presents with a 3-day history of fever, abdominal pain, and severe diarrhea. The diarrhea is described as watery and bloody. Before falling ill, the boy was at a friend's birthday party and ate some chicken. His stool demonstrates many WBCs and S-shaped bacteria that are motile. Gram stain results indicate gram-negative rods. Question What antimicrobial agent should be used to treat this patient?

Correct answer: Erythromycin Explanation The clinical picture is suggestive of Campylobacter gastroenteritis. Common findings include fever, abdominal pain, and diarrhea characterized by loose, watery, and bloody stools. Stool examination reveals motile, curvy, or S-shaped rods. The treatment of choice in children with Campylobacter gastroenteritis is erythromycin. Metronidazole is effective for Trichomonas. Ampicillin is ineffective against Campylobacter. Ciprofloxacin is used for Campylobacter infections in adults. Doxycycline is effective for chlamydial infections.

Question A 10-year-old girl had a group A (β-hemolytic) streptococcal pharyngitis from which she recovered on penicillin therapy. A month later, she presents with fever, joint swelling, involuntary movements, and a skin rash. What valve is MOST commonly altered in her disease?

Correct answer: Mitral Explanation This 10-year-old girl has a classic presentation of acute rheumatic fever; the disease may develop after infection with group A streptococcal pharyngitis. Rheumatic heart disease most often affects the mitral valve.

Question What trophozoite demonstrates "falling leaf" motility in the wet mount?

Correct answer: Giardia lamblia Explanation Giardia lamblia is an intestinal flagellate that has both a trophozoite and cyst form. The pear shaped trophozoite of Giardia lamblia exhibits characteristic "falling leaf" motility on a wet mount. Trophozoites are bilaterally symmetrical with 2 oval nuclei containing large central karyosomes on each side of midline. It has 4 pair of flagella, midline axonemes, 2 median bodies, and a large ventral sucking disk for attachment. Trophozoites of Entamoeba histolytica have progressive directional motility while Endolimax nana and Dientamoeba fragilis trophozoites have non-directional motility. Trophozoites of Trichomonas vaginalis exhibit a non-directional, jerky motility.

Case A 60-year-old woman presents with pyrexia, malaise, and dry cough. 15 days ago, the patient had an upper respiratory tract infection with pyrexia. The chest shows an interstitial pattern of lymphocytic inflammatory infiltrate. Question What pulmonary infection is causing the patient's symptoms?

Correct answer: Influenza A Explanation The most likely pulmonary infection in this patient is influenza A pneumonia. An interstitial pattern of involvement is common for viral pneumonias. The elderly are at risk of Influenza A in the fall and winter. Streptococcus pneumoniae is classically a lobar or bronchopneumonia with polymorphonuclear exudates in the alveoli. Candida albicans is a complication of an immunocompromised patient, particularly those with neutropenia. It can appear as a granulomatous or a bacterial pneumonia. Actinomyces and Nocardia produce a chronic, abscessing pneumonia. Mycobacterium kansasii produces a morphologic appearance similar to tuberculous granulomatous inflammation.

Case A 25-year-old man, who claims to have never been acutely ill in his life, presents after developing an abrupt onset of fever, headache, and painful body aches. The patient is now developing a cough and sore throat. The cough is non-productive. It is the middle of January; his travel history is unremarkable, except for a ski trip within the state. There is no history of any insect or arthropod bite. The physical examination is notable for small, tender cervical lymph nodes, an elevated body temperature of 39°C, and red, watery eyes. Transient scattered rhonchi or localized areas of rales are noted. A direct fluorescent antibody test is ordered stat and comes back positive for the suspected etiological agent. Question Based on the abrupt onset, clinical findings, and laboratory result, what is the most likely diagnosis?

Correct answer: Influenza virus Explanation Influenza virus belongs to the family Orthomyxoviridae that contain 2 genera: influenza virus type A and influenza virus type B. There may also be another member of the family, influenza virus type C, which has yet to be officially classified as such. All types, subtypes, and strains have similar structure (80 - 120 nm in diameter, enveloped, and covered with surface projections or spikes). Classic flu symptoms have an abrupt onset after an incubation period of 1 - 2 days. The initial symptoms are systemic; they present as shaking chills, fever, headache, myalgias, malaise, and anorexia. Headache, gastrointestinal symptoms, and myalgias are the symptoms that are most pronounced, and they are related to the fever elevation (which can be between 100 - 104°F). Arthralgia is not uncommon. There is severe eye muscle pain when gazing laterally in some patients. There are usually respiratory symptoms present initially (e.g., dry cough and nasal discharge), which are overshadowed by the systemic symptoms. Early in the illness the patient physically appears toxic, the face appears flushed, the skin is hot and moist, and small, tender cervical lymph nodes can be detected. Transient scattered rhonchi, or localized areas of rales, can occasionally be found. Fevers usually last for 3 - 8 days. Coughs can last longer, and a convalescent period of 1 - 2 weeks or more is needed for full recovery. Influenza illness can lead to complications such as primary influenza viral pneumonia, secondary bacterial pneumonia, cardiac complications, Reye's syndrome, and toxic shock syndrome. Laboratory diagnosis is by viral culture, serological testing, rapid EIA (enzyme immunoassay), and DFA (direct fluorescent antibody) techniques. Antivirals such as amantadine, rimantadine, and ribavirin can be used to lessen the duration and severity of the symptoms. Influenza vaccines are the best method for preventing influenza illness. Mycoplasma pneumoniae is a short rod (about 10 x 200 nm) that has an organelle responsible for cell membrane attachment at 1 end. It has no cell wall, making it unable to be stained; it is resistant to beta-lactam antibiotics. The culturing process is slow for Mycoplasma pneumoniae (5 - 20 days), as opposed to other bacteria. Mycoplasma pneumoniae produces an atypical pneumonia syndrome. The organism, unlike most viral respiratory infections (which have a 1 - 3 day incubation period), has an incubation period of 2 - 3 weeks. Initially, the infection is upper respiratory in nature. It has an insidious onset, which includes symptoms such as fever, malaise, headache, and cough. The onset is gradual as opposed to influenza infections, and 5 - 10% of patients will progress to tracheobronchitis or pneumonia. The fever is usually 100 - 102° F, sputum Gram stain is positive for inflammatory cells but absent of any predominating organisms, and myalgia and gastrointestinal symptoms are generally rare. The physical appearance of the patient is not indicative of any terrible illness; hence, the origin and use of the term 'walking pneumonia'. Chest examination may produce little or no auscultative or percussive findings. Rales are minimally present. Pleural effusion occurs in 5 - 20% of patients. Chest pain, when present, is usually due to chest strain from coughing. Chest radiographs show patchy areas of lung infiltration. Cold agglutinin testing and complement fixation tests are the most commonly ordered laboratory tests to aid in the diagnosis. Cold agglutinin titers >1:64 are indicative of Mycoplasma pneumoniae infection. Tetracycline, its derivatives, and erythromycin are the drugs of choice in treating Mycoplasma pneumoniae. Respiratory syncytial virus (RSV) is an enveloped, single negative-strand RNA virus measuring 120 - 300 nm. Repeated infections with RSV are common, and the virus is highly contagious. Patients are generally infants who present with a lower respiratory tract illness, pneumonia, bronchiolitis, tracheobronchitis, or an upper respiratory tract illness; it is often accompanied by fever and otitis media. There may be severe hypoxia present that can be life-threatening. Fever can last 2 - 4 days and cough may be the most frequent and predominant sign. After several days of upper respiratory tract signs and deepening cough, it is not unusual for the onset of dyspnea, increased respiratory rate, and retractions of intercostal muscles. In bronchiolitis, expiration tends to be prolonged; the respiratory rate can reach 80/min. Intercostal retractions are also particularly prominent in bronchiolitis, which emphasizes the inspiratory obstruction of the lower airway exists, as well as the more obvious expiratory obstruction. On auscultation, the patient may have rhonchi, wheezing, and rales that are intermittently present and may fluctuate in intensity. Chest radiographs typically show multiple areas of interstitial infiltration and hyperinflation of the lung. Ribavirin is the drug of choice to treat RSV in infants; it is delivered by aerosol since oral (p.o.) administration can result in hepatic or bone marrow toxicity. Streptococcus pneumoniae is a Gram-positive staining cocci. It is catalase-negative, alpha-hemolytic on blood agar, appears as diplo-cocci that are 'lancet-shaped', and are bile esculin soluble or sensitive to the copper-containing compound optochin. It is the most common cause of bacterial pneumonia. Streptococcus pneumoniae is most common in the elderly, individuals with a malignancy, immunosuppressed individuals (e.g., HIV, drug use, chronic alcoholism, and organ transplant recipients), diabetics, asplenic individuals, and those with chronic liver, heart, and/or kidney disease. Penicillins and cephalosporins are the drugs of choice; however, resistant isolate-possessing penicillin-binding proteins (PBPs) have increased in frequency. Staphylococcus aureus pneumonia can result either from aspiration or hematogenous spread. The organism is a Gram-positive coccus, catalase-positive, coagulase-positive, predominantly beta-hemolytic on blood agar, and appears in characteristic grape clusters on Gram stain. It is common for Staphylococcus aureus pneumonia to occur in a patient that has been stricken by influenza a few days prior. This is especially true in the debilitated, immunocompromised, and the elderly. X-rays can show a continuous spectrum of findings; however, there are indicators that can point to a Staphylococcus aureus pneumonia (e.g., rapid cavitation of the lungs, poor response to therapy aimed at treating pneumococcal pneumonia, and the development of pleural empyema). A semisynthetic penicillin or vancomycin is the treatment of choice. Cases of empyema may require surgery, treatment by the IV route, and drainage of the empyema by the insertion of a chest tube.

Case A 17-year-old boy presents with pain in his wrists, elbows, and knees bilaterally. He has felt fatigued, and he has been unable to work his summer job as a cashier and bagger in his family's community grocery store for the past 2 weeks. He also reports intermittent fevers and a large rash on his back in the area of his right shoulder; the rash is depicted in the image. All of these symptoms have emerged in the last 4 weeks, after a weeklong backpacking trip in upper state New York. He has no significant past medical history. His only medication is acetaminophen daily for joint pain. He does not smoke tobacco; he does not drink alcohol or use illicit drugs. He has no known allergies. Physical examination reveals a thin male adolescent in no acute distress. Temp 99.1°F, P 100 bpm, RR 14, BP 120/70 mm Hg. Small, nontender, mobile lymph nodes are palpable in the neck and axilla bilaterally. There is a large, warm, erythematous patch with central clearing at the patient's posterior right shoulder region; it extends across the arm and axilla and measures approximately 25 cm in diameter. There is limited range of motion in his right wrist and left elbow. There were no gross focal neurologic deficits. Question What is the most likely cause of these symptoms?

Correct answer: Lyme disease Explanation Lyme disease is the most likely diagnosis given the arthritic clinical picture and history of backpacking in an endemic area. 90% of Lyme disease in the United States occurs in the Northeast or upper Midwest (New York, New Jersey, Connecticut, Rhode Island, Massachusetts, Pennsylvania, Wisconsin, and Minnesota). Classically, Lyme disease consists of 3 stages: Stage 1 features flulike symptoms (arthralgia, headache, malaise, and weakness) and the typical single skin lesion of erythema migrans (EM) or "bull's-eye" rash at the site of the tick bite. Stage 2, after a latent period, features a rash similar to the one described here and systemic symptoms similar to those of stage 1. Stage 3, after a varying prolonged asymptomatic period that ranges from months to years, features synovitis, arthritis, central nervous system impairment, dermatitis, keratitis, as well as neurologic and myocardial abnormalities. It is important note that a great percentage of cases do not follow this neatly described sequence. Moreover, a significant proportion of afflicted patients give no history of a tick bite. Pityriasis rosea is incorrect. Pityriasis rosea is a harmless pruritic, scaly skin disease that is often found in people 10 - 35yrs of age. Initially, most people develop 1 large scaly 'herald patch'; within 1 - 2wks, smaller pink patches can occur on the trunk, arms and legs. Rocky Mountain spotted fever is incorrect. RMSF is a tick-borne disease caused by the bacterium Rickettsia rickettsii. It is a potentially fatal illness that occurs in North and South America; it is characterized by nausea, vomiting, fever, headache, and abdominal pain. It can have a maculopapular skin rash in the late phase of the disease. Secondary syphilis is incorrect. The skin rash of secondary syphilis is typically characterized by symmetrical, reddish-pink, non-itchy papules, and nodules on the trunk and extremities, including the palms and soles. Pityriasis versicolor is incorrect. Pityriasis versicolor is caused by yeasts of the genus Malassezia, which may also be found on normal skin. It affects the trunk, neck, and/or arms; it is characterized by asymptomatic hypopigmentation in dark-skinned persons and hyperpigmentation in fair-skinned persons.

Case A 49-year-old man presents with chronic fatigue, headache, fevers, and muscle and joint pain. He describes transient "reddish spots", which can be quite large, that have appeared on his skin and then faded away. His symptoms developed about 4 months ago with no apparent cause; they have gradually gotten worse with the recent addition of the musculoskeletal pains. He is married and monogamous, and lives in a small rural community; nevertheless, he is concerned that he may have somehow contracted a communicable STD. Question What condition is causing the patient's symptoms?

Correct answer: Lyme disease Explanation The pathogenic agent of Lyme disease is a spirochete bacterium, Borrelia burgdorferi. It is passed to humans by the bite of the hard-shelled ticks of the Ixodes genus. Several days to a month after infection, a rash develops at the sight of the bite. Known as erythema chronicum migrans, this erythematous lesion often expands over the course of several weeks and typically appears with a red flat border and central area of clearing. The ECM lesion gradually fades over several weeks, but new transient lesions may subsequently appear in various areas. Early signs and symptoms include general malaise and severe fatigue with fevers and chills. Headache, musculoskeletal pains, myalgias, and lymphadenopathy are common. If untreated, later phases may include neurological symptoms, cardiac dysfunction, and an asymmetrical pattern of arthralgias and arthritis that may resemble early rheumatoid arthritis. Confirmatory diagnosis relies on serum antibody assays using ELISA or immunofluorescence. Antibiotic treatment is usually effective. Group A streptococcal bacteria infection initiating in the upper respiratory tract, causes acute rheumatic fever, generally in young children. Inflammatory changes may damage the heart, joints, blood vessels, and subcutaneous tissues. Clinical course usually begins with an episode of acute pharyngitis. After a few weeks, it is followed by fever and migratory polyarthritis. Typically, this involves painful swelling of the joint, which subsides after several days with no residual disability. Skin lesions may develop as subcutaneous nodules or a rash called erythema marginatum, which usually has a bathing-suit distribution. The rash has prominent erythematous margins and may expand, but transient lesions developing in other areas are not expected. Myocarditis often develops. It may involve friction rubs, arrhythmias, cardiac dilation, or valvular damage. Recurrent episodes may appear, and they often follow the same pattern as the initial episode. Culture or serological testing can be used for confirmation. Treatment with antibiotics is usually successful. Alcoholic cirrhosis of the liver typically leads to clinical symptoms of general malaise, weakness, weight loss, and loss of appetite. Later, jaundice, ascites, and peripheral edema develop. The general syndrome usually takes decades of abuse to develop, and it progresses slowly with deteriorating hepatic function and the development of secondary problems associated with portal hypertension. Malaria is caused by the protozoan parasite Plasmodium, and it is transmitted by the Anopheles mosquito. Several different types are possible, depending on the particular infecting specie. General symptoms are flu-like and include headache, nausea and vomiting, myalgia, photophobia, and anorexia. The chills, fevers, and rigors reappear periodically as the cycle of infection, replication, and RBC lysis progresses (36, 48, or 72 hours is typical, depending on the species involved). Diagnosis is by microscopic examination of blood smears for the presence of the parasite. Serological tests are also available. Treatment is with chloroquine or parenteral quinine and primaquine. Polymyalgia rheumatica is characterized by pain and stiffness in the proximal muscle groups of the neck, shoulders and pelvis, usually symmetrically. Temporal arteritis with associated headache symptoms is common. Fever, malaise, anorexia, and weight loss may be present, as well as swelling of 1 or 2 joints. There is no associated rash expected. The erythrocyte sedimentation rate will be elevated, but there will be negative rheumatoid factor presence. Treatment with corticosteroids should produce a quick, favorable response.

Case An 8-year-old boy develops fever and an upper respiratory infection in late December. On physical examination, the pediatrician notes tiny white spots in his mouth near his second molars (see photo). A day later the boy develops a rash on his face. The rash consists of reddened macules and papules. Over time, the rash becomes confluent and moves downward over his whole body. Desquamation of the rash is not present. Question What is the diagnosis?

Correct answer: Measles Explanation Prior to the onset of the rash of measles, there is a fever, an upper respiratory infection, and Koplik's spots. Koplik spots are blue-gray spots that can be seen in the mouth. The rash is a maculopapular rash that may become confluent. Typically, the rash begins on the face and moves downward and outward over the whole body. The rash can go to the hands and feet. A patient with measles can have associated cough, adenopathy, and high fever. Winter and spring are the typical times of year for measles to occur. Hand-foot-mouth disease is a disease that occurs in children. The rash associated with hand-foot-mouth disease consists of vesicles on a reddened base. The hands and feet are the most common body parts affected. Oral ulcers or vesicles can be seen. Fever sometimes occurs with the other symptoms. Coxsackie virus is the etiologic agent of hand-foot-mouth disease. The typical time for hand-foot-mouth disease to occur is the summer and fall. Kawasaki disease is a disease that occurs in children. There is a high fever before the onset of the rash. The rash seen with Kawasaki disease is erythematous with desquamation. The entire body can be affected by the rash. Winter and spring are the typical times for Kawasaki disease to occur. Toxic shock syndrome is caused by a toxin produced by Staphylococcus aureus. There is accompanying high fever and hypotension. There is no predilection for a particular season. Rocky Mountain spotted fever is transmitted by ticks. With Rocky Mountain spotted fever, there is a fever, headache, and malaise. The wrist, ankles, palms of the hands, and soles of the feet are initially affected. The trunk can also be affected. Rocky Mountain spotted fever is caused by Rickettsia rickettsii. April through September is the typical time frame for Rocky Mountain spotted fever to occur.

Question A 3-year-old child presents because her mother found thin worms in her underpants. An infection of Enterobius vermicularis is suspected, so she is put on therapy with a drug that interferes with the synthesis and function of microtubules; this leads to inhibition of protein secretion and glucose transport in all species of intestinal nematodes. What drug is she being treated with?

Correct answer: Mebendazole Explanation Mebendazole is a synthetic benzimidazole that has a wide spectrum of anti-helminthic activity. It inhibits microtubule synthesis in nematodes, thus impairing glucose uptake. Clinical uses of mebendazole include: Pinworm infection Ascaris Trichuris Hookworm Hydatid disease (drug of choice is Albendazole) Adverse effects include nausea, vomiting, diarrhea, and abdominal pain. The drug is contraindicated in pregnancy. The primary use of praziquantel is in the treatment of infection with flat worms and flukes; it reduces the parasites' resistance to host immune mechanisms. Metronidazole is used primarily for entamoebiasis, anti-giardiasis, or anti-trichomoniasis; it covalently binds to protozoan guanine, preventing further replication of DNA. Niclosamide disrupts mitochondrial function in tapeworms by uncoupling oxidative phosphorylation. Acyclovir, an antiviral purine analog, is effective orally, intravenously and topically, incorporating into DNA to terminate the chain and to inhibit DNA polymerase.

Question Herpes virus can be detected by the Tzanck smear of the skin lesion. This test stains for the presence of which one of the following?

Correct answer: Multinucleated giant cells Explanation The Tzanck smear stains for the presence of multinucleated giant cells that are formed as a result of fusion of cell membranes due to insertion of viral proteins into the membrane. Tzanck smear does not detect the presence of inclusion bodies, dead cells, herpes DNA, or cell envelope.

Case A 25-year-old man with no prior history of any significant illness comes to your office presenting with a wound infection to the hand; he acquired the wound 3 weeks ago, and it has been worsening. Physical examination of the hand is remarkable for the presence of an initial wound site that is ulcerated and surrounded with papules that are blue-purple in color. Nodules are present (of lymphatic origin) ascending from the wound site on the forearm. The patient acquired the initial wound while at work at a pet store, where he accidentally cut his hand while cleaning out an aquarium. Material for bacterial, fungal and mycobacterial culture is obtained from the wound site. Special instructions on incubation requirements are given for the mycobacterial culture. Bacterial and fungal cultures prove to be negative; however, at day 7 there is 4+-mycobacterial growth detected that produces the following characteristics: GROWTH RATE NIACIN NITRATE REDUCTION OPTIMUM TEMPERATURE TWEEN HYDROLYSIS UREASE 7 DAYS NEG NEG 32° C POS POS Question Based on the clinical and isolate characteristics, what is the infection in the hand of this patient?

Correct answer: Mycobacterium marinum Explanation Mycobacterium marinum is a mycobacteria. The organism characteristically produces infections that are almost always located on superficial tissues and extremities. These infections are acquired through contact with Mycobacterium marinum-contaminated water; the contaminated water must enter broken or traumatized skin areas. Infection may follow trauma from fish spines or nips by crustaceans. Initial lesions begin at 2 - 3 weeks incubation and appear as small papules that enlarge and acquire a blue-purple hue. Suppuration then occurs and may progress to ulceration. Small nodules may appear in the efferent lymphatics that resemble those seen in sporotrichosis. The organism will grow optimally at 32° C with a growth rate of 7 - 14 days; it is niacin and nitrate-negative and Tween hydrolysis and urease-positive. Mycobacterium kansasii is a mycobacteria. The organism can produce a chronic bronchopulmonary disease (usually in adults) that closely resembles Mycobacterium tuberculosis. Symptoms are somewhat milder than with Mycobacterium tuberculosis; they are usually present with other underlying pulmonary conditions, such as COPD. Patients usually present with physical findings that are minimal, and most routine tests are normal. There may appear an ESR elevation with slight leukocytosis. The organism has a growth rate of 10 - 21 days, niacin-negative, nitrate-positive, Tween hydrolysis-positive, and urease-positive. Mycobacterium gordonae is a mycobacteria. The organism is not considered a human pathogen; it only rarely causes disease, and is usually considered a water contaminant. In cases where the laboratory has isolated Mycobacterium gordonae, multiple isolates from multiple patients is indicative of a water contamination problem, either in the media or decontamination procedure. The organism has a growth rate of 10 - 28 days, niacin and nitrate-negative, Tween hydrolysis-positive, and urease-negative. Mycobacterium avium-intracellulare is a mycobacteria and is often known as MAC. In non-AIDS patients, the disease is generally pulmonary in nature. The clinical manifestations are described as subtle and unobtrusive when other lung disorders are suspect or concomitantly present. 84% of patients will have a cough, 79% will have a productive cough, and 21% will have hemoptysis. Patients generally do not present as being acutely ill, and the features of chronic disease (e.g., anemia, hypoalbuminemia) are often absent. Disseminated disease is rare in non-AIDS patients. In AIDS patients the symptoms are much more acute, with fever, drenching night sweats, and weight loss. Patients will also present with diarrhea, and they will have elevated alkaline phosphatase levels. The organism has a predilection for the reticuloendothelial system and thus causes lymphadenopathy, splenomegaly, and hepatomegaly. Because of the reticuloendothelial system involvement, the isolation of MAC from the blood is frequent, and a diagnostic sign of dissemination. Growth rate is 10 - 21 days, niacin and nitrate-negative, and Tween hydrolysis and urease-negative. Mycobacterium tuberculosis is a mycobacteria. Mycobacterium tuberculosis, along with Mycobacterium bovis, is the causative agent of tuberculosis. Mycobacterium tuberculosis disease can manifest itself in a variety of infections, but is most commonly seen as pulmonary tuberculosis. Early pulmonary tuberculosis is asymptomatic, and is usually discovered on a chance chest roentgenogram. However, an increase in the bacillary population will produce nonspecific systemic reactions, resulting in anorexia, fatigue, weight loss, and night sweats. As the disease progresses, it is accompanied by chronic cough, sputum production, and, on occasion, hemoptysis. Chest pain will usually develop at this stage, and the patient will then seek medical attention. The organism has a growth rate of 12 - 28 days, is positive for both niacin and nitrate, is Tween hydrolysis-negative, and is urease-positive. Mycobacterium bovis is a mycobacteria. Mycobacterium bovis, along with Mycobacterium tuberculosis, and Mycobacterium africanum, is the causative agent of the infection tuberculosis. Mycobacterium bovis disease is similar to Mycobacterium tuberculosis disease in that it can manifest itself in a variety of situations, but it is most commonly seen as pulmonary tuberculosis. Early in the disease, there are few or no symptoms, and the disease may be discovered only by chance through a chest roentgenogram. As the disease progresses, there is the development of fever, weight loss, night sweats, chronic and productive cough, and hemoptysis. These symptoms, along with developing chest pain, eventually force the patient to seek medical attention. The organism has a growth rate of 21 - 40 days, is niacin and nitrate-negative, Tween hydrolysis-negative, and urease-positive.

Case A 21-year-old man is a recent immigrant from Somalia; he presents with back pain. The patient reports a dull aching pain localized to the T9 - T10 vertebral region. He describes intermittent pain for 6 months, and an episode earlier in the week of severe pain following heavy lifting. Physical examination shows tenderness from T9 - T12 with point tenderness at T10. The patient has normal vital signs, and laboratory measures are within normal limits. He denies fever, chills, and parenteral drug use. A myelogram shows a nearly complete block at T9 - T10, and an MRI reveals erosion of T9 and T10 vertebral bodies; there is a large surrounding paraspinal abscess. A tissue specimen obtained by biopsy reveals granuloma with necrosis. Gram stains of abscess drainage are uninformative. Auramine O staining and culture results confirm the diagnosis. Question What pathogen is most likely responsible for the patient's symptoms?

Correct answer: Mycobacterium tuberculosis Explanation The correct response is Mycobacterium tuberculosis. Currently, up to 15% of tuberculosis (TB) cases are extrapulmonary. Vertebral TB is a significant cause of paralysis and mortality; it results in kyphosis and spinal cord compression. A high index of suspicion for TB should accompany immigration status if the region of origin is endemic. Individuals emigrating from Somalia, for example, have a 10-fold higher incidence of TB than United States residents; about 40% of TB is extrapulmonary in these patients. An MRI or CT of tuberculous spondylitis usually reveals bony destruction with extensive paraspinous abscess formation. As in spinal abscesses with Brucella, Actinomyces, viral, fungal, and parasitic etiologies, tuberculous spondylitis results in a granulomatous reaction. Actinomyces and Brucella are likely to be recognizable by Gram stain, and Actinomyces can be identified by its filamentous morphology and the presence of characteristic sulfur granules in abscess drainage. While both Mycobacterium tuberculosis and M. leprae can be visualized by the fluorescent acid-fast stain Auramine O, infection with M. leprae is limited to cooler regions of the body (e.g., the skin and the upper respiratory tract). Calymmatobacterium granulomatis is an encapsulated Gram-negative rod; it is the etiologic agent of granuloma inguinale, a sexually transmitted disease found in developing countries.

Case A 24-year-old man presents with a 2-day history of skin rash on his back. He also notes pain in his joints and tiredness. He has just returned from a camping trip. Vital signs are normal. Examination reveals an expanding lesion with a concentric circle of erythema. The rash is shown in the image. Question If left untreated, what is a complication of early dissemination of this condition?

Correct answer: Myocarditis Explanation The correct answer is myocarditis. This patient's history and clinical picture are suggestive of Lyme disease. Lyme disease is an inflammatory disease characterized by a skin rash, joint inflammation, and flu-like symptoms; it is caused by the bacterium Borrelia burgdorferi and is transmitted by the bite of a deer tick. The figure shows the classic appearance of the rash, erythema chronica migrans. Early disseminated infection may cause secondary erythema migrans, disease of the nervous system (facial palsy or other cranial neuropathies, meningitis, and radiculoneuritis), musculoskeletal disease (arthralgia), and myocarditis (an inflammation of the cardiac muscle) that may lead to transient atrioventricular block. If left untreated, Lyme disease can cause late disseminated manifestations weeks to months after infection. These late manifestations include arthritis, polyneuropathy, and encephalopathy. Endocarditis is not seen in Lyme disease. It is may occur in IV drug abusers who are prone to developing septicemia. Restrictive cardiomyopathy is not typical of Lyme disease. Fibromylagia or chronic arthritis may be seen in a subset of patients, even after treatment, as part of a post-Lyme syndrome. It is not a specific feature of early infection. Endarteritis obliterans is a manifestation of syphilis.

Case A 55-year-old male firefighter suddenly develops fever, palpitations, and shortness of breath seven days after skin transplant for severe facial burns. He also has fever resistant to intravenous antibiotics therapy introduced at the onset of fever. Because of oropharyngeal lesions, he is on parenteral nutrition. On examination, transplant shows neither signs of infections nor signs of rejection. Patient appears lethargic, with a blood pressure of 80/40, pulse rate of 120, respiratory rate of 18, and temperature of 103.4. Stat CBC shows neutropenia and eosinophilia. Question For what reason would you order a blood culture for systemic fungal infection?

Correct answer: Neutropenia Explanation In a patient with antibiotic resistant fevers and signs and symptoms of systemic infection, including tachycardia, dyspnea, and hypotension, you should think about systemic mycosis due to cell mediated immune deficiency presented as neutropenia. Fever resistant to antibiotics indicates that there is inflammation; neutropenia indicates cell mediated immune deficiency; and the presence of certain predisposing factors (severe burns, impairment of oropharyngeal mucosa, parental nutrition, use of antibacterial drugs, surgery) raises the probability of fungal infection. Most probably, blood culture will reveal the presence of Candida. Also, infectious diseases associated with eosinophilia typically include parasitic (both protozoan and metazoan infestations) and fungal diseases. Patients are at risk of fungal infections when they are on long-term antibiotic therapy, wherein antibiotics disturb the balance between microorganisms, thus allowing the overgrowth of fungi. Fever resistant to antibiotics should prompt you to think about causes other than bacterial infection, particularly when the wound shows no signs of inflammation. Parental nutrition raises the probability of fungal infection, but it is not the reason for you to think about fungal infection in this instance. Surgery is a risk factor for lot of complications, including infections, but it is not a specific risk factor for fungal infection

Case A 31-year-old man presents with a tick bite. He describes locating a tick in the left axillary area while showering in the evening, and he denies that the tick was present the previous evening. He removed the tick with forceps and cleaned the wound with alcohol. Examination of the bite shows local erythema and mild induration of 3 - 5 mm diameter. The tick is identified as an adult female Ixodes scapularis. Question What is the recommended course of management for this patient?

Correct answer: No treatment is needed Explanation The correct response is that no treatment is needed. Lyme disease is a spirochetal infection caused by Borrelia burgdorferi and transmitted by ticks of the genus Ixodes. In the United States, Lyme disease is highly endemic in the northeastern and north-central states. Infection is often accompanied by the development of an expanding annular lesion (erythema migrans, or EM) at the site of the tick bite. A small amount of induration at the site of tick extraction, such as seen in this case, is normal and easily differentiated from EM. Studies have shown that infected ticks must feed for 24 - 36 hours before transmission of the spirochete is likely. Even in cases where ticks are attached for longer periods of time, the risk of transmission is insufficient to justify antimicrobial prophylaxis. Testing ticks for infection with B. burgdorferi is not helpful because detection methods are uncharacterized and the presence of the spirochete is uninformative. Serology would only be indicated if there is a risk or suspicion of infection. Other methods of detecting the spirochete (e.g., antigen tests) have not yet been developed.

Case A 32-year-old Hispanic woman presents with a 3-day history of vaginal pruritus. She is worried she has another infection. She was treated with penicillin 2 weeks ago for Group A streptococcus pharyngitis. The patient reports occasional sinus and urinary tract infections and always struggles with vaginal symptoms afterward. She has episodes of vaginal pruritus and thick white discharge approximately 3 times per year. She usually returns to her urgent care clinic for evaluation and treatment, then her symptoms resolve. She has no chronic medical conditions. She is non-obese and is not pregnant. On exam, the vaginal mucosa is inflamed and coated with thick, clumpy white discharge. On a wet mount slide treated with potassium hydroxide, you see pseudohyphae. The patient would like treatment for her current vaginal symptoms and advice in preventing her symptoms in the future. Question What is the best intervention for prevention of the patient's current condition?

Correct answer: Offer fluconazole, if antibiotics are indicated in the future Explanation This patient is presenting with a vaginal yeast infection. It would be most reasonable to offer fluconazole (an antifungal) if antibiotics are indicated in the future. Recent use of antibiotics is a risk factor for development of a vaginal yeast (usually Candida) infection, because the normal protective flora is diminished. Increased duration of antibiotic use is another risk factor for vaginal candidiasis. Of course, appropriate antibiotic prescribing guidelines should be followed. Several other conditions, such as AIDS, diabetes, and pregnancy, can predispose women to frequent vaginal yeast infections. This patient should be evaluated to ensure she does not have any of these other risk factors, as well. If her symptoms did not resolve with treatment, she could be treated for chronic vaginal candidiasis with lengthier regimens of antifungals. Cranberry juice is often recommended as a preventive method for development of urinary tract infection. It does not have a role in the prevention of yeast vaginitis. Yogurt consumption has been touted as a natural treatment and preventative measure for yeast vaginitis. The active lactobacillus cultures are thought to help re-colonize "protective" normal vaginal flora. However, the evidence base thus far does not support oral yogurt consumption as having any role in prevention of yeast infections. While antibiotics appear to be a trigger for this patient's development of yeast vaginitis, it would be inappropriate to withhold antibiotic treatment, especially in the case of her recent group A Streptococcus infection, due to the potentially serious sequellae of the infection, such as post-streptococcal glomerulonephritis and rheumatic fever. Nitrofurantoin is an antibiotic with good activity for targeting urinary tract organisms. It is useful for both treatment of lower urinary tract infections (UTIs) and prevention of UTIs. It is sometimes given prophylactically post-intercourse for women who develop frequent UTIs. It does not have antifungal properties and would not prevent this woman's yeast infections.

Case A 24-year-old intravenous drug abuser who is HIV-positive and relatively noncompliant with medical care presents with a 4-week history of fever, dry cough, tachypnea, and dyspnea. There is a diffuse interstitial pattern on their X-ray. Refer to the image. Question What is the most likely cause of the patient's symptoms?

Correct answer: Pneumocystis jiroveci Explanation The patient most likely has pneumonia caused by Pneumocystis jiroveci, which is a relatively common opportunistic infection in patients who are infected with the human immunodeficiency virus. Diffuse, bilateral alveolar shadowing, involving mainly the central lung fields, is the typical picture on chest X-ray. Microscopy of lung tissue, or bronchoalveolar lavage, gives the conclusive diagnosis. Pulmonary embolism (PE) is a medical emergency that presents with a sudden collapse and breathlessness. In the case of acute PE, the chest X-ray is often normal in appearance. Kaposi's sarcoma is another opportunistic disease common among patients with advanced HIV infection. Pulmonary Kaposi's presents with fever, nonproductive cough, and dyspnea. Typical chest X-ray findings include bronchial wall thickening and nodular opacities affecting the middle and lower lobes; however, Kaposi's sarcoma more commonly presents as violaceous skin lesions. Legionnaires' disease is transmitted by water droplets. It frequently originates in infected humidifier cooling towers and in stagnant water in cisterns and showerheads. The presentation is rapid, with gastrointestinal symptoms, mental confusion, hyponatremia, and proteinuria accompanying pneumonia. Chest X-ray findings may be unilateral or variable; pleural effusions are not uncommon.

Case A 25-year-old sexually active man presents with what he describes as an infection of his penis. His history is significant for having had sex with a prostitute in the past 2 months. He has no history of fever, chills, headache, joint pain, malaise, or anorexia. There is no history of drug use. Physically, he appears in very good shape and all vital signs are normal. Physical examination of the penis is significant for the presence of a chancre that is located on the glans penis. The base appears smooth with raised borders. Although painless, the chancre is sensitive to touch. There appears to be little to no exudate at the chancre. The patient also has the presence of regional lymphadenopathy, which is non-tender to the touch. The chancre is scraped for material to be examined. The scraped material is placed on a slide with sterile isotonic saline. It is observed under dark field microscopy, and it is reported by the laboratory as positive. Question What is the most likely diagnosis?

Correct answer: Primary stage syphilis Explanation Primary stage syphilis is caused by a spirochete called Treponema pallidum; it is characteristically initiated with a generalized bacteremia. This bacteremia occurs within 30 hours after infection. It is a sexually transmitted disease. The multiplication of the Treponema pallidum organism occurs at the site of entry where there is the formation of a "primary chancre." This primary chancre usually appears after an average 21-day incubation period; it quickly erodes and becomes indurated. Regional lymphadenopathy is present consisting of moderately enlarged, firm, nonsuppurative, painless lymph nodes, or satellite buboes. The chancre is described as having a smooth base, with a raised border that is firm. It is generally clean in appearance unless there is a secondary infection present. The chancre is painless but slightly tender to the touch. When obtaining scrapings for analysis, there is little or no pain and bleeding from the chancre. These scrapings are of value because they are generally rich in Treponema organisms that can be seen when observed under dark-field microscopy. The visualization of motile Treponema in scrapings under dark-field examination is diagnostic for Treponema pallidum. Both treponemal and nontreponemal antibodies appear 1-4 weeks after the appearance of the primary chancre. The primary chancre usually heals within 3-6 weeks and usually does not leave any scarring. The lymphadenopathy usually will persist for a longer period of time. Secondary syphilis symptoms will begin to occur often while the primary chancre is still present. Chancroid is caused by the bacteria Haemophilus ducreyi. It is characterized by one or more painful, exudative, indurated ulcers (associated with a tender lymphadenopathy) that eventually suppurate if untreated. It is a sexually transmitted disease. Haemophilus ducreyi is a small pleomorphic gram-negative coccobacillus with fastidious growth requirements. It is most common in uncircumcised non-Caucasian men, populations of low socioeconomic status, and poor hygienic conditions. The number of reported cases in women is very low; this may be due to a clinically inapparent carrier state, or asymptomatic and overlooked lesions. The organism has an incubation period (exposure to clinical disease) of 1 day to several weeks. The lesions are generally confined to the genital and perianal areas, with lesions on the mouth, fingers, and breasts being very rare. The appearance of multiple lesions (up to 10) is the norm. Detection can be made by culturing for the organism, but most routine clinical laboratories are not equipped for the isolation of this organism. Primary genital herpes is caused by the herpes simplex virus type 2 (HSV-2). The infection is most common in adolescents and young adults. Incubation periods are generally 2-7 days. Men generally present with vesicular lesions on an erythematous base. These lesions usually appear on the glans penis or the penile shaft. Women generally present with lesions that involve the vulva, perineum, buttocks, cervix, and vagina. Vaginal discharge is frequently reported. These primary infections are often associated with fever, anorexia, tender bilateral inguinal adenopathy, and malaise. The lesions of primary genital herpes may last for several weeks before healing is complete. Urethral involvement may result in dysuria or urinary retention. Detection can be made by viral culture. Localized candidiasis is most commonly caused by the yeast Candida albicans. Candida albicans is the most common clinically significant yeast isolated. This is an infection more commonly seen in women than men. Generally, the symptoms of localized candidiasis are pruritus, erythema, and a thick and adherent whitish discharge that contains curds. The vaginal pH is normal, as in comparison to the pH of bacterial vaginosis and trichomoniasis where the pH is elevated. The addition of 10% potassium hydroxide to vaginal discharge does not elicit any fishy odor, as is seen with bacterial vaginosis. The diagnosis can be made quickly by performing a KOH prep, which will allow you to visualize the presence of the fungi; however, 30-50% of women with candidiasis will have yeast numbers that are low in the discharge and thus produce false negatives. Culture methods are best utilized for a definitive diagnosis. The diagnosis of trichomoniasis is established by the presence of pear-shaped trichomonads on wet mount. Most men with this infection are asymptomatic, whereas in women, dysuria and a malodorous discharge are common presenting symptoms. Anogenital warts are caused by a human papillomavirus (HPV). These viruses produce anogenital warts that are flesh to gray in color, hyperkeratotic, and exophytic papules. The warts are usually attached to the skin by a short broad peduncle, or they may be sessile on the skin. Circumcised men will have these warts occurring on the penile shaft; in uncircumcised men, the warts will commonly appear in the preputial cavity. The urethral meatus is also involved in 1-25% of patients. In the homosexual population, the warts will occur most commonly in the perianal area. The occurrence of warts on the groin, scrotum, pubic area, and perineum is only occasionally encountered. Women are most apt to have the warts distributed over the posterior introitus and over the labia majora, minora, and clitoris. Most patients present with anogenital warts that are asymptomatic, but frequently there is itching and burning, as well as pain and tenderness.

Case A 17-year-old boy presents with tiredness, weakness, and a marked sore throat. There is no history of contact with others having the same symptoms. On review of symptoms, he admits to chills and excessive sweating, but he denies coryza, cough, nausea, vomiting, or diarrhea. On physical examination, there is pharyngitis, marked cervical lymphadenopathy, a fever of 38°C (100.4°F), and the spleen is palpable 3 cm below the left costal margin. There is neither hepatomegaly nor jaundice. Initial laboratory data reveals hemoglobin of 13%, a white blood count of 13,000, and a platelet count of 240,000. The differential white count includes 35% neutrophils, 5% bands, 50% lymphocytes, and 10% monocytes. Many of the lymphocytes have irregular or dark blue cytoplasm. Question Ico-delete Highlights What is the best short-term management of this patient?

Correct answer: Restriction of sport activities Explanation The diagnosis is infectious mononucleosis. It is a common disease among young adults (although patients may be of any age), and it may pass unnoticed or cause acute illness that is sometimes followed by lethargy for months. It is caused by the Epstein-Barr virus (EBV), which preferentially infects B-lymphocytes. This follows a proliferation of T cells, the "atypical" mononuclear cells, which are cytotoxic to EBV-infected cells. Most patients exhibit malaise, sweats, sore throat, and anorexia; diarrhea is rare. Splenomegaly occurs in 50% of patients. Hepatomegaly is uncommon. A rash is likely to appear if the patient is given ampicillin. The atypical lymphocytes (of which only a few may be seen) may also be present in many viral infections (especially CMV), toxoplasmosis, drug hypersensitivity, leukemias, lymphomas, and lead intoxication. It is caused by the Epstein-Barr virus (EBV), which preferentially infects B-lymphocytes. This follows a proliferation of T cells, the "atypical" mononuclear cells, which are cytotoxic to EBV-infected cells. Most patients exhibit malaise, sweats, sore throat, and anorexia; diarrhea is rare. Splenomegaly occurs in 50% of patients. Hepatomegaly is uncommon. A rash is likely to appear if the patient is given ampicillin. The atypical lymphocytes (of which only a few may be seen) may also be present in many viral infections (especially CMV), toxoplasmosis, drug hypersensitivity, leukemias, lymphomas, and lead intoxication. Treatment includes bed rest and restriction of sport activities to protect the fragile spleen, which is subject to rupture; alcohol should be avoided, and oral prednisolone should be given for severe symptoms or complications. Fatigue is exacerbated by physical exertion. Digitalis is a cardiac glycoside used to slow the ventricular rate in fast atrial fibrillation. It has a very weak positive inotropic effect and is generally not now used in heart failure. It also has a low therapeutic range, and older people are especially at risk of toxicity. High doses of steroids can cause suppression of the adrenal cortex, leading to adrenocortical insufficiency. This is a life-threatening event, presenting as shock, salt loss, and hypoglycemia; it is often precipitated by stress, such as infections and malignancies. This is the rationale behind tapering the dose of steroids whenever it is given for a prolonged period for inflammation. High doses of steroids are only given on a therapeutic basis for conditions that lead to adrenocortical insufficiency. Neither penicillin (a bactericidal antibiotic sensitive to a wide range of mostly gram-positive and certain gram-negative organisms) nor chlorambucil (an immunosuppressive drug used for advanced rheumatoid arthritis and other neoplastic conditions) has any role in the treatment of infectious mononucleosis. References

Case A 1-year-old boy presents with a rash that followed a 2-day fever of 102°F (38°C). His mother states, "I saw a pink, raised rash on his trunk when the fever ended. The rash looks like pink or red spots that turn white when I touch them." On physical exam, the child has a temperature of 98°F (36.6°C), and there is a diffuse maculopapular rash on the chest, abdomen, and thighs. The rash spares the face, palms, and soles. The child has no symptoms of upper respiratory infection, and the physical examination is otherwise normal. Question What is the most likely diagnosis?

Correct answer: Roseola Explanation The description of the presentation best fits that of roseola. Roseola infantum is the sixth of the traditional exanthems of childhood. The condition is an acute benign disease of childhood characterized by a history of a prodromal febrile illness lasting approximately 3 days, followed by defervescence and the appearance of a faint pink maculopapular rash. Erythema infectiosum, caused by human parvovirus B19, affects children ages 3-12. It exhibits a bright red facial (slapped-cheek) rash. Rubella is more common in older children and is accompanied by other systemic symptoms. If a woman has rubella during the first 3 months of her pregnancy, the virus can induce many different birth defects, some of which are quite severe. Among the defects doctors have seen in congenital rubella are eye defects (cataracts, glaucoma), microphthalmia (small, non-functional eyes), and heart problems (defects of the wall between the two sides of the heart, narrowing of the arteries to the lungs, and an open duct bypassing the lungs). In addition, doctors have seen ear problems (deafness caused by defects in the nerves and sound-sensing organs) and neurologic problems (including intellectual disability). Rubeola (or measles) usually has a prodromal phase of respiratory symptoms, and it is most common in children ages 5-9. Scarlet fever due to group A beta-hemolytic Streptococci would also be unusual in this age group, and it is usually accompanied by tonsillopharyngitis.

Case A 13-month-old boy presents with a rash. The mother tells you that the child has had high fevers over the past 4 days, although he has not had a fever for the past 24 hours. The rash began 6 hours ago; it started on his chest and back, and it spread to his neck, face, and arms. The child doesn't appear to be itchy, and he has been acting normally since the fever subsided. The mother denies cough, runny nose, vomiting, and diarrhea. The only medication that the child has taken is acetaminophen. On examination, the child is happy and playful. The only physical finding is the rash that is shown in the image. Question What is the most likely diagnosis?

Correct answer: Roseola Explanation This child exhibits the typical rash of roseola. Roseola infantum (also called exanthem subitum) is caused by Human Herpesvirus-6 and occurs almost exclusively during infancy. There is usually no prodromal period. The illness begins with high temperatures (averaging 103 F) for 3 to 5 days; the fever typically resolves rather abruptly. The rash appears within 12 to 24 hours of the fever resolution. The rash of roseola is rose-colored and begins as discrete, small, slightly raised pink lesions on the trunk and spreads to the neck, face and proximal extremities. The rash is not pruritic and no vesicles or pustules develop. The lesions may become confluent. Roseola is self-limited and the treatment is supportive only (e.g., antipyretics during the febrile phase). Rubella differs from roseola in that it has a distinct prodromal period, with prominent occipital and postauricular lymphadenopathy, and the low-grade fever is coincident with the rash. Measles (rubeola) is caused by a paramyxovirus and is manifested by cough, coryza, conjunctivitis, and Koplik's spots. The rash associated with measles is generalized, maculopapular, and erythematous, and occurs at the height of the fever. Drug hypersensitivity is a common condition that resembles roseola. Antibiotics are often given during the initial febrile phase of roseola before the rash appears. The rash associated with drug hypersensitivity is morbilliform and pruritic; it disappears after the offending drug is discontinued. Varicella (chickenpox) is caused by a human herpes virus and is still seen despite the availability of a vaccine. Often, there is a prodrome of fever, malaise, headache, anorexia, and mild abdominal pain 24 to 48 hours before the rash. The lesions start on the scalp, face, or trunk, and they consist of intensely pruritic erythematous macules that develop to form clear, fluid filled vesicles. The lesions then umbilicate and crust over while new crops form elsewhere on the body.

Case A 21-year-old male college student presents in the student health office with a maculopapular rash that began 1 day ago. He explains that the rash started on his forehead and has spread down to his trunk and extremities over the past 24 hours. Upon further questioning, you learn that he also experienced a 2-3 day history of mild fever, aches, and pink eye that has since resolved. For religious reasons, his parents refused immunizations for him. Physical exam reveals a maculopapular rash scattered about the face and trunk, with no oropharyngeal lesions noted. Mildly enlarged, slightly tender anterior cervical adenopathy is noted. The remainder of the physical exam was unremarkable. Question What is the most likely diagnosis?

Correct answer: Rubella Explanation Rubella is caused by a virus and causes a systemic illness. It begins with a prodromal period of mild fever, body aches, and conjunctivitis; it then develops into a fine maculopapular body rash that begins on the head and face and rapidly moves to the rest of the body. This prodromal period is rare in young children but more common in adolescents and in adults. It occurs about 1-5 days before the rash is seen. Rubeola is not correct, as fever associated with the illness is often high and patients are generally more ill-appearing. Koplik's spots, pathognomonic for rubeola, are not present in this patient on examination. Varicella and infectious mononucleosis are not likely, as their clinical presentations are different than how this patient presented. Varicella infection typically begins with fever and malaise and vesicular lesions that quickly break forming ulcerations. A maculopapular rash begins on the trunk, face, and scalp immediately forming vesicles that eventually form crusts. Infectious mononucleosis typically causes fever, sore throat, and fatigue with painful, posterior cervical lymphadenopathy and hepatosplenomegaly. The occurrence of a rash is much less common. Fifth disease occurs in small children and typically has a slapped cheek appearance.

Question A 35-year-old woman with acute lymphocytic leukemia and undergoing chemotherapy suddenly developed small vesicular eruptions across her left breast. The eruptions were asynchronous, painful, and lasted 6 days before clearing. She scratched the affected area, which only intensified her discomfort. The affected area developed small vesicles which progressed to ulcerated and crusted pustules, causing her clothing to stick to the open sores. She had a similar occurrence last year. The appearance of localized cutaneous lesions over a specific dermatome is most characteristic of what agent?

Correct answer: Shingles Explanation The herpes zoster virus causes shingles. It is identical to the varicella virus, which causes chickenpox. Exposure to the varicella virus results in the establishment of a latent state in many individuals. The virus may remain dormant for many years before being activated and expressed as a different disease, shingles. Each year, over 300,000 individuals experience severe episodes of shingles. There are numerous factors that been attributed to the reactivation of the herpes zoster virus. Exposure to sunlight, chemotherapy, immunosuppression, menstrual cycle changes, trauma, depression, and anxiety are but a few of the factors reported to induce the expression of shingles. Rash-like lesions associated with each episode range from undetectable to severe. Scratching can cause secondary bacterial infections, which exacerbate the situation. Shingles affects a specific dermatome (portion of the body innervated by a segmental sensory dorsal root ganglion). Fluid recovered from the lesions contains active virus, capable of infecting unexposed individuals. The patient rarely autoinfects with new dermatomal presentation. Both humoral and cell-mediated immune responses restrict the spread of the virus to new body sites, but are unable to eradicate the virus from the body, since it is sequestered in protected ganglia. Interferon may stabilize the infection. Current treatments do not eliminate the infection. They only reduce the severity of the episode. Hyperimmune globulin is effective in suppressing varicella infections in cases of known exposure. Immunosuppressed individuals are at risk from varicella and would benefit from passive immunization. The attenuated chickenpox (herpes zoster) vaccine, first introduced in 1995, is relatively safe and effective. Prior to the introduction if this vaccine, over 12,000 people were hospitalized annually due to severe cases of chickenpox.

Question A 25-year-old man presents with a painless, ulcerated lesion on the glans of his penis. He admits he had unprotected sexual intercourse with a prostitute 3 days ago. Since that time, he has not had sexual intercourse. Other than his wife, he denies having had other sex partners in the last 12 months. Past medical history and family history are unremarkable. The patient has normal vital signs. Physical examination reveals an indurated, painless, non-secreting chancre at the tip of his penis. There is no urethral discharge. Enlarged, painless lymph nodes are palpated on both inguinal regions. The remainder of the examination is within normal limits. What is the best next step in management?

Correct answer: Single IM dose of penicillin G benzathine Explanation The correct response is single IM dose of penicillin G benzathine. Primary syphilis is indicated by an indurated, painless chancre on the penis. The chancre is a disease hallmark. Syphilis is caused by the spirochete Treponema pallidum, and it is transmissible during early disease through exposure to open lesions rich in spirochetes. Non-sexual infection is rare, but it can occur through personal contact, blood transfusions, and accidental inoculations. Primary syphilis develops within several weeks of exposure and involves 1 or more painless chancres. Secondary syphilis develops after the chancre is healed, and patients develop clinical illness associated with malaise, anorexia, headache, lymphadenopathy or rash. Diagnosis is established by serologic tests (RPR or VDRL). Tertiary syphilis is associated with cardiovascular, neurologic, and systemic symptoms. The incubation period of syphilis is 10 - 90 days, and symptoms usually develop within about 3 weeks after exposure. The patient indicated sexual contact with the prostitute 3 days ago, so the patient probably contracted the disease from his wife. The drug of choice for primary syphilis in immunocompetent hosts is single-dose penicillin G benzathine, given IM. If the disease duration is greater than 1 year, 3 doses of penicillin G IM should be given 1 week apart. Alternatives include doxycycline or erythromycin for 14 days. Intravenous penicillin is not indicated in primary syphilis, but it is used in neurosyphilis, where it is administered for 14 days. Single dose azithromycin is a first-line treatment for gonorrhea, but it is not used in syphilis management. Biopsy of the lesion is not required because dark-field microscopy of the lesion exudate is used to visualize spirochetes confirming the diagnosis. Because dark field microscopy is time consuming and a specialized skill, serologic tests such as RPR or VDRL are preferred. Lumbar puncture (LP) is not indicated in primary syphilis. LP is indicated when neurologic or ophthalmic signs and symptoms are present, and cerebrospinal fluid (CSF) data should be correlated with cardiovascular, neurologic, and systemic symptoms.

Question A 13-year-old girl presents with a very itchy foot. Laboratory tests show a microcytic anemia. A fecal sample for ova and parasite shows that there are the microscopic hookworm eggs in her feces. Where does the causative agent reside?

Correct answer: Small intestine Explanation Hookworm infection is caused by either Necator americanus or Ancylostoma duodenale. A common mechanism for humans to become infected is by walking barefoot in contaminated soil or handling fertilizer that contains human feces. The larvae are able to enter the skin, go through to the respiratory system, and ultimately end up in the small intestine. In the small intestine, the larvae develop into the adult worms and attach to the mucosa of the small intestine. 'Ground itch' occurs at the site of penetration. Hookworm infections are associated with the development of anemia. The anemia seen with a hookworm infection is usually microcytic hypochromic, as in this patient. The anemia is due to blood loss; sometimes, protein deficiency can also be seen. Occasionally, respiratory symptoms can be seen during the pulmonary migration. Diagnosis can be made by finding the eggs or larvae in a stool sample.

Case A 38-year-old man presents with nausea, vomiting, diarrhea, and abdominal cramping for the past 2 hours. The patient reveals that he had consumed meat that had been left uncovered for a night. Laboratory examination revealed that the meat was contaminated with Gram-positive circular, clustered organisms. Fairly large, yellow-colored colonies were formed when the organism was grown on mannitol agar media at 40°C. The organism showed positive results for coagulase enzyme activity and showed resistance to penicillin and methicillin antibiotics. Question Ico-delete Highlights What organism was the causative agent of the food poisoning?

Correct answer: Staphylococcus aureus Explanation The causative agent of food poisoning in this case is Staphylococcus aureus. S. aureus is a Gram-positive, aerobic, circular (cocci) organism that appears to be in pairs or in grape-like clusters on microscopic examination. S. aureus is found as normal human body flora colonized in the nasal passages. The organism is pathogenic, pertaining to its ability to produce and release toxins. It is resistant to high salt concentrations, especially sodium chloride (NaCl) and grows on mannitol salt agar forming large, yellow-colored colonies. S. aureus grows well in a temperature range of 15 to 45°C. The majority of the strains of S. aureus show positive results for coagulase test, but it is not a differential diagnostic test to detect the organism, as few strains of S. epidermidis also show positive results for the test. S. aureus is resistant to penicillin and methicillin. Individuals with S. aureus food poisoning have loss of fluid due to diarrhea; therefore, they are advised to drink plenty of water and electrolyte solutions. The preventive measures include proper washing of hands and maintaining good hygiene when preparing or handling foods. Clostridium perfringens is a Gram-positive, anaerobic, spore-forming bacilli (rod shaped). C. perfringens is commonly found as a soil microorganism. It is also found in the feces of infected persons and thus becomes a source of infection. C. perfringens food poisoning is due to the ingestion of the organism and is associated with intense abdominal cramps and diarrhea. The diagnostic tests include stool examination to detect the toxin or the organism in the infected person's feces. The organism can grow in a wide range of temperatures, and it is cultured in anaerobic liquid media in laboratories. There is no specific treatment for C. perfringens food poisoning, but the preventive measures include cooking food at high temperatures, rapid cooling of cooked foods, and hygienic handling of foods. Enterococcus faecalis, previously known as Streptococcus faecalis, is a Gram-positive, circular, facultative anaerobe. It is a normal inhabitant of the intestinal tract and female genital tract. It may cause endocarditis, urinary tract infections, bacteremia, wound infections, catheter-related infections, and intra-abdominal/pelvic infections. Escherichia coli are a common flora of the human body and are generally found in the gut of humans. E. coli are Gram-negative, non-spore forming bacilli. Most strains of E. coli are non-pathogenic, but strains called enterohaemorrhagic E. coli (EHEC) are responsible for foodborne illness due to the toxins they release. The symptoms include abdominal cramps and diarrhea, which may lead to hemorrhagic colitis. The organism grows in a temperature range of 7 to 50°C, but the optimum temperature at which the organism grows best is 37°C. Diagnosis of the infection is generally by microscopic examination of the stool from the infected person. Preventive measures include practicing hygiene when preparing food and using boiled and cooled water for consumption. Salmonella species are non-spore forming, Gram-negative bacilli. Salmonella are found in the intestinal tract of humans and other warm and cold-blooded animals. They are responsible for causing acute gastroenteritis as a result of foodborne ingestion or intoxication. They are transmitted to the humans via contaminated water and food. Salmonella typhi and Salmonella paratyphi are responsible for typhoid and typhoid-like fever in humans. Measures to prevent infection include hygienic practices when handling food.

Case A 17-year-old football player is being evaluated for fatigue and a sore throat. He denies coughing or fevers. He does not smoke or drink. In addition, he denies blood in his stool or urine and any dysuric symptoms. He practices safe sexual intercourse with his girlfriend. Physical exam demonstrates a blood pressure of 130/65 mmHg, a pulse of 72/minute, respirations of 16/minute, and a temperature of 98 degrees Fahrenheit (36.6 Celsius). The lung and heart sounds are normal. There is no cervical adenopathy. When you look at the pharynx, there is tonsillar enlargement with evidence of exudates. The abdominal exam demonstrates a spleen tip that is easily palpable 1.5 centimeters below the left costal margin. Results of a Monospot test are positive. Question What management scheme is best for this patient?

Correct answer: Symptomatic treatment only Explanation Infectious mononucleosis is caused by double-stranded Epstein-Barr virus or EBV (part of the Herpesviridae family). The incidence is highest in young adults aged 15-35 years of age. Symptomatic infections are common, and most adults are seropositive to the EBV. Symptoms include fever, headache, generalized fatigue, and malaise. Signs include lymphadenopathy (posterior cervical chain), hepatosplenomegaly, jaundice, periorbital edema, exudative pharyngitis, palatine petechiae, and rash. There is lymphocytosis with 20% atypical lymphocyte, positive heterophile agglutination (Monospot test) after the second week of illness, and a heterophile titer greater than 1:56. Liver function tests are elevated. There is also granulocytopenia, thrombocytopenia, and hemolytic anemia. No specific treatment is required for mild cases, and symptoms usually improve in 2 to 4 weeks. No contact sports should be carried out in view of the splenomegaly. Treatment with amoxicillin or ampicillin may lead to severe maculopapular rashes, and it should be avoided. Specific antiviral treatments (e.g., acyclovir) are not beneficial. In severe cases in which the pharyngitis may obstruct airway, a 5-day course of steroids may be needed

Case You are covering a weekend shift at a local inner city free clinic. Your first patient of the day is a 41-year-old woman who presents with what she describes as "a cough". The patient states that this cough just will not go away and has been present for several months. It has also gotten to the point that the patient is coughing up blood; she has been doing so for the last 4 days. She admits to unintentionally losing about 10 pounds in the last 2 months. Progressively worsening fatigue, night sweats, and chills are also present. She is a nonsmoker and lives in a rent controlled apartment with 3 adults and 4 kids. Question Given the most likely diagnosis and need for confirmation of the diagnosis, what laboratory study is viewed as the gold standard and the most sensitive test for confirming the diagnosis?

Correct answer: TB culture Explanation The correct response is TB culture. Tuberculosis (TB) is a chronic mycobacterial infection that most commonly affects the lungs but potentially can spread to other organs of the infected individual. TB is usually spread person to person via droplets after a cough or a sneeze. TB can be actively infectious, but also may have a latent phase; this is when the patient has the TB mycobacteria in his/her body does not have symptoms of the disease. There is are many groups of patient who have a higher risk of developing a TB infection, including those who live/work around people who have TB, medically underserved populations, and patients that may reside in more of a group setting, such as a nursing home or jail. Symptoms of TB initially may be just a nonproductive cough; as the disease is allowed to progress to hemoptysis. Other signs and symptoms may include chest pain, fatigue, loss of appetite, unintentional weight loss, fever, and chills or night sweats. To confirm the diagnosis of TB, a TB culture should be ordered and completed. An acid-fast bacilli (AFB) smear is usually completed to help support the diagnosis of TB, but it cannot by itself indicate diagnosis as other bacterial may display all positive result. AFB smear may also be used to monitor treatment of TB once it has been initiated. Tuberculin skin test, also known as purified protein derivative (PPD) or Mantoux test, is administered intradermally to test for prior exposure to TB. However, PPD testing cannot indicate active or latent/dormant TB. A chest radiography will often only demonstrated results of an acute granulomatous infection (Ghon complex) and again, does not confirm diagnosis. QuantiFERON-TB Gold is a whole-blood test that can detect TB in those patients recently exposed or who are suspected to have a TB infection; however, this test does not indicate if the TB infection is active or latent.

Question A 7-year-old boy from a family of Caribbean immigrants presents for a regular medical check-up. He is in good health. His immunization status is not known. What will be your recommendation regarding pertussis protection?

Correct answer: Tetanus and diphtheria toxoids and acellular pertussis vaccine (Tdap) Explanation There are pertussis vaccines for children, pre-teens, teens, and adults. The childhood vaccine is DTaP (the pertussis booster vaccine for adolescents and adults is called Tdap). DTaP (full-strength doses of diphtheria (D) and tetanus (T) toxoids and pertussis (P) vaccine) is given to infants and children ages 6 weeks through 6 years. Children should get 5 doses of DTaP, 1 dose at each of the following ages: 2, 4, 6, and 15-18 months, and 4-6 years. Children aged 7 through 10 years who are not fully immunized against pertussis (including those never vaccinated or with unknown pertussis vaccination status) should receive a single dose of Tdap. Tdap can be administered regardless of the interval since the last tetanus and diphtheria toxoid-containing vaccine. Herd immunity has an indirect impact on immunity. It reduces the transmission of infectious disease agents from immunized people to others, thereby reducing the impact of infection spread. This patient will provide herd immunity after the immunization. Influenza vaccine should be given every year, but it will not protect the child from pertussis. Vaccines containing the whole cell pertussis component (DTP) are no longer recommended for use in the United States. Even though most side effects of the vaccination are moderate, severe problems closely following DPT immunization led to the introduction of acellular pertussis vaccine. The most severe problems with DTP vaccine were symptoms of neurological damage (occurring within the first 7 days following vaccination), thereby raising the risk of permanent brain damage in children. Preventive antibiotics could be recommended to close contacts, including all household members of a pertussis patient, regardless of age and vaccination status. This might prevent or reduce the chance of getting pertussis. Your patient is not at risk of getting pertussis from close contacts.

Case A 27-year-old woman who is 18 weeks pregnant states that her 3-year-old daughter was just diagnosed with erythema infectiosum (Fifth disease). An antibody test is ordered, and the woman is found to be seronegative. She is healthy and has no signs or symptoms of parvoviral infection. Two weeks later, the test is repeated and the patient is IgM positive. Question With regard to the patient's fetus, what is the significance of this test result?

Correct answer: The patient should undergo serial fetal ultrasounds to monitor for the appearance of signs of hydrops fetalis. Explanation Erythema infectiosum, also known as Fifth disease (FD), is a common childhood exanthem caused by parvovirus B19. Spread by respiratory secretions, FD is often preceded by a prodrome of low-grade fever, malaise, pharyngitis, and coryza. FD is readily distinguished from other eruptions by a characteristic "slapped-cheek" rash. This presentation is notable for a bright red macular appearance that favors the malar surfaces, sparing the bridge, orbits, and mouth. Within 1-2 days, another rash develops; it is characterized by symmetric eruptions and blotchy areas that take on a reticular or lacy pattern. The rash may involve other areas of the skin, but most commonly evolves to the extremities and buttocks. For most children, FD is self-limiting and resolves spontaneously in 1-2 weeks, although the rash may recur after bathing, exercise, and sunlight. In adults, infection may cause an arthritis-like syndrome. In immunocompromised individuals, parvovirus B19 infection may cause anemia. No vaccine is available to prevent parvovirus B19 infection. For pregnant women, exposure to a child with FD represents a risk to the fetus. Children are most contagious before the rash develops, so exposure has often occurred by the time FD is detected. The pregnant patient should be tested for antibodies. Immunity is protective and lifelong, however only about 50% of the population is immune to parvovirus B19. Although the virus is not highly contagious, risk to those living with an infected individual is high. The pregnant patient who is seronegative should be retested in 2 weeks, whether or not symptoms are present. In about 15-30% of cases, maternal infection will be transmitted to the fetus, and the fetus will develop hydrops fetalis, a condition marked by swelling of the organs - especially liver - pleural and pericardial effusions, and severe anemia. Pregnant women who develop IgM antibodies - a marker of recent infection - should undergo serial ultrasound examinations to monitor for signs of fetal hydrops. It is possible to detect parvovirus in amniotic fluid; however, amniocentesis exclusively for the determination of fetal infection is not routinely performed due to the inherent risk associated with this test. If the fetus becomes infected, intra-uterine transfusions are often the treatment of choice.

Case A 12-year-old girl presents with a 1-week history of a rash. The rash is annular in appearance and is slightly erythematous. There is no history of allergies and no illness in family members. Vaccinations are up to date. The family recently returned from a vacation in the northeast. Physical examination is unremarkable except for the rash. There has been consistent lethargy and fatigue; headache, fever, and chills have been intermittent. Question What is the treatment of choice in this patient?

Correct answer: Tetracyclines Explanation Lyme disease is a tick-borne spirochetal disease caused by Borrelia burgdorferi. It is characterized by a distinctive skin lesion known as erythema chronicum migrans. Initial signs and symptoms often resolve in 3-4 weeks, and then there is a latent period of weeks to months. After the latent period, neurologic, cardiac, or musculoskeletal manifestations may appear. The classic neurological triad is meningitis, cranial neuropathy (including Bell's palsy), and peripheral radiculoneuropathy. Cardiac involvement may include AV block, pericarditis, or cardiomegaly. Migratory arthralgias and arthritis involving large joints, especially the knees, are the most common latent manifestation. Tetracyclines, penicillin, and erythromycin are all effective in resolving the symptoms during the acute period. The treatment of choice, however, is penicillin in children under 8 and tetracycline in children 8 and older. Studies suggest that early treatment, especially with tetracyclines, prevents late complications of Lyme disease. High doses of benzylpenicillin, cefotaxime, or ceftriaxone are used for later complications. Sulfa drugs and steroids are not recommended.

Case A 43-year-old woman is admitted for nausea, emesis, ataxia, and dizziness. Previous medical history reveals HIV positive status for 4 years, with the most recent CD4 T-cell count of 100/mm3. Vital signs and neurologic exam are normal. An MRI shows 2 ring-enhancing lesions of the basal ganglia, each approximately 1 cm in diameter. The patient is treated with sulfadiazine plus pyrimethamine, and she is discharged following rehydration. Within 1 week, the patient's symptoms improve; a 2nd MRI 4 weeks later shows a significant reduction in the size of the lesions. Question What is the most likely diagnosis?

Correct answer: Toxoplasmosis Explanation Toxoplasmosis is caused by Toxoplasma gondii, a sporozoan that is ubiquitous among animals. Humans become infected congenitally, or through ingestion of cat feces or undercooked meat. Toxoplasmosis is the 2nd most common central nervous system infection in persons with AIDS. Depending on the size and location of the lesions, patients may present with focal neurologic deficits, headache, seizures, or fever. Diagnosis is commonly presumptive, and the results of empirical treatment are evaluated. Serology may not be helpful because of high false-negative rates; 25% to 50% of patients with AIDS may be seropositive. The primary differential diagnosis for toxoplasmosis is lymphoma. Multiple lesions increase the likelihood of toxoplasmosis, as does a lesion location within the basal ganglia. Fungal brain abscesses and CMV encephalitis may cause abnormal radiologic findings, but symptoms would not be expected to respond to anti-protozoan drugs. Fungal brain abscesses in patients with AIDS may result from Aspergillus, Mucor, Rhizopus, andNocardia. Radiologic appearance and neuroanatomic location vary according to the fungus. HIV encephalitis, the most common CNS infection in AIDS (refer to table), can be differentiated radiologically from toxoplasmosis by the appearance of atrophy or hypodense areas.

Case A 38-year-old HIV-positive man presents with fever, headache, right hemiparesis, and expressive aphasia. Recent CD4 count is 45, but he is not on any prophylactic medications. Computed tomography scan (CT) reveals multiple ring-enhancing lesions in the parietal and frontal lobes. Question What is the most likely cause?

Correct answer: Toxoplasmosis Explanation The differential diagnosis for this patient should include all of the above; however, the presence of multiple ring-enhancing lesions on CT or MRI provides sufficient evidence for toxoplasmosis. A definitive diagnosis would be established serologically. Infection by Toxoplasma gondii is common, with seropositive rates varying from 7 - 80% worldwide.

Case A 36-year-old woman returning from a conference in Indonesia presents with profuse watery diarrhea. She reports having consumed fried rice, iced tea, and fruit at a street market several hours before her flight to the United States. The patient has 4 - 6 bowel movements daily, and they are preceded by cramps and nausea. She also has overwhelming thirst. Examination reveals dehydration, diaphoresis, orthostatic hypotension, and abnormal laboratory values for potassium (2.8 mmol/L). No leukocytes are seen in stool specimens, and cultures yield an oxidase-positive curved Gram-negative rod on blood and TCBS agar. Question What is the most likely etiology of the patient's illness?

Correct answer: Vibrio cholerae Explanation Vibrio cholerae is a comma-shaped Gram-negative rod. V. cholerae are oxidase-positive and grow on selective medium (TCBS), allowing their laboratory differentiation from Salmonella, Shigella, and Campylobacter. In patients infected with the cholera agent, stool samples lack leukocytes and show highly motile rods by darkfield microscopy examination. Epidemiologically, cholera should be suspected in a patient with profuse watery diarrhea, a history of travel to an endemic region, and a history of exposure (by consumption) to food or water potentially contaminated with sewage. Illnesses caused by V. cholera exotoxin range from mild abdominal cramping to life-threatening hypovolemic shock. Clinical signs and symptoms include profuse watery diarrhea 12 - 24 hours after exposure, abdominal pain, vomiting, anorexia, apathy, fever, malaise, seizures, and extreme thirst. Clostridium perfringens, a Gram-positive anaerobic rod, is a common cause of food poisoning in the United States. Vomiting and diarrhea occur 8 - 12 hours after ingestion of a heat-labile toxin, often in association with inadequately reheated meat. Fever is usually absent, and the illness resolves in about 24 hours. Clostridium difficile is a Gram-positive anaerobic rod that causes pseudomembranous colitis in patients treated with antimicrobials. Campylobacter jejuni, like Vibrio, is a curved Gram-negative rod. Gastroenteritis caused by C. jejuni occurs worldwide; in the United States, it is more common than gastroenteritis caused by Salmonella and Shigella combined. Fecal-oral transmission is common, particularly when associated with chicken consumption. Clinically, Campylobacter-induced illness differs from that caused by V. cholerae by a more insidious onset and the presence of a high fever and blood in the stool. Shigella sonnei causes dysentery; it is a significant problem in pediatric populations. Within 24 - 48 hours of ingesting Shigella, the patient often experiences acute onset fever, abdominal cramping, and profuse watery diarrhea. Like Campylobacter, Shigella diagnosis can be aided by the detection of leukocytes in stool specimens.

Question What noninvasive pathogen is involved in acute diarrheal illness?

Correct answer: Vibrio cholerae Explanation Major Etiological Agents in Acute Diarrheal Illnesses Invasive/destructive pathogens Noninvasive pathogens Bacterial toxins (food Poisoning) Shigella Salmonella Campylobacter jejuni Vibrio parahaemolyticus Yersinia enterocolitica Clostridium difficile Rotavirus Other viruses Entamoeba histolytica

Case A 21-year-old man presents with acute onset of pleuritic chest pain, which has been accompanied by 2-3 days of fever, chills, arthralgias, and myalgias. Four weeks ago, he had a severe sore throat and fever, but was not evaluated for these symptoms. Physical examination reveals a febrile patient in mild distress. A diastolic murmur is noted in the left 4th/5th intercostal space that radiates to the left axilla. A friction rub is also appreciated on exam. Laboratory results reveal an elevated erythrocyte sedimentation rate (ESR) and antistreptolysin antibodies. Question What is the most likely diagnosis for this patient's presentation?

Correct answer: Acute rheumatic fever Explanation The patient in the above scenario most likely has acute rheumatic fever. Rheumatic fever is a systemic immune process that is often a complication of undertreated b-hemolytic streptococcal infection of the throat, resulting in inflammatory changes in the heart, skin, joints, and other tissues. Clinical presentation of acute rheumatic fever is often an acute febrile illness and a migratory polyarthritis of the larger limb joints. Cardiac symptomatology is less common but can manifest as murmurs or a friction rub, indicating valvular and pericardial involvement respectively. The diagnosis of acute rheumatic fever is based on clinical features plus evidence of a preceding streptococcal infection. The Jones criteria for diagnosis of acute rheumatic fever were developed in 1944 and most recently updated in 1992. The presence of two major criteria, or one major and two minor criteria, establishes the diagnosis. Major criteria include: Carditis - including evidence of pericarditis, cardiomegaly, congestive heart failure, or mitral/aortic valvular disease; Erythema marginatum - a rash consistent of rapidly enlarging macules that assume a ring-like or crescent shape; Subcutaneous nodules - palpable small, firm, nontender nodules that present primarily over bony prominences; Sydenham chorea - involuntary choreiform movements of the face, tongue and upper extremities; and Migratory polyarthritis involving the large joints. Minor criteria include fever, arthralgias/myalgias, previous diagnosis of rheumatic fever or rheumatic heart disease, evidence of systemic inflammation (elevated ESR or C-reactive protein), plus supporting confirmation of recent streptococcal infection (positive throat culture or elevated anti-streptococcal antibody titers). Pericarditis is an inflammation of the pericardial sac surrounding the heart; it has many possible etiologies. Common presentations include acute onset of pleuritic chest pain and a friction rub. Pleurisy is an inflammation of the parietal and/or visceral pleura that lines the chest wall and the lungs. It can present as chest pain that worsens with inspiration or deep breathing (pleuritic chest pain). Endocarditis is an infection of the valvular or endocardial surface of the heart, primarily caused by bacterial or fungal pathogens. The clinical presentation varies greatly depending on the infecting organisms and the areas of the heart that are affected. Symptoms can include non-specific low-grade febrile illness; chest pain, cough, dyspnea, and arthralgias. Physical examination can reveal petechiae of the palate, conjunctivae, or peripheral extremities; painful violaceous lesions of the fingers, toes, or feet (Osler's nodes); Janeway lesions (erythematous lesions of palms and soles), subungual "splinter" hemorrhages, and Roth spots (retinal exudates). Patients can also present with signs/symptoms of embolic events. Influenza is a highly contagious viral infection that is primarily transmitted via respiratory droplets. Symptoms include a sudden onset of fever, chills, malaise, myalgias, headache, substernal soreness, cough, sore throat, and nasal congestion. While pericarditis, pleurisy, endocarditis, and influenza can all be included in the differential diagnosis of this patient, the complete clinical presentation, as well as the positive anti-streptococcal antibodies make acute rheumatic fever the most likely diagnosis.

Question A 51-year-old man presents with what he describes as "fast, shooting pains" in his legs; he is also experiencing a slight tremor when he writes or eats. In the past, he has experienced an occasional URI, but he appears to be immunocompetent and has not required treatment for any infections subsequent to these viral syndromes. His past medical history does not include any significant neurological events, but the examination today reveals a loss of reflexes in his leg and bilateral Argyll Robertson pupils, pointing to a diagnosis of general paresis. How long after infection with Treponema pallidum would these symptoms manifest themselves?

Correct answer: 10 to 20 years Explanation Treponema pallidum is the causative agent for neurosyphilis. Clinical manifestations include syphilitic meningitis, meningovascular syphilis, general paresis, and tabes dorsalis. General paresis and tabes dorsalis are considered latent manifestations because they occur 10-20 years after infection. Tabes is characterized by lightning pains, especially in the legs; however, it is a disease of the spinal cord and not a peripheral neuropathy. Other symptoms include sensory ataxia and dysuria, but chronic fever, chorea, and encephalopathic syndrome are not characteristic of the latent stage of syphilis. Symptoms of general paresis are similar to Alzheimer's disease and may be difficult to distinguish unless a tremor or the characteristic Argyll-Robertson pupil is present. Syphilitic meningitis is often classified as "early" neurosyphilis because it usually occurs within a few years after infection. Meningovascular syphilis appears 1 to 10 years after infection and probably represents the clinical manifestation of arterial involvement that occurs in all patients with chronic syphilitic meningitis. This can lead to stroke-like events and seizures. Many patients have both of these "syndromes" together, but they can certainly occur separately. Treatment for neurosyphilis consists of IV penicillin for 10-14 days; ceftriaxone (Rocephin) is an alternative drug

Case A 23-year-old man presents with a 2-day history of burning urine. He also reports a slight purulent urethral discharge. He denies any fever, malaise, or chills. He smokes 1 pack of cigarettes daily and drinks socially; he has multiple sexual partners. On exam, his vitals are normal, and lungs are clear; abdominal exam is unremarkable, without any renal angle or suprapubic tenderness, and external genitals reveal only slight urethral discharge. Labs show WBC of 6500/uL, and urinalysis has 5 - 10 WBC and 0 RBC. Gram stain of the urethral discharge shows neutrophils and intracellular Gram-negative diplococci. Question What is the best treatment regimen for this patient?

Correct answer: Ceftriaxone 250 mg IM plus azithromycin 1g PO each single dose Explanation The correct response is ceftriaxone 250 mg IM plus azithromycin 1g PO each single dose. This patient has gonococcal urethritis, which is evidenced by the diplococci in the urethral discharge. The causative organism of gonorrhea is Neisseria gonorrhoeae, which is a Gram-negative intracellular diplococcus. It is most commonly seen in people between 15 and 29 years old. It is usually transmitted during sexual activity, and it has increasing incidence in homosexual men. Multiple sexual partners, unprotected intercourse, and anal sex are important risk factors. The incubation period of Neisseria gonorrhoeae is 2 - 8 days. Other than urethritis, it can also cause epididymitis, prostatitis, proctitis, cervicitis, vaginitis, and salpingitis. Pharyngeal infection is atypical, but it is not uncommon. Disseminated disease is associated with fever, rash, arthritis, and tenosynovitis. Chronic infection can cause urethral strictures, chronic salpingitis, infertility, and chronic prostatitis. Asymptomatic infection is quite common and can occur in men and women. Chronic cervicitis is an important reservoir of infection. Gram stain of the urethral discharge shows Gram-negative diplococci inside neutrophils. The gold standard is a positive culture from any site, including the urethra, cervix, pharynx, and rectum. DNA probes can be used for urethral and endocervical specimens. Nucleic acid amplification tests are available for testing urine and urethral specimens. Testing urine in the office is quite popular with patients and staff because it is noninvasive, fairly sensitive, and specific. Such patients have a high frequency of co-infection with chlamydia; therefore, treatment for both is given at the same time. Ceftriaxone plus azithromycin is given for gonococcus, and azithromycin or doxycycline are the recommended regimens for Chlamydia. Erythromycin base, erythromycin ethylsuccinate, levofloxacin, ofloxacin, or doxycycline are acceptable alternatives to azithromycin for nongonococcal urethritis (NGU) only, but there is a distinct possibility of noncompliance with a 7-day course. A 3-day course of trimethoprim-sulfamethoxazole is suitable only for uncomplicated cystitis in women, and it would be useless in urethritis in men. Ceftriaxone alone does not cover chlamydial infections.

Case A 17-year-old boy presents to your office with a 2-week history of a red, itchy rash. The rash began on his feet, and over the last 2 weeks it appears to be migrating up his legs. The rash is intensely pruritic. There is no fever, vomiting, diarrhea, cough, weakness, or weight loss. The patient has no significant past medical history. He has just returned from a summer job in Georgia where he was working as a landscaper. There is no travel outside the United States. He denies the use of alcohol and intravenous drug use, but has smoked marijuana occasionally over the summer. On physical exam, there are serpiginous, raised, erythematous lesions streaking up both lower extremities. The rest of his exam is normal. Question What complications may occur as a result of this boy's infection?

Correct answer: Pneumonitis Explanation The correct response is pneumonitis. The patient is suffering from cutaneous larva migrans, also known as creeping eruption. It is caused by the larvae of the cat and dog hookworm (Ancylostoma braziliense and Ancylostoma caninum). These organisms are commonly found in the Southeast United States. Parasite eggs are deposited in the feces of animals and hatch to form infective larvae. The larvae penetrate the skin and migrate in the epidermal-dermal junctions at a rate of 1 - 2 cm/day. The lesions are serpiginous, raised erythematous tracks that are intensely pruritic. The eosinophilic pneumonitis associated with a large burden of parasites is often referred to as Loeffler syndrome. Other parasites that cause pneumonitis are Ascaris lumbricoides, Toxocara canis, Toxocara cati, Strongyloides stercoralis, and lung flukes. Meningitis is a complication of infection by Angiostrongylus cantonensis, the rat lungworm, and is found in Southeast Asia, Egypt, and Cuba. This organism also causes a creeping skin eruption. Peritonitis is a complication of Ascaris lumbricoides, the largest and most common human intestinal roundworm. Seizures are a complication of cysticercosis, caused by the pork tapeworm Taenia solium. Decreased visual acuity is a symptom of ocular larva migrans associated with toxocariasis (caused by Toxocara canis and Toxocara cati).

Case A 20-year-old woman on the orthopedic floor develops redness and inflammation of a knee that had ACL repair done. The knee is inflamed and tender to the touch. The surgical sites are significant for a white, pus-like drainage. There is no significant odor present, which would be indicative of a possible anaerobic infection. Drainage material was collected for Gram stain and culture. The Gram stain result report had many neutrophils present and many Gram-positive cocci. The culture grew colonies on blood agar that were beta-hemolytic (refer to the image) and catalase/coagulase-positive. Question This patient has what infection?

Correct answer: Staphylococcus aureus Explanation Staphylococcus aureus is the most common Gram-positive cocci to cause clinical disease in humans. The organism is a common cause of surgical wound infections acquired in a hospital. It can also cause pneumonia, sepsis, peritonitis, sore throats, and food poisoning. S aureus is a Gram-positive cocci appearing clusters, and it is catalase and coagulase-positive. Escherichia coli is a Gram-negative bacillus and is a member of the Enterobacteriaceae group. It is the number 1 significant clinical isolate in the microbiology laboratory. The organism is 90% lactose-positive and thus grows pink on MacConkey agar, is bile-positive on MacConkey agar, and is indole-positive and oxidase-negative. The organism is found to cause a variety of infections such as urinary tract infections, septicemia, and wound infections. Pseudomonas aeruginosa is a lactose-negative bacillus (has no pink color on MacConkey) that is indole-negative and oxidase-positive. It is a non-fermenter and not a member of the Enterobacteriaceae group. On MacConkey, it frequently has a slight blue-green color and is said to have a grape-like odor. The organism is a common cause of nosocomial infections and is usually multiply resistant to a variety of antibiotics. It can cause eye infections, urinary tract infections, wound infections, septicemia, and ear infections. Streptococcus pyogenes is a Gram-positive cocci that appears in pairs or chains under the microscope. The organism is beta-hemolytic on blood agar, catalase-negative, and is Lancefield grouped as A. The organism is a major cause of pharyngitis, especially in children where the infection can progress to rheumatic fever. It can cause necrotizing fasciitis in which severe damage is done to muscle tissue. The disease can progress into a toxic shock-like syndrome leading to death. Necrotizing fasciitis usually begins at an inconspicuous site of entry, such as a small vesicle. The host is prone to this infection due to some type of immunocompromised state. Streptococcus pneumoniae is a Gram-positive cocci that is catalase-negative. On blood agar, it is alpha hemolytic and the colonies (typically) are perfectly round with an indentation in the center of the colony. The colonies are bile soluble and susceptible to optochin (P-disc). The organism is a major cause of pneumonia, meningitis, otitis media, wound infections, and septicemia.

Case A 20-year-old woman presents with a 5-day history vaginal discharge and vulvar itching. She has had unprotected sexual intercourse with a new partner for the last month. On clinical examination, there is vulvar erythema and edema. The vaginal discharge is noted to have an unpleasant odor, a white-gray color, and a frothy appearance. The upper vagina and the cervix have a strawberry appearance with multiple erythematous epithelial papillae. The vaginal pH is 6.5. Question What test is likely to give a positive diagnostic?

Correct answer: Wet vaginal prep Explanation Trichomonas vaginalis is a sexually transmitted protozoan that causes high concomitant infections in both sexual partners. The strawberry appearance of the upper vagina and cervix is a characteristic appearance, but it is found in only a small percentage of patients. Microscopic examination of wet vaginal prep shows the characteristic motile protozoan with an undulating membrane and flagella. A single dose of 2g metronidazole is the treatment of choice for both partners. Culture on Sabouraud agar is the most sensitive test to diagnose the vaginal yeast infections. The adherent white plaques and the cottage cheese-like discharge are characteristic for vaginal infections with Candida albicans. Dark field microscopy and rapid plasma reagins are used to diagnose the syphilis caused by Treponema pallidum. The characteristic lesion in the primary stage is the painless, firm ulcer. Tzanck smear shows multinucleated giant cells from vesicles and ulcers in genital infections with herpes virus.

Question A 20-year-old college student spent his summer working in the forest on eastern shore of Maryland. A month after returning home to Virginia, he experienced a sudden onset of fever, nausea, muscle aches, and headaches. Blood and stool cultures were negative for obvious bacterial and parasitic pathogens. His symptoms subsided, and he assumed he had recovered. Two weeks later, the symptoms reappeared, now accompanied by marked splenomegaly. He also noticed a single, spreading rash with a pale halo surrounding a brighter red rash (Figure K1.3) on the back of his right shoulder which increased in size and sensitivity. The etiologic agent responsible for the infection was MOST likely acquired by which of the following methods?

Correct answer: a tick bite Explanation Lyme disease, first detected in Lyme, Connecticut, is a bacterial infection caused by the spirochete Borrelia burgdorferi. It is a zoonotic (animal-to-human) infection within the deer and rat populations and accidentally transmitted to man through the bite of the deer tick, Ixodes dammini. Different arthropod vectors are instrumental in the transmission of other Borrelia infections. The extremely small size (< 2 mm) of deer ticks makes them difficult to detect, compared to the larger wood tick, Dermancentor andersoni, or dog tick, D. varibilis. A large, circular, red petechial rash usually develops at the site of the tick bite. It may grow to over 10 cm in diameter and can become surrounded by an even more extensive halo of less intense rash. The bacteria may cause joint pain (arthlagia) resulting in a form of arthritis. The primary lesion grows into an enlarged rash as the spirochete proliferates and invades the blood and lymphatic vessels. The lesion usually disappears after several weeks. Some individuals progress into a second stage of infection involving neurologic or cardiac abnormalities. An arthritic response may develop months to years after the initial infection, probably due to an autoimmune response. This is indicative of the third stage of Lyme disease. Relapsing fever is another disease caused by bacteria of the genus Borrelia. It can be either epidemic (louse-borne) or endemic (tick-borne). Syphilis and leptospirosis are diseases also caused by different spirochetes.

Question A 24-year-old woman started her new job as an elementary school teacher. She graduated from a college in Colorado and looked forward to her first full-time job. During the winter months, many of her students became ill with fevers, runny noses, and headaches. The teacher also experienced the same symptoms, which lasted several days then disappeared. A week or so later, the illness reappeared with similar symptoms, sometimes with a sore throat. Some students received antibiotics. The symptoms described above are characteristics of what condition?

hide Correct answer: Common cold Explanation Seasonal upper respiratory infections (URI) that occur during the winter months are often features of the common cold. Young and older individuals may experience a runny nose, sore throat, nasal congestion, headaches, and sneezing. The severity of the URI will vary with different individuals. Levels of partial immunity, induced by prior infections, may alter the severity and duration of the infection; in some instances, they may totally prevent infection. The average individual is infected 2-3 times per year because multiple strains of the infectious agents circulate in the population at any given time. Common colds are caused by a variety of viruses, such as rhinoviruses, coronaviruses, and adenoviruses. Most URIs are caused by rhinoviruses. Infections spread easily via respiratory droplets produced during sneezing and breathing. More severe respiratory infections are caused by the orthomyxoviruses (influenza viruses). There are over a hundred different viral serotypes capable of causing common colds. Therefore, it is not practical to produce a vaccine. It is also impractical because the infections are generally brief and mild. Older individuals and those who are immunosuppressed are at risk for complications. Antibiotics, often requested by concerned parents, are ineffective against viral cold infections. Generally, the infections are treated symptomatically in order to ease the discomfort of the patient until the immune system takes over.

Case A 26-year-old man, residing in Thailand, presents with a 7-day history of high-grade fever, dull frontal headache, malaise, anorexia, and vague abdominal discomfort. He has mild diarrhea, dry cough, and myalgia. On examination, temperature is 39°C and pulse is 65 per minute. He has a coated tongue, tender abdomen, and a soft, palpable spleen. Clinical diagnosis is enteric fever. Appropriate clinical samples are sent for culture and serology. Question What laboratory data would be most helpful in making a definitive diagnosis?

Correct answer: Blood culture positive for S. typhi Explanation Definitive diagnosis depends on a positive blood culture for Salmonella typhi. The term 'enteric fever' includes typhoid caused by the Salmonella enterica serotype, typhi (referred to as S. typhi), and paratyphoid caused by Salmonella paratyphi A, B, and C. Isolation of S. typhi from stool and urine samples helps in the diagnosis, but could also be positive in carriers. Due to various factors, the interpretation of agglutinin titers detected in a single serum sample is difficult. In enteric fever, the infection is acquired by ingestion. After reaching the small intestine the bacteria invade the mucosa, reaching the mesenteric lymphoid follicles and the draining mesenteric lymph nodes. Some pass to the reticuloendothelial cells of the spleen and liver where Salmonellae are able to survive and multiply within the mononuclear phagocytic cells. Depending on the virulence of the bacteria and host response, bacteria are released into the blood stream and become widely disseminated. The bacilli are seeded in the liver, spleen, gall bladder, bone marrow, and Peyer's patches of the terminal ileum, where further multiplication occurs. The lungs and kidneys may be involved. Toward the end of the incubation period (7-14 days) massive bacteremia occurs, leading to the onset of clinical symptoms. During the first week of clinical disease, Salmonella can be isolated from blood culture in 60-80% of patients. As the humoral and cellular responses of the host come into action, the chances for isolation from the blood gradually decrease. Since multiplication of organisms occurs in the intestinal lymphatic tissue and in the gall bladder, Salmonellae are excreted in feces. The stool culture positivity rate is greater during the second and third weeks of illness. When Salmonellae localize in the kidneys, they are excreted in urine (infrequently and in small numbers). It is possible to isolate the organisms from urinary deposits by inoculating enrichment media. The sensitivity of the blood culture is reduced by prior administration of antibiotics. In such patients, bone marrow culture is very useful for the isolation of the bacteria. Positive results are obtained in 80-95% of patients with typhoid and are not affected by the duration of the illness and/or prior antibiotic therapy. DNA probes and PCR-based tests have been developed to detect S. typhi directly in blood, though these are not available for use in many areas where typhoid is common. Prevalence of multidrug-resistant (MDR) S. typhi (e.g., resistant to chloramphenicol, ampicillin, and trimethoprim/sulfamethoxazole) is a major problem faced by countries where typhoid is endemic. The resistance is plasmid-mediated. Third-generation cephalosporins and fluoroquinolones are recommended for treatment of infections by MDR strains. Recently, the emergence of strains with reduced susceptibility to fluoroquinolones has been noticed. This is chromosomally-acquired resistance and is largely due to mutations in the gyrA gene. Development of resistance to ceftriaxone has been reported, though is rare in occurrence. In making appropriate decisions regarding the choice of an antibiotic for the treatment of typhoid, susceptibility testing of the isolate is very important. A positive stool culture is helpful in diagnosis, though it is not always confirmatory. Fecal carrier state is common after clinical, as well as subclinical, infection by Salmonella typhi. In fecal carriers, Salmonellae persist in the gall bladder and the biliary tree, and they are excreted in the feces (intermittently). The chronic carrier state is seen more commonly in patients with cholelithiasis. Demonstration of Vi agglutinins in the serum and culture of stool and bile samples are useful for detection of carriers. Urine culture can be positive in urinary carriers; urinary carriers are less frequent than fecal carriers. The carrier state is generally associated with some abnormality of the urinary tract (e.g., calculi or schistosomiasis). O (somatic) and H (flagellar) agglutinins against Salmonella typhi and paratyphi are tested by Widal agglutination assessment. A single serum sample test result has no diagnostic significance. Serum agglutinins usually appear by the end of the first week and rise sharply during the second and third weeks of Salmonella infection. A rise in the titer of agglutinins is indicative of active infection, and it is demonstrated by testing at least 2 samples of serum that are taken at an interval of 7-10 days. Demonstration of a 4-fold rise in the titer of agglutinins against H and O antigens of S. typhi can be helpful in diagnosing typhoid during later stages of the disease when a blood culture is often negative. Though a high titer of O agglutinins could be suggestive of recent infection, the possible presence of cross-reacting antibodies limits the utility of this test. H agglutinins are influenced by a previous infection and past immunization against typhoid. They persist for a longer period than O agglutinins. Though the Widal test is still used for the diagnosis of enteric fever in countries where the disease is endemic, the test faces controversies. Widal test results are influenced by a variety of factors (e.g., previous antibiotic therapy, previous immunization with Salmonella antigens, and cross-reacting antibodies to other infectious agents); therefore, the interpretation of the test should be done with caution. Information on the distribution of agglutinin levels in normal sera and in different geographical areas may help to assess the significance of the test results. During recent years, rapid tests such as Dot blot and DOT enzyme immunoassay for serodiagnosis of enteric fever have been developed and evaluated. The results are promising, and these tests are reported to be of use in regions where typhoid is endemic and laboratory facilities are limited.

Question A 20-year-old man spent his summer working in the forest on eastern shore of Maryland. A month after returning home to Virginia, he experienced a sudden onset of fever, nausea, muscle aches, and headaches. Blood and stool cultures were negative for obvious bacterial and parasitic pathogens. His symptoms subsided and he assumed he had recovered. 2 weeks later, the symptoms reappeared, now accompanied by marked splenomegaly. He also noticed a single, spreading rash with a pale halo surrounding a brighter red rash on the back of his right shoulder. It has been increasing in size and sensitivity. This time, a blood smear showed the presence of spirochetes. What is the MOST probable etiologic agent for the above illness?

Correct answer: Borrelia Explanation Lyme disease, first detected in Lyme, Connecticut, is a bacterial infection caused by the spirochete Borrelia burgdorferi. It is a zoonotic (animal-to-human) infection within the deer and rat populations and accidentally transmitted to man through the bite of the deer tick, Ixodes dammini. Different arthropod vectors are instrumental in the transmission of other Borrelia infections. The extremely small size (< 2 mm) of deer ticks makes them difficult to detect, compared to the larger wood tick (Dermacentor andersoni) or dog tick (D. variabilis). A large, circular, red petechial rash usually develops at the site of the tick bite. It may grow to over 10 cm in diameter and can become surrounded by an even more extensive halo of less intense rash. The bacteria may cause joint pain (arthlagia) resulting in a form of arthritis. The primary lesion grows into an enlarged rash as the spirochete proliferates and invades the blood and lymphatic vessels. The lesion usually disappears after several weeks. Some individuals progress into a second stage of infection involving neurologic or cardiac abnormalities. An arthritic response may develop months to years after the initial infection, probably due to an autoimmune response. This is indicative of the third stage of Lyme disease. Relapsing fever is another disease caused by bacteria of the genus Borrelia. It can be either epidemic (louse-borne) or endemic (tick-borne). Syphilis and leptospirosis are diseases also caused by different spirochetes.

Question A 17-year-old girl presents with her boyfriend due to vaginal discharge. She has not had any previous episodes and she states that she has been in a monogamous relationship with her boyfriend. She takes the oral contraceptive pill. The couple does not regularly use condoms. Her last menstrual period was 2 weeks ago. When reviewing her charts, you notice that she has not showed for several of her appointments in the past. On physical exam, she is comfortable. Her temperature is 98 degrees Fahrenheit (36 Celsius), blood pressure of 120/80 mmHg, pulse 72/minute and respirations of 16/minute. You do a speculum examination that confirms a mucopurulent discharge. You obtain a cervical swab. The manual exam does not demonstrate cervical motion tenderness or adnexal tenderness. The cervical swab is sent for a polymerase chain reaction analysis, which returns positive for chlamydia trachomatis. At this stage, what is appropriate therapy for this patient?

Correct answer: Azithromycin 1 gram in a single dose Explanation Chlamydial infection is seen in young adults and adolescents, and it should be thought of when seeing sexually active adolescent patients in the office. Although this patient presented with vaginal discharge, the infection may be asymptomatic. One should also consider dual treatment for gonorrhea since Chlamydia might coexist with other sexually transmitted disease. This teenage patient has missed her previous appointments; therefore, whether or not she will return is debatable. Azithromycin has the advantage of single-dose therapy, therefore enhancing compliance. Follow-up cervical swabs for chlamydial analysis and cultures should be obtained to document success of therapy (if the patient returns), particularly in adolescents. Although doxycycline and erythromycin are appropriate choices, the patient may not complete the course as indicated. Careful assessment for sexually transmitted diseases, counseling about HIV infection, and condom use should be given along with appropriate pharmacotherapy. One should consider metronidazole in recurrent or more persistent disease. One should also treat the sexual partner as well as the patient, and both partners should abstain from sex for some time.

Case A 24-year-old woman presents for follow-up 7 days after a medical abortion with mifepristone and misoprostol; she has a fever, chills, and abdominal pain. Examination shows a temperature of 38°C, tachycardia, slight vaginal bleeding, and sanguinopurulent cervical discharge. Blood cultures are negative, and pre-existing Chlamydia trachomatis is suspected. Question What is the best course of action for this patient?

Correct answer: Azithromycin with ceftriaxone Explanation The risk of infection following medical abortion is less than surgical abortion, making prophylactic antibiotics unnecessary. Infection risk is increased with retained tissue and pre-existing sexually transmitted diseases (STDs). Bacterial vaginosis and Chlamydia are associated with increased risk of infection 4-5 days following medical abortion. Azithromycin and ceftriaxone is an effective treatment for postabortal endometritis. For serious infections, curettage and surgical drainage are essential. While rare, clostridial sepsis may occur; it is treated with high-dose penicillin. Prostaglandins are contraindicated in the presence of acute infection, but 15-methyl prostaglandin F2α may be used with curettage of retained products of conception.

Question A 35-year-old woman with acute lymphocytic leukemia and undergoing chemotherapy suddenly developed small vesicular eruptions across her left breast. The eruptions were asynchronous, painful, and lasted 6 days before clearing. She scratched the affected area, which only intensified her discomfort. The affected area developed small vesicles that progressed to ulcerated and crusted pustules, causing her clothing to stick to the open sores. She had a similar occurrence last year. Appearance of her current symptoms most likely occurred through which of the following pathways?

Correct answer: Activation from latency Explanation The symptoms in this are most consistent with shingles, which is caused by the herpes zoster virus. Herpes zoster virus is identical to the varicella virus, which causes chickenpox. Exposure to the varicella virus results in the establishment of a latent state in many individuals. The virus may remain dormant for many years before being activated and expressed as a different disease, shingles. Each year, over 300,000 individuals experience severe episodes of shingles. There are numerous factors that have been attributed to the reactivation of the herpes zoster virus. Exposure to sunlight, chemotherapy, immunosuppression, menstrual cycle changes, trauma, depression, and anxiety are but a few of the factors reported to induce the expression of shingles. Rash-like lesions associated with each episode range from undetectable to severe. Scratching can cause secondary bacterial infections, which exacerbate the situation. Shingles affects a specific dermatome (portion of the body innervated by a segmental sensory dorsal root ganglion). Fluid recovered from the lesions contains active virus, which is capable of infecting unexposed individuals. The patient rarely autoinfects with new dermatomal presentation. Both humoral and cell-mediated immune responses restrict the spread of the virus to new body sites; however, since it is sequestered in protected ganglia, the responses are unable to eradicate the virus from the body. Interferon may stabilize the infection. Current treatments do not eliminate the infection; they only reduce the severity of the episode. Hyperimmune globulin is effective in suppressing varicella infections in cases of known exposure. Immunosuppressed individuals are at risk from varicella; therefore, they would benefit from passive immunization. The attenuated chickenpox (herpes zoster) vaccine, first introduced in 1995, is relatively safe and effective. Prior to the introduction if this vaccine, over 12,000 people were hospitalized annually due to severe cases of chickenpox.

Case A 10-year-old boy presents with chest pain and joint swelling. His mother says about 3 weeks ago he had a sore throat with fevers and cervical lymphadenopathy. He was prescribed penicillin, but stopped taking it after a day because of nausea. His symptoms soon resolved. About a week ago, he complained of knee pain and swelling and later had elbow pain and swelling. He also experienced chest pain and recurrence of his fever. His mother is concerned. His vital signs are blood pressure 110/70 mm Hg, heart rate 120/min, temperature 100.1°F. On physical examination, normal S1/S2 and a II/VI holosystolic murmur is heard, and mild knee tenderness without swelling is present. Elbow still has swelling and pain. His EKG shows a sinus tachycardia with a prolonged PR interval. His chest X-ray is normal. Question What is the most likely diagnosis?

Correct answer: Acute Rheumatic fever Explanation The clinical picture is suggestive of acute rheumatic fever. Peak incident occurs between the ages of 5-15 years old, usually after a streptococcal infection of the pharynx. Signs of rheumatic fever usually commence 2-3 weeks after infection. Diagnosis is based on Jones criteria and confirmation of streptococcal infection. The presence of 2 major criteria, or 1 major and 2 minor criteria, establishes the diagnosis. Major criteria include carditis, erythema marginatum and subcutaneous nodules, Sydenham's chorea, and polyarthritis. Minor criteria include fever, polyarthralgias, reversible prolongation of the PR interval, and an elevated erythrocyte sedimentation rate or C-reactive protein. The clinical presentation of infective endocarditis depends on the infecting organisms and the valve or valves being infected. The clinical diagnosis is based on the modified Duke criteria, which include 2 positive blood cultures for a microorganism that typically causes infective endocarditis. Positive blood cultures are not indicative in this patient. With pericarditis, the patient does present with chest pain, but it is usually described as sharp, retrosternal with radiation to the back, and worse with deep breathing or coughing. The pain is position dependent: worse when lying flat, and improved while sitting up and leaning forward. On physical examination, the pericardial rub is pathognomonic of pericarditis, which is not present in this patient. Kawasaki's disease is characterized by fever and 4 of the following symptoms for at least 5 days: bilateral painless, nonexudative conjunctivitis; lip and oral cavity changes; cervical lymphadenopathy > 1.5cm and usually unilateral; polymorphous exanthema; and extremity changes. 80 percent of patients are < 5 years old. Most of these symptoms are not present in this patient. The typical clinical description of Lyme disease divides the illness into 3 stages: stage 1, flu-like symptoms and a typical skin rash; stage 2, a week to months later, Bell's palsy or meningitis; and stage 3, months to years later, arthritis. Exposure to the Ixodes tick in certain geographic locations is needed for infectivity. There is no history of tick exposure or flu-like symptoms and rash in this patient.

Case A 52-year-old man is hospitalized for a left lower lobe pneumonia. The patient has a past medical history of HIV and is known to be neutropenic. He also has type 2 diabetes mellitus and diabetic nephropathy. He is started on ciprofloxacin, ceftriaxone, and clindamycin. However, his fever does not resolve, and his blood cultures have been negative. The physician notes a white coating of the tonsils and oropharynx. The patient is able to eat normally. The physician obtains scrapings of the white coating. The sample is observed microscopically and confirms the presence of fungal hyphae and budding. The physician suspects a candidal infection of the oropharynx. Question What is the next step in patient management?

Correct answer: Begin oral fluconazole immediately Explanation This patient has a candidal infection. Of the antifungal therapies available, fluconazole is generally better tolerated than amphotericin B. The side effects are rare and include nausea, vomiting, rash, abdominal pain, and hepatotoxicity. It has almost equal bioavailability in the oral and intravenous routes. This patient has no underlying gastrointestinal disorder and would benefit from getting the oral fluconazole. The normal locations for Candida are the gastrointestinal tract, oropharynx, and vagina. The most common site to isolate Candida in hospitalized patients is in urine. The morbidity rate of candidiasis with symptoms (once blood borne) is high in hospitalized patients. This becomes more serious in the presence of an abdominal abscess, endocarditis, endophthalmitis (seen as cotton-wool exudates on funduscopic exam), myocarditis, esophagitis, pneumonia, peritonitis, and thrush. Candida can also be indicated by fever despite antibiotic therapy, hypotension, leukocytosis, colonization in a wound, or post-operative surgical site and suppurative phlebitis. Thrush in this patient is often a marker of HIV, overzealous use of antibiotics, or steroids. Risk factors for candidiasis include using more than 3 antibiotics simultaneously, using broad-spectrum antibiotics for longer than 4 days, a stay of longer than 4 days in the intensive care unit, the use of mechanical ventilation for longer than 48 hours, abdominal surgery, neutropenia, diabetes mellitus, and immunosuppression. In cancer patients, thrush is often a marker for esophageal candidiasis. The presence of Candida in this patient cannot be ignored due to the potential for prolonged hospital stay and subsequent high health costs; there is rapid progression of the organism once blood borne; there is also high mortality rate. Once Candida is suspected, specimen cultures should be obtained from the oropharynx, sputum, stool, urine, and surgical sites. Antifungal therapy needs to be initiated immediately. Once the culture and sensitivities are available, then the therapy can be adjusted. One should not wait for the culture results before starting antibiotic therapy. Amphotericin B has acute toxicity, and because of this it often needs to be stopped prematurely. Important side effects include fever, rigors, hypotension, anorexia, and tachypnea. A chronic side effect is renal toxicity. Since the patient has diabetic nephropathy, it should not be administered to him.

Question What body fluid is considered potentially infectious for HIV as well as hepatitis B and C?

Correct answer: CSF (cerebral spinal fluid), non-bloody Explanation The correct response is CSF (cerebral spinal fluid), non-bloody. Any body fluid that is contaminated by blood is potentially infectious. In addition, other body fluids considered to be potentially infectious include semen, vaginal secretions, CSF, synovial fluid, plural fluid, peritoneal fluid, and amniotic fluid. Those body fluids considered by the CDC to be non-infectious include feces, nasal secretions, saliva, sputum, vomitus, sweat, tears, and urine.

Case A 26-year-old woman presents for her second obstetric visit in the first trimester. Routine screening tests (blood typing, testing for syphilis, hepatitis, rubella immunity, and HIV) are performed; the test returns positive for HIV. She is counseled to start antiretroviral therapy and to have a cesarean delivery. Question To reduce the risk of mother-to-newborn transmission, the best drug treatment is a drug that prevents what mechanism of the infection?

Correct answer: Virus replication Explanation All pregnant HIV-infected patients beyond 14 weeks' gestation should be on a highly active antiretroviral therapy (HART) regimen. With cesarean delivery and appropriate antiretroviral therapy, risk of transmission of HIV to the newborn is less than 1%; the risk would be 25% without such a treatment. Zidovudine is the only anti-HIV drug that is fully approved for use during pregnancy. It is inhibits HIV's reverse transcriptase and is placed within the viral DNA. When placed in the viral DNA by the reverse transcriptase, transcription of the viral genes is inhibited. This prevents virus replication. Other drugs that belong to this group (abacavir, emtricitabine, didanosine, zalcitabine, lamivudine, tenofovir, reverset, and stavudine) are not approved because of side effects. Protease inhibitors (amprenavir, atazanavir, fosamprenavir, ritonavir, indinavir, nelfinavir, saquinavir, and tipranavir) inhibit the retroviral protease from cleaving the viral proteins. HIV produces its own proteases that play role in the production of infective viral particles by cleaving viral proteins to sizes of a mature viral particle (viral assembly). Protease inhibitors inhibit proteases from cleaving the viral proteins; in doing so, they reduce the spread of the virus to other uninfected cells. They are not approved in pregnancy because of their serious side effects. Enfuvirtide interferes with the viral gp41 protein and prevents fusion of HIV with the host cell. It is used in combination therapy with other antiretrovirals when all other treatments have failed. Maraviroc and other CCR5 entry inhibitors act on the human T-cell, changing it in such a way that HIV is unable to bind and therefore continue the replication process. They interfere with the host immune system and cause serious side effects; therefore, they are not approved in pregnancy. Integrase inhibitors, like raltegravir, inhibit the enzyme that integrates HIV genetic material host. Since the integration is a distinct step in HIV life cycle, integrase inhibitors may be taken in combination with other HIV drugs to minimize viral drugs resistance. They are also useful in the case of the resistance to other drugs. Due to their adverse effects, they are not recommended in pregnancy.

Case A 23-year-old man presents with a 2-day history of watery nasal discharge, malaise, sneezing, and nasal congestion. On examination, you notice inflammation of the nasal mucosa. Pulse is 80/min. BP is 130/80 mm Hg, and temperature is 98.8 °F. Question What investigations will you order to establish the diagnosis?

Correct answer: No specific diagnostic workup needed Explanation The correct answer is no specific diagnostic workup needed. This patient is suffering from viral rhinitis (common cold). No specific workup beyond a clinical diagnosis is necessary for an otherwise healthy patient1. Nasal swab with polymerase chain reaction PCR can help establish a specific viral etiology, if needed. Complete blood count may show increased total leukocyte count. Viral serology establishes a specific viral etiology. X-rays may be positive in cases of sinus involvement. These investigations are not done routinely to establish a diagnosis, but may be necessary in specific clinical scenarios, such as in patients who are immunocompromised.

Question Following an uncomplicated pregnancy, a 34-year-old woman gives birth to a healthy boy. 2 days after birth, the woman develops a low-grade fever and vesicular lesions on her face and abdomen. History reveals an outbreak of chickenpox in her 2-year-old daughter's day care facility. The mother is diagnosed with chickenpox. What is the best management for the newborn?

Correct answer: Varicella-zoster immune globulin Explanation If birth is 2 - 3 weeks after onset of clinical varicella, it is less likely the neonatal infection will be serious. However, if birth is <1 week after, or up to 2 days before maternal varicella, neonatal chickenpox is likely to be severe. In this case, infants should be passively immunized with varicella-zoster immune globulin (VZIG). All premature infants should receive VZIG if maternal varicella is present, regardless of the timing of infection. Risk of varicella transmission by breast-feeding is unknown. Perinatally acquired varicella may be life-threatening. Acyclovir can be used to treat varicella infection in neonates. Giving birth within 1 week before or after the mother has clinical varicella may result in neonatal disease, which is often severe. Varicella transmission may be intrauterine or perinatal. Maternal anti-VZV antibody protects the fetus; accordingly, the severity of neonatal varicella reflects the interval between parturition and maternal onset of chickenpox. Varicella (chicken pox) is a childhood disease caused by varicella-zoster virus (VZV), a herpes virus. Although it is thought to be a mild disease, there are approximately 10,000 varicella-related hospitalizations and 100 deaths annually in the United States. In children, secondary skin infections are the most common complication of varicella. Children with chickenpox have a 3-fold increase in risk for group-A streptococcal infections and sequelae, including necrotizing fasciitis and toxic shock syndrome. Central nervous system complications in children include cerebellar ataxia (1 in 4,000) and encephalitis (1 in 50,000). Varicella is more serious in adults than in children; adults represent 5% of cases and 45% of deaths due to VZV. Hospitalization for varicella among adults is most often a result of varicella pneumonia. Typically viral, this complication occurs in about 15% of healthy adults who contract varicella. In immunocompromised individuals, varicella may represent a life-threatening illness. Varicella is frequently an initial presenting infection in people with HIV disease. About 25% of pregnant women who contract varicella will have an infected fetus, and up to 2% of infected fetuses are clinically affected. Congenital varicella syndrome is a serious condition because VZV is teratogenic. The greatest risk occurs during development of innervation of the limbs and eyes (weeks 8 - 20). The benefit of VZIG in preventing fetopathy is unknown, as is the safety and efficacy of maternal administration of antiviral agents such as acyclovir. Varicella can be prevented with a vaccine that consists of an attenuated strain of VZV. Varicella immunization is recommended for all children 12 months to 18 years of age regardless of varicella history, and for all susceptible children 19 months to 12 years old who have not been immunized previously. Adults may also be vaccinated. HIV-infected children who are asymptomatic and have no evidence of immunosuppression (>25% CD4+ T-cells) should receive the varicella vaccine at age 12 - 15 months, or older.

Case A 33-year-old man presents with a 2-day history of severe diarrhea and vomiting. He had been on a business trip to Asia 3 days prior to presentation, and he reports eating food bought from street vendors. He describes his stools as watery and not bloodstained. He is allergic to seafood, and he takes antacids for peptic ulcers. On examination, he is moderately dehydrated; temperature is 37 C, PR is 100, and BP is 120/60 mm Hg. Question What pathogenic organism is most likely causing his symptoms?

Correct answer: Vibrio cholera Explanation This patient's clinical picture is suggestive of cholera. Cholera is caused by Vibrio cholerae, which is a small, Gram-negative bacillus that produces an enterotoxin, which stimulates adenylate cyclase, resulting in increased secretion of fluids and electrolytes. Cholera is spread by ingestion of water, seafood, and other foodstuffs contaminated by the feces of symptomatic or asymptomatic food handlers. It is endemic in parts of Asia, the Middle East and Africa, but persons living in these areas gradually acquire a natural immunity. There is increased susceptibility to infection in persons with hypochlorhydria and achlorhydria or taking antacids, as the bacilli are sensitive to gastric acid. The incubation period is 1 to 3 days. Patients usually present with painless and profuse watery diarrhea and vomiting of sudden onset. The resultant severe water, sodium, chloride, bicarbonate, and potassium depletion, results in intense thirst, muscle cramps, hypovolemia, oliguria, and anuria with severe metabolic acidosis. If untreated, it can result in circulatory collapse. Uncomplicated cholera is self-limiting, and recovery occurs in 3 to 6 days. Severe cases have a high mortality rate, which is usually due to dehydration. The diagnosis is confirmed by isolation of V. cholerae in cultures from direct rectal swabs or fresh stools, and its subsequent identification as serogroup 01 or 0139 through agglutination by specific antiserum. The treatment is intravenous replacement of fluids and electrolytes, such as potassium, with correction of metabolic acidosis. Early treatment with an effective oral antimicrobial eradicates the V. cholerae, reduces stool volume, and stops diarrhea within 2 days. Drugs effective for susceptible strains include tetracycline, doxycycline, erythromycin, trimethoprim-sulfamethoxazole, and norfloxacin. Most patients are free of V. cholerae within 2 weeks, but a few become chronic biliary tract carriers. To control cholera, human excrement must be properly disposed of, and water supplies must be purified. Drinking water should be boiled or chlorinated, and vegetables and fish should be cooked thoroughly. Patients with staphylococcus aureus food poisoning present with severe abdominal pain and severe vomiting of sudden onset, usually 2 to 8 hours after ingestion of the foodstuffs, which are usually meat and dairy products. Patients with bacillus cereus food poisoning present with sudden onset vomiting usually 2 to 8 hours after ingestion of the contaminated food. This type of food poisoning is commonly associated with ingestion of reheated fried rice. Escherichia coli serotype O157:H7 infection usually begins acutely with severe abdominal cramps and watery diarrhea, which may become grossly bloodstained within 24 hours. This type of food poisoning is associated with ingestion of raw beef, unprocessed milk, and dirty water. Vibrio parahaemolyticus food poisoning is associated with ingestion of saltwater seafood. The patients usually present with fever, vomiting, and severe diarrhea. Patients with bacillus cereus food poisoning present with the sudden onset of vomiting; symptoms typically start 2 to 8 hours after ingestion of the contaminated food. This type of food poisoning is commonly associated with ingestion of reheated fried rice. Escherichia coli serotype O157:H7 infection usually begins acutely with severe abdominal cramps and watery diarrhea; the diarrhea may become grossly bloodstained within 24 hours. This type of food poisoning is associated with ingestion of raw beef, unprocessed milk, and dirty water.


संबंधित स्टडी सेट्स

MGT 300: Chapter 14, MGT 300: Chapter 15, MGT 300: Chapter 16

View Set

Tissues Chapter 4 - Checking Understanding

View Set

Egg, Milk and Cheese test review

View Set

TA-270: Small and Medium Entities (SME's)

View Set

Vocabulary for John Lewis' Letter to Young People

View Set

Unit 9.1 Definitions Under The uniform Securities Act (Series 65)

View Set